You are on page 1of 128

To fly, we have to have resistance

Volume - 5 Issue - 10
April, 2010 (Monthly Magazine)

Editorial / Mailing Office :


112-B, Shakti Nagar, Kota (Raj.) 324009
Tel. : 0744-2500492, 2500692, 3040000
e-mail : xtraedge@gmail.com
Editorial
Editor : Dear Students,

Pramod Maheshwari Motivate Yourself


[B.Tech. IIT-Delhi] One of the greatest virtues of human beings is their ability to think
and act accordingly. The emergence of the techno savvy human from
Cover Design the tree swinging ape has really been a long journey. This transition
Govind Saini, Om Gocher
has taken a span of countless centuries and lots of thinking caps have
been involved. Inquisitiveness and aspiration to come out with
Layout : the best have been the pillars for man's quest for development.
Self-motivation is the sheer force, which pulled him apart and
Mohammed Rafiq distinguished him from his primitive ancestors.
Many times, in our life, when we are reviving old memories we get
Circulation & Advertisement into a phase of nostalgia. We feel that we could have done better
Ankesh Jain, Praveen Chandna than what we had achieved. But thinking back won’t rewind the
tireless worker called time. All we can do is promise ourselves that
Ph (0744)- 3040007, 9001799502
we will give our very best in the future. But do we really keep up to
Subscription
our mental commitments? I can guess that 90% answers are in the
negative. This is because of that creepy careless attitude which is
Sudha Jaisingh Ph. 0744-2500492, 2500692 slowly, but surely entering into our mind. We easily forget the pains
© Strictly reserved with the publishers of yesterday to relish the joys of today. This is the only time in our
life, when we can control our fate, by controlling our mind. So it is
time to pull up our socks and really motivate ourselves so that we
• No Portion of the magazine can be can give our best shot in the future. Self-motivation is the need of the
published/ reproduced without the written
hour. Only we can control and restrict ourselves. It’s up to us, how
permission of the publisher
we use our mental capabilities to the best of our abilities.
• All disputes are subject to the exclusive Here are some Funda's for self-motivation. Don't just read them
jurisdiction of the Kota Courts only.
digest each one of them and apply them and I bet it will make a
Every effort has been made to avoid errors or better YOU.
omission in this publication. In spite of this, errors • The ultimate motivator is defeat. Once you are defeated, you
are possible. Any mistake, error or discrepancy
noted may be brought to our notice which shall be have nowhere to go except the top.
taken care of in the forthcoming edition, hence any • Then only thing stopping you is yourself.
suggestion is welcome. It is notified that neither the • There is no guarantee that tomorrow will come. So do it today.
publisher nor the author or seller will be
responsible for any damage or loss of action to any • Intentions don't count, but action's do.
one, of any kind, in any manner, there from. • Don't let who you are, stunt what you want to be.
• Success is the greatest motivator.
Unit Price Rs. 20/- • Your goals must be clear, but the guidelines must be flexible.
Try to include these one liners in your scrapbook or on your favorite
Special Subscription Rates
poster. You will be sub-consciously tuned to achieve what you want.
6 issues : Rs. 100 /- [One issue free ]
Also do keep in mind that nothing can control your destiny but you!
12 issues : Rs. 200 /- [Two issues free]
With Best Wishes for Your Future.
24 issues : Rs. 400 /- [Four issues free]

Yours truly

Owned & Published by Pramod Maheshwari,


112, Shakti Nagar, Dadabari, Kota & Printed
by Naval Maheshwari, Published & Printed at Pramod Maheshwari,
112, Shakti Nagar, Dadabari, Kota. B.Tech., IIT Delhi
Editor : Pramod Maheshwari

XtraEdge for IIT-JEE 1 APRIL 2010


XtraEdge for IIT-JEE 2 APRIL 2010
Volume-5 Issue-10
April, 2010 (Monthly Magazine) CONTENTS
INDEX PAGE
NEXT MONTHS ATTRACTIONS
Regulars ..........
Key Concepts & Problem Solving strategy for IIT-JEE.

Know IIT-JEE With 15 Best Questions of IIT-JEE NEWS ARTICLE 4


Challenging Problems in Physics, Chemistry & Maths • President buries 'Time Capsule' on IIT Kanpur
campus
Much more IIT-JEE News.
• IITs admission criteria set for an overhaul
Xtra Edge Test Series for JEE – 2010 & 2011
IITian ON THE PATH OF SUCCESS 6
Mr. Krishnamurthy Rengarajan
KNOW IIT-JEE 7
Previous IIT-JEE Question

Study Time........
DYNAMIC PHYSICS 14

Success Tips for the Month 8-Challenging Problems [Set# 12]


Students’ Forum
Physics Fundamentals
• "The way to succeed is to double your Calorimetry, K.T.G., Heat Transfer
error rate." Atomic Structure, X-Ray & Radio Activity
• "Success is the ability to go from failure to
CATALYST CHEMISTRY 30
failure without losing your enthusiasm."
• "Success is the maximum utilization of the Key Concept
ability that you have." Aromatic Hydrocarbon
Solubility Product
• We are all motivated by a keen desire for Understanding: Organic Chemistry
praise, and the better a man is, the more
he is inspired to glory. DICEY MATHS 38
• Along with success comes a reputation for Mathematical Challenges
wisdom. Students’ Forum
Key Concept
• They can, because they think they can.
Calculus
• Nothing can stop the man with the right Algebra
mental attitude from achieving his goal;
nothing on earth can help the man with Test Time ..........
the wrong mental attitude.
XTRAEDGE TEST SERIES 52
• Keep steadily before you the fact that all Mock Test IIT-JEE Paper-1 & Paper-2
true success depends at last upon yourself. Mock Test AIEEE
Mock Test BIT SAT
SOLUTIONS 92

XtraEdge for IIT-JEE 3 APRIL 2010


President buries 'Time IITs admission criteria set research. Although the admission
Capsule' on IIT Kanpur for an overhaul criteria is becoming stringent with
the proposal of IIT-Kharagpur
campus New Delhi: The admission criteria
Director's recommendation, the
for admission to the premier
HRD minister retreated with a
engineering institutes of the
firm 'No' to IIT-Kanpur Director's
country - the Indian Institutes of
proposal for a fee hike. According
Kanpur: President Pratibha Patil Technology (IITs), is all set for an
to the fee-hike proposal, which
recently buried a 'Time Capsule' overhaul.
the IIT-Kanpur's Director framed,
on the Indian Institute of According to the recommendation it suggested to hike fees by eight
Technology, Kanpur (IIT-K) of a panel set up under IIT times. At present, the B.Tech
campus on the occasion of its Kaharagpur's Director, Mr. students pay Rs.50,000 per year as
golden jubilee celebrations and Damodar Acharya, the Joint their fee which the committee
also unveiled a nanosatellite Entrance Examination (JEE) for proposed to increase up to Rs.4
developed by the institute. The undergraduates and Graduate lakh per annum over the period of
capsule, which is made of a special Aptitude Test in Engineering 10 years with a Rs.35,000 mark-up
metal, contains pen-drives, chips, (GATE) for postgraduates may every year.
images and several other documents soon not be the only criteria for
related to the landmark achieve- "The proposal on fee hike should
their admission. The committee
ments of the IIT-K. proposes to consider the Class XII be discussed as the government
marks as well. was planning to set up a Higher
Lauding the nanosatellite Jugnu's
Education Funding Corporation
development team, Patil said it Presently, the eligibility for a
projects the complex nature of which would address poor
student to sit for a JEE is 60% in
tasks that the students there were students," said a senior official.
class XII after which a cut-off is
equipped to handle. decided every year for admission During the meeting, Sibal also
Congratulating IIT-K students and into an IIT. Following the asked the IITs to come up
faculty, Patil said that the institute consensus of the last meeting of with their plans for the future
has come a long way in its 50 the IIT Council to give more endeavors in a specialized area in
years of its existence, and also weightage to the school leaving which they want to emerge as
called upon the institute's students examination, the committee global giants by 2020 in four
and faculty members to develop proposes to mull over the issue of weeks time.
such devices that can harness factoring in class XII examination
energy in efficient ways with result in the cut-off for admission Nanotech is used to treat
minimal negative impact on the to the IITs. cancer - IIT-B & Docs
environment. The IIT Directors in a meeting In Mumbai it could have been
"It (IIT-K) had made an impact on with the Human Resource India’s second nano success if it
technical education within the Development (HRD) Minister, passes the muster. Only this nano
country, while its students Kapil Sibal, in Manesar, also creation is revealed in the
through their innovations, have expressed their discontent over healthcare field and it was possible
played an important role in India, the existing pattern for GATE as because of partnership between
as well as around the world," the they felt that not enough students
President said. Jugnu, developed oncologists and scientists of the
with research orientation were
by a team of 50 IIT students, will Indian Institute of Technology-
being picked up through this exam.
help in collection of information Bombay.
Sibal said that each IIT should
related to floods, drought and The joint effort of the IIT -B and
other natural calamities. submit a proposal within a month
doctors from Tata Memorial
on one area of expertise in higher

XtraEdge for IIT-JEE 4 APRIL 2010


Hospital in Parel and Apollo In his 15-minute interaction with presence of Chief Minister Ashok
Hospital in Hyderabad has the the PM in New Delhi, Anand Gehlot who was on a visit to
possibility to transform treatment enlightened him of his efforts Jodhpur, signed the MoU.
of retinoblastoma—a rare cancer usher in a new awakening in Bihar The CM also inaugurated the
of the retina that mainly affects by sending 182 poor students to work to extend the building.
children under two years of age. IITs within the past seven years. Gehlot pointed out in an official
They have developed a nano- He further informed the PM that press release that the
particle that could conquer the he has decided to increase the establishment of IIT in Jodhpur
child killer. student intake to 60 from 30. Very was recommended and headed by
Shirin Thakur, Guntur-based soon he would open schools for the noted economist V.S. Vyas
teenager was suffering from talented poor children so as to after he studied the facilities and
recurrent retinoblastoma since provide them the right momentum educational standard of various
she was two years old. Last week, at the secondary school level towns in the State.
she took the third injection of a itself. He added that government
"The final decision to nod for an
special mixture into the tissues should run coaching programmes
IIT in the city was accepted at the
around her left eye which on the pattern of Super-30 for
central level," added Gehlot. He
was made of nano-particles of talented students from the
further announced that the
carboplatin which is commonly countryside and it should not be
construction of a new building in
used to treat retinoblastoma. confined to IIT only. Rajasthan would require some
While she was standing with her Audibly elated Anand told TOI time, and for the time being the
doctor in the Apollo hospital, over the phone that Pm patted classes would be held in JNV
Hyderabad said, “I have been him on his back and also University while those in the
suffering from attacks of instructed PMO to look into his extended portion of the building
retinoblastoma in my left eye since suggestions. would begin from May his year.
I was two. Even in the US, they He said that many problems asked Following special efforts made by
told me there is no hope but to in JEE are of Maths Olympiad level the state government Mr. Gehlot
remove my eye.’’ Now, she has so students from villages find it said that the union government
“fuzzy’’ vision in the nearly blind difficult to solve such problems has also nodded to use the newly
eye. “My vision gets better every even if they have sound knowledge constructed A.S.K. Hostel in the
day.’’ of the subject at +2 level. He university for IIT students.
pleaded that the exam should be "The establishment of an IIT in the
PM was asked to fine-tune
designed in such a way that state would definitely boost its
JEE by Maths Wizard conceptual or analytical problems higher education and will prove to
PM Manmohan singh was given of the +2 level should be asked in be a milestone," said Gehlot.
suggestions on the improvement JEE." According to the MoU, it
of IIT-JEE pattern by Maths
IIT Rajasthan moves to approves the functioning of IIT-
Wizard Anand Kumar. He pleaded
state varsity campus Rajasthan only on temporary basis
that poor aspirants should be
for two years in the engineering
given at least three attempts for Jaipur: The Indian Institute of
faculty building.
IIT-JEE exam, as they are usually Technology, Rajasthan (IIT-R) has
the late starters. been temporarily shifted to the State Technical Education Minister
engineering faculty building of Jai Mahendrajit Singh Malviya and JNV
Every year Anand’s Super-30
Narain Vyas University according University Vice-Chancellor
offers free accommodation to 30
to a Memorandum of Understanding Naveen Mathur signed the MoU as
poor students and provides witnesses. Among others, Jodhpur
coaching to them free of cost in (MoU) signed in Jodhpur over the
weekend. Students of IIT-R earlier MP Chandresh Kumari, Barmer-
order to help them to crack JEE. Jaisalmer MP Harish Chaudhary,
This noble initiative, has of late had to rely on IIT-Kanpur and
other institutions to attend Jodhpur Mayor Rameshwar
reported 100% success rate with Dadich, Deputy Mayor Niyaz
all its 30 students making it to classes.
Mohammed and Collector Naveen
IIT’S, without seeking any financial IIT-Rajasthan founder-director P. Mahajan were present on the
support from government or non- K. Kalra and JNV University occasion.
government sources. Registrar Nirmala Meena, in the

XtraEdge for IIT-JEE 5 APRIL 2010


Success Story
This article contains story of a person who is successful after graduation from different IIT's

Mr. Krishnamurthy Rengarajan


IIT-B Gold Medallist

Knowledge is indeed wealth. Who better exemplifies it than On studies


Krishnamurthy Rengarajan,IIT-B gold medallist (B Tech Before joining IIT, I used to study for 7 to 8 hours daily.
dual-degree course). Krishnamurthy's story is that of hard After joining IIT, I used to spend about a couple of hours. I
work, sheer grit and determination. His undying passion for did not go for anything coaching classes. I learnt through
learning and excellence has paid off. Coming from a lower Brilliant Tutorial correspondence course and my preparation
middle class background, Krishnamurthy has made his began after I finished my 10th standard.
parents proud when he passed with flying colours. Why IIT
His father, who works as a typist at Bharatiya Vidya IIT is one of the premier institutes in India. I always wanted
Bhavan is overwhelmed by his son's achievement. to get good higher education, so I opted for IIT.
Rengarajan, who hails from Tamil Nadu, came to Mumbai
My mantra for success
28 years ago and settled down in a distant Mumbai suburb
of Dombivli. Though the family went through a lot of There is no short cut to success. One has to work very
hardships initially, he made sure that his children were well hard, put in a lot of effort, should have a problem-solving
educated. mentality and a right approach to every problem. My
parents always stood by me, their support has been
"My son always wanted to join the IIT. When people asked invaluable and am overwhelmed.
him what if you don't get through the entrance
examinations, he used to say, `there is no question of me Advice to IIT aspirants
not clearing the test," says his proud father. And, of course, Work hard. You have to spend a lot of time preparing as
he did top all the five years at IIT, a result of sheer hard exams are getting more and more competitive. You must
work and brilliance, says his mother, barely able to control also have problem-solving skills.
her excitement. "I am very happy for him," says Radha Interests
Rengarajan. Krishnamurthy did his schooling at the Kidland Solving math puzzles, reading books. I used to play cricket,
School in Dombivli and pre-degree from V G Vaze College but now I don't get the time.
at Mulund. His favourite subject being mathematics it was Next move
obvious that he would pursue a degree in engineering. He
Money is the least important thing for Krishnamurthy. So no
won the Rakesh Mathur award of Rs 1 lakh (Rs 100,000)
jobs for the time being. "I have been selected for the
during his third year and other scholarships throughout the
scholarship programme at Stanford University for a PhD in
four years. Here's what Krishnamurthy had to say on his IIT
operations research. I would like to research on optimising
experience.
computer networks and operation systems. Quality research is
My IIT experience available abroad. After the PhD programme I would like to
The five years I spent at IIT were the best in my life. I will join any academia of good repute and continue my research
cherish each and every moment here. I loved everything activities. Among corporates, I admire Google. It is the one
here: the professors are the best one can ever get, the company that reflects perfection, hard work and efficiency."
facilities to study and the extra-curricular activities are Will you come back to India?
excellent. I made best of friends and thoroughly enjoyed Of course, I will. The brain drain phenomenon is dying out.
my college life. It's the time for reverse brain.
I don't think I will ever get this experience anywhere else.

XtraEdge for IIT-JEE 6 APRIL 2010


KNOW IIT-JEE
By Previous Exam Questions

The graph between NA and cos θ


PHYSICS From eq (iii) when θ = 0, NA = mg.

1. A spherical ball of mass m is kept at the highest point 2mg NB


in the space between two fixed, concentric spheres A NA
and B (see figure in solution). The smaller sphere A
has a radius R and the space between the two spheres mg 5mg
has a width d. The ball has a diameter very slightly less
than d. All surface are frictionless. The ball is given a 2mg
gentle push (towards the right see figure in solution)
The angle made by the radius vector of the ball with 1
cos θ –1 cos θ
the upward vertical is denoted by θ [IIT-2002]
2
(a) Express the total normal reaction force exerted by When θ = cos–1   ; NA = 0
the sphere on the ball as a function of angle θ. 3
(b) Let NA and NB denote the magnitudes of the The graph is a straight line as shown.
normal reaction forces on the ball exerted by the 2
sphere A and B, respectively. Sketch the variations of When θ > cos–1  
3
NA and NB as functions of cos θ in the range 0 ≤ θ ≤ π
by drawing two separate graphs in your answer book, mv 2
NB – (– mg cos θ) =
taking cos θ on the horizontal axes. d
R+
Sol. The ball is moving in a circular motion. The 2
necessary centripetal force is provided by mv 2
(mg cos θ – N). Therefore ⇒ NB + mg cos θ = …(iv)
 d
R + 
B  2
C NA Using energy conservation
A d/2
D 1  d  d 
V mv 2 = mg  R +  −  R +  cos θ
θ θ mgsinθ 2  2  2 
R mgcosθ
mg mv 2
= 2 mg [1 – cos θ] …(v)
 d
R + 
 2
From (iv) and (v) we get
NB + mg cos θ = 2 mg – 2mg cos θ
mv 2 NB – mg (2 – 3 cos θ)
mg cos θ – NA = …(i)
 d 2
R +  When cos θ = , NB = 0
 2 3
According to energy conservation When cos θ = – 1, NB = 5 mg
1  d Therefore the graph is as shown.
mv2 = mg  R +  (1 – cos θ) …(ii)
2  2 2. A cylindrical block of length 0.4 m and area of cross
From (i) and (ii) NA = mg (3 cos θ – 2) …(iii) section 0.04 m2 is placed coaxially on a a thin metal
The above equation shows that as θ increases NA disc of mass 0.4 Kg and the same cross section. The
decreases. At a particular value of θ, NA will become upper face of the cylinder is maintained at a constant
zero and the ball will lose contact with sphere A. This temperature of 400 K and the initial temperature of
condition can be found by putting NA = 0 in eq. (iii) the disc is 300 K. If the thermal conductivity of the
0 = mg (3 cos θ – 2) materical of all cylinder is 10 watt/kg. K, how long
will it take for the temperature of the disc to increases
2
∴ θ = cos–1   to 350 K? Assume, for purpose of calculation, the
3 thermal conductivity of the disc to be very high and

XtraEdge for IIT-JEE 7 APRIL 2010


the system to be thermally insulated except for the Sol. Because the forces due to parallel electric and
upper face of the cylinder. [IIT-1992] magnetic fields on a charged particle moving
Sol. Initially the temperature at the open end is 400 K and perpendicular to the fields will be at right angles to
the temperature at the metal disc-cylinder interface is each other (electric force being along the direction of
300 K. →
θ E while magnetic force perpendicular to the plane
containing v and B ) so magnetic force will not
affect the motion of charged particle in the direction
H of electric field and vice-versa. So the problem is
equivalent to superposition of two independent
Open motions as shown in the adjoining figures.
end Y
Metal disc Cylinder B E Fe
Insulation
As heat passes through the cylinder and reaches the
metal disc, the temperature of metal disc rises. Since ĵ

the conductivity (thermal) of metal disc is very high


so temperature of the whole disc will rise and along î
with that the temperature of the other end of the
k X
cylinder (metal disc-cylinder interface) also rises
simultaneously. Let at any instant of time, the Fm v0
temperature of the metal disc-cylinder interface is θ. Z
At this instant the rate of heat crossing the cylinder. So for motion of the particle under electric field
alone,
dQ KA(400 – θ)
= ...(i) qE dv y qE
dt l ay = i.e., =
where K = thermal conductivity m dt m
A = area of cross section of cylinder y y qE qE
l = length of cylinder
or ∫0
dv y = ∫0 m
dt i.e., vy =
m
t ...(1)
The same amount of heat is received by the metal While at the same instant, the charged particle under
disc. Therefore the action of magnetic field will describe a circle in
dQ dθ the x-z plane with
= mc ...(ii)
dt dt mv 0 v qB
r= i.e., ω = 0 =
m = mass of disc qB r m
c = specified heat of metal disc
v0
From (i) and (ii) X
dθ KA(400 – θ)
mc = θ v0cos θ
dt l θ
 dθ  mcl
  × = dt v0 sin θ v0
 400 − θ  KA
On integrating At t = 0, θ = 300
Z
t mcl 350 dθ
∫ 0
dt = ∫
KA 300 (400 − θ)
At t = t, θ = 350 So angular position of the particle at time t in the x-z
plane will be given by
− mcl qB
⇒t= [log (400 – θ) ] 350
300 θ = ωt = t
KA m
−0.4 × 600 × 0.4 × 2.303 400 − 350 and therefore in accordance with figure
= log10
10 × 0.04 400 − 300  qB 
vx = v0 cosθ = v0 cos ωt = v0 cos  t ...(2)
= 166.38 sec. m 
3. A particle of mass m and charge q is moving in a  qB 
region where uniform, constant electric and magnetic and vz = v0 sinθ = v0 sin ωt = v0sin  t ...(3)
fields E and B are present. E and B are parallel to m 
each other. At time t = 0 the velocity v0 of the particle So in the light of equations (1), (2) and (3), we get
is perpendicular to E. (Assume that its speed is →
v = î vx + ĵ vy + k̂ vz
always <<c, the speed of light in vaccum.) Find the
velocity v of the particle at time t. You must express  qB   qE   qB 
your answer in terms of t, q, m, the vectors v0, E and = î v0 cos  t  + ĵ  t  + v0 sin  t  k̂
m  m  m 
B and their magnitudes v0, E and B. [IIT-1998]

XtraEdge for IIT-JEE 8 APRIL 2010


→ → → A=L×d
v E B 2πR
But because here, î = 0 , ĵ = = ∴ R=ρ ...(ii)
v0 E B Ld
v0 × B The induced current I will be
and k̂ =
v0B |e| [πµ 0 na 2i 0 ω cos ωt ] × Ld
I= =
→  → R ρ × 2πR
→ v   qBt   E  qE
So, v0 = 0  v0 cos   +   t πµ 0 na 2 i 0 ω cos ωt × Ld
 v0   m  E m I=
    2ρR
v ×B  qBt  5. A beam of light has three wavelength 4144 Å, 4972
+ 0 v0sin  
v0B  m  Å and 6216 Å with a total intensity of 3.6 × 10–3
Wm–2 equally distributed amongst the three
4. A long solenoid of radius 'a' and number of turns per wavelengths. The beam falls normally on an area
unit length n is enclosed by cylindrical shell of radius 1.0 cm2 of a clean metallic surface of work function
R, thickness d(d<<R) and length L. A variable 2.3 eV. Assume that there is no loss of light by
current i = i0 sin ωt flows through the coil. If the reflection and that each energetically capable photon
resitivity of the material of cylindrical shell is ρ, find ejects one electron. Calculate the number of photo
the induced current in the shell. [IIT-2005] electrons liberated in two seconds. [IIT-1989]
Sol. The magnetic field in the solenoid is given by Sol. Work function
B = µ0ni = 2.3 eV = 2.3 × 1.6 × 10–19J = 3.68 × 10–19 J
a R Energy of a photon of wave length (4144 Å)
d hc 6.6 × 10 −34 × 3 × 108
= =
λ 4144 × 10 −10
= 4.78 × 10 × 10–16
–3

L = 4.78 × 10–19 J
Energy of a photon of wave length 4972 Å
6.6 × 10 −34 × 3 × 108
= = 3.98 × 10–19J
4972 × 10 −10
Energy of a photon of wavelength 6216 Å
⇒ B = µ0 n i0 sin ωt [Q i = i0 sin ωt given] 6.6 × 10 −34 × 3 × 108
The magnetic flux linked with the solenoid = −10
= 3.18 × 10–19 J
→ →
6216 × 10
φ = B . A = BA cos 90º This means that light of wavelength 4144 Å and
= (µ0 n i0 sin ωt) (πa2) 4972Å are capable of ejection of electrons from the
∴ The rate of change of magnetic flux through the metal surface
solenoid The energy incident on 1 cm2 area of metal per
second = 3.6 × 10–3 × 10–4 = 3.6 × 10–7 J

= π µ0 n a2 i0 ω cos ωt The energy/s of wavelength
dt 4972 Å = 1.2 × 10–7 J
The same rate of change of flux is linked with the No of photon incident of wavelength 4972 Å
cylindrical shell. By the principle of electromagetic n hc Eλ
induction, the induced emf produced in the E= ∴ n=
λ hc
cylinderical shell is
1.2 × 10 −7 × 4972 × 10 −10
⇒n= = 301.33 × 109
6.6 × 10 −34 × 3 × 108
= 3.01 × 1011
Similarly number of photon incident of wavelength
I 4144 Å.
TOP VIEW
1.2 × 10 −7 × 4144 × 10 −10
dφ n= = 2.51 × 1011
e=– = – πµ0 n a2 i0 ω cos ωt ...(i) 6.6 × 10 −34 × 3 × 108
dt
⇒ Total number of photons capable of ejection of
The resistance offered by the cylindrical shell to the
electrons per second
flow of induced current I will be
= 3.01 × 1011 + 2.51 × 1011 = 5.52 × 1011
l
R=ρ ∴ Total number of photoelectrons ejected in two
A seconds ≈ 11 × 1011.
Here, l = 2πR,

XtraEdge for IIT-JEE 9 APRIL 2010


CHEMISTRY or 2.303 log
1.067 × 109
2.67 × 108
=
E a  320 − 300 
 
8.314  320 × 300 
6. From the following data, form the reaction between Ea  20 
or 2.303 × 0.6017 =  
A and B. [IIT-1994] 8.314  320 × 300 
[A] [B] Initial rate (mol L–1s–1) 2.303 × 0.6017 × 8.314 × 320 × 300
mol L–1 mol L–1 300 K 320 K or Ea =
20
2.5 ×10–4 3.0 ×10–5 5.0 ×10–4 2.0 × 10–3 = 55.3 kJ mol–1
5.0 × 10–4 6.0 × 10–4 4.0 × 10–3 – (d) According to Arrhenius equation,
1.0 × 10–3 6.0 × 10–5 1.6 × 10–2 –
k = Ae − E a / RT
Calculate E
(a) the order of reaction with respect to A and with or 2.303 log k = 2.303 log A – a
RT
respect to B, At 300 K,
(b) the rate constant at 300 K,
(c) the energy of activation, 55.3 × 10 3
2.303 log (2.67 × 108) = 2.303 log A –
(d) the pre exponential factor. 8.314 × 300
Sol. Rate of reaction = k[A]l [B]m or 2.303 × 8.4265 = 2.303 log A – 22.17
where l and m are the order of reaction with respect 19.4062 + 22.17 41.5762
or logA = = = 18.0531
to A and B respectively. From the given data, we 2.303 2.303
obtain following expressions : A = Antilog 18.0531 = 1.13 × 1018 s–1
5.0 × 10–4 = k[2.5 × 10–4]l [3.0 × 10–5]m ...(i)
7. 1 g of a mixture containing equal number of moles of
4.0 × 10–3 = k[5.0 × 10–4] l [6.0 × 10–5]m ...(ii) carbonates of two alkali metals, required 44.4 ml of
1.6 × 10–2 = k[1.0 × 10–3]l [6.0 × 10–5]m ..(iii) 0.5 N HCl for complete reaction. The atomic weight
From eq. (ii) and eq. (iii), we get of one metal is 7, find the atomic weight of other
l
4.0 × 10 −3  5.0 × 10 −4  metal. Also calculate the amount of sulphate formed
=   on quantitative conversion of 1.0 g of the mixture in
1.6 × 10 − 2  1.0 × 10 −3 
  two sulphates. [IIT-1972]
or 0.25 = (0.5)l Sol. Let, Mass of one alkali metal carbonate M2CO3 = xg
or (0.5)2 = (0.5) l Then, mass of other alkali metal carbonate M2´CO3
or l=2 = (1 – x)g
From eq. (i) and eq. (ii), we get Step 1.
2 m Molecular mass 74
5.0 × 10 −4  2.5 × 10 −4   3.0 × 10 −5  Equivalent mass of M2CO3 = =
−3
=  −4



 6.0 × 10 −5

 2 2
1.0 × 10  5.0 × 10    = 37 (at mass of M = 7)
m
1 1 1 x
or = ×  Meq. of M2CO3 = ×1000
8 4 2 37
m x
1 1 Moles of M2CO3 =
or =  74
2 2 Step 2
or m=1 Molecular mass
(b) At T1 = 300 K, Equivalent mass of M2´CO3 =
2
Rate of reaction 5.0 × 10 −4
k1 = = 2m + 60
[A]2 [B]1 [2.5 × 10 − 4 ]2 [3.0 × 10 −5 ] =
8 2 –2 –1
2
= 2.67 × 10 L mol s ( m = atomic mass of M´)
(c) At T2 = 320 K, 2(1 − x )
Rate of reaction Meq. of M2´CO3 = × 1000
k2 = 2m + 60
[A]2 [B]1 1− x
Moles of M2´CO3 =
2.0 × 10 −3 2m + 60
=
[2.5 × 10 − 4 ]2 [3.0 × 10 −5 ] Step 3. Meq. of HCl = NHCl × VHCl = 0.5 × 44.4
= 1.067 × 109 L2 mol–2 s–1 Step 4. According to the question,
k E  T − T1  Moles of M2CO3 = Moles of M2´CO3
We know, 2.303 log 2 = a  2 
k1 R  T1T2  or
x
=
1− x
...(i)
74 2m + 60

XtraEdge for IIT-JEE 10 APRIL 2010


And Meq. of M2CO3 + Meq . of M2´CO3 112
= Meq. of HCl ∴ 112 ml O2 = = 0.005 mol
22400
x 2(1 − x )
or × 1000 + × 1000 112
37 2m + 60 ∴ At STP, no. of moles of O2 used =
22400
= 0.5 × 44.4 ...(ii) = 0.005 mol
Solving eq. (i) and (ii), we get Volume of CO2 obtained = 560 – 112 = 448 ml
m = 23
448
and x = 0.41 ∴ At STP, no. of moles of CO2 used =
∴ Mass of M2CO3 = x = 0.41 g 22400
and Mass of M2´CO3 = 1 – x = 0.59 g = 0.02 mol
Step 5. 0.01 mol organic compound yields = 0.02 mol CO2.
Molecular mass ∴ 1 mol organic compound yields
Equivalent mass of M2SO4 = = 2 mol CO2 or 2 mol C
2
∴ The molecular formula of organic compound is
110
= = 55 C2HyOz.
2 The reaction is :
WM 2SO 4 1 y
Meq. of M2SO4 = × 1000 C2HyOz + O2 → 2CO2 + H2O
55 2 2
But, Meq. of M2SO4 = Meq. of M2CO3 Equating no. of oxygen atoms,
WM 2SO 4 0.41 y
∴ × 1000 = × 1000 z+1=4+
55 37 2
or WM 2SO 4 = 0.6095 g y
or z =3+
Step 6. 2
Molecular mass Molecular mass of C2HyOz = 2 × 12 + y × 1 + z × 16
Equivalent mass of M2´SO4 =  y
2 Hence, 2 × 12 + y × 1 +  3 +  × 16 = 90
142  2
= = 71 or y=2
2
WM 2 ´SO 4 y
Meq. of M2´SO4 = × 1000 and z=3+ =4
71 2
But, Meq. of M2´SO4 = Meq. of M2´CO3 ∴ The molecular formula of organic compound is
WM 2 ´SO 4 C2H2O4.
2 × 0.59
∴ × 1000 = × 1000
71 106 9. (a) Write the intermediate steps for each of the
or WM 2 ´SO 4 = 0.7904 g following reactions.
+
∴ Total mass of sulphates = WM 2SO 4 + WM 2 ´SO 4 (i) C6H5CHOHC ≡ CH H
3O
→ C6H5CH=CHCHO
= 0.6095 + 0.7904 H+
= 1.3999 g (ii)
OH O CH3
8. 0.9 g of a solid organic compound (molecular mass
90), containing carbon, hydrogen and oxygen, was (b) Show the steps to carry out the following
heated with oxygen corresponding to a volume of transformations :
224 ml at STP. After combustion the total volume of (i) Ethylbenzene → benzene
the gases was 560 ml at STP. On treatment with (ii) Ethylbenzene → 2-phenylpropionic acid
potassium hydroxide, the volume decreased to 112 [IIT-1998]
ml. Determine the molecular formula of the Sol. (a) (i)
compound. [IIT-1972] H+
C6H5CH(OH)C ≡ CH C6H5CH – C ≡ CH
Sol. Given that,
Mass of solid organic compound = 0.9 g OH2+ –H2O
Molecular mass of organic compound = 90 ⊕ ⊕
∴ No. of moles of organic compound available C6H5CH = C = CH C6H5CH – C ≡ CH
0.9
= = 0.01 OH–
90
Volume of O2 taken = 224 ml Tautomerism
Volume of O2 used = 224 – 112 = 112 ml C6H5CH = C = CHOH C6H5CH = CHCHO
22400 ml O2 = 1 mol.

XtraEdge for IIT-JEE 11 APRIL 2010


⊕ P
(ii) H +
CH3 P
OH OH or
Cl Cl
Cl

O CH3 O CH3 Cl Cl Cl

H⊕ Thus due to repulsion between lp – bp, geometry is


(b) (i) distorted from tetrahedral to pyramidal.T
C2H5 COOH COONa
alk. KMnO4 NaOH NaOH MATHEMATICS
–H2O +CaO
11. 7 relatives of a man comprises 4 ladies and
Ethyl benzene Benzoic acid Sodium benzoate 3 gentlemen; his wife has also 7 relatives; 3 of them
are ladies and 4 gentlemen. In how many ways can
+ Na2CaO3 they invite a dinner party 3 ladies and 3 gentlemen so
that there are 3 of man’s relative and 3 of the wife’s
Benzene
relatives ? [IIT-1985]
(ii) Sol. The possible cases are :
CH2CH3 CHBrCH3 CH3CH – MgBr Case I : A man invites 3 ladies and women invites 3
Br2/hv Mg gentlemen
⇒ 4C3.4C3 = 16
Ether
Case II : A man invites (2 ladies, 1 gentleman) and
CO2 women invites (2 gentlemen, 1 lady)
⇒ (4C2.3C1).(3C1.4C2) = 324
CH3CHCOOH CH3CHCOOMgBr Case III : A man invites (1 lady, 2 gentlemen) and
Br H2O/H + women invites (2 ladies, 1 gentleman)
Mg + ⇒ (4C1.3C2).(3C2.4C1) = 144
OH Case IV : A man invites (3 gentlemen) and women
2-Phenylpropionic acid invites (3 ladies)
⇒ 3C3.3C3 = 1
10. Interpret the non-linear shape of H2S molecule and
non-planar shape of PCl3 using valence shell electron ∴ Total number of ways = 16 + 324 + 144 + 1 = 485
pair repulsion (VSEPR) theory. (Atomic numbers :
H = 1, P = 15, S = 16, Cl = 17.) [IIT-1998] 12. Let n be a positive integer and
(1 + x + x2)n = a0 + a1x + ..... + a2nx2n
1
Sol. In H2S, no. of hybrid orbitals = (6 + 2 – 0 + 0) = 4 Show that a02 – a12 + ...... + a2n2 = an. [IIT-1994]
2 Sol. (1 + x + x2)n = a0 + a1x + .... + a2nx2n ...(1)
Hence here sulphur is sp3 hybridised, so Replacing x by –1/x. we obtain
16S = 1s2, 2s22p6, 3s 2 3p 2x 3p1y 3p1z  1 1 
n
a a a a
144244 3 1 − + 2  = a0 – 1 + 22 – 33 +...+ 22nn ..(2)
sp 3 hybridisation
 x x  x x x x
S Now, a02 – a12 + a22 – a32 + ... + a2n2 = coefficient of
the term independent of x in
S [a0 + a1x + a2x2 + ... + a2nx2n]
or
H H  a1 a 2 a 2n 
H H a 0 − x + x 2 − ... + x 2 n 
 
Due to repulsion between lp - lp; the geometry of = coefficient of the term independent of x in
H2S is distorted from tetrahedral to V-shape. n
 1 1 
1 (1 + x + x2)n 1 − + 2 
In PCl3, no. of hybrid orbitals = [5 + 3 – 0 + 0] = 4  x x 
2 n
Hence, here P shows sp3-hybridisation  1 1 
But, R.H.S. = (1 + x + x2)n 1 − + 2 
15P = 1s2, 2s22p6, 3s 2 3p1x 3p1y 3p1z  x x 
144244 3
sp 3 hybridisation
(1 + x + x 2 ) n ( x 2 − x + 1) n
=
x 2n

XtraEdge for IIT-JEE 12 APRIL 2010


[( x 2 + 1) 2 − x 2 ]n Now, let x ∈ R then applying formula of
= differentiability.
x 2n
 2 x + 2h 
(1 + 2x 2 + x 4 − x 2 ) n f  − f (x)
= f (x + h) − f (x)  2 
x 2n f´(x) = lim = lim
h →0 h h →0 h
(1 + 2 x + x 4 ) n
2
= f ( 2 x ) + f ( 2h )
x 2n − f (x )
= lim 2
Thus, a0 – a1 + a2 – a32 + ... a2n2
2 2 2
h →0 h
= coefficient of the term independent of x in
1   2x   2h  
1
(1 + x2 + x4)n 2f   − 1 + 2f   − 1 − f ( x )
2  2   2  
x 2n = lim 
= coefficient of x2n in (1 + x2 + x4)n
h →0 h
= coefficient of tn in (1 + t + t2)n = an [using equation (1)]
1
{2f ( x ) – 1 + 2f (h ) − 1} − f ( x )
13. Solve for x the following equation : = lim 2
log(2x + 3) (6x2 + 23x + 21) h →0 h
= 4 – log(3x + 7) (4x2 + 12x + 9) [IIT-1987] f (h ) − 1
Sol. log(2x + 3) (6x2 + 23x + 21) = lim = –1
h →0 h
= 4 – log(3x + 7) (4x2 + 12x + 9) Therefore f´(x) = – ∀ x ∈ R
⇒ log(2x + 3)(2x + 3) . (3x + 7) = 4 – log(3x + 7)(2x + 3)2
⇒ 1 + log(2x + 3) )(3x + 7) = 4 – 2log(3x + 7) (2x + 3) ⇒ ∫ f ´(x) dx = ∫ −1 dx
Put log(2x + 3) (3x + 7) = y ⇒ f(x) = – x + k where k is a constant.
2 But f(0) = 1, therefore f(0) = – 0 + k
⇒ y+ – 3 = 0 ⇒ y2 – 3y + 2 = 0
y ⇒ 1=k
⇒ (y – 1) (y – 2) = 0 ⇒ f(x) = 1 – x ∀ x ∈ R
⇒ y=1 or y=2 ⇒ f(2) = – 1
⇒ log(2x + 3) (3x + 7) = 1
or log(2x + 3)(3x + 7) = 2 15. If (a + bx)ey/x = x, then prove that
2
⇒ 3x + 7 = 2x + 3 or (3x + 7) = (2x + 3)2 d2y  dy 
x3 = x − y [IIT-1983]
⇒ x = – 4 or 3x + 7 = 4x2 + 12x + 9 dx 2  dx 
4x2 + 9x + 2 = 0
Sol. (a + bx).ey/x = x ...(1)
4x2 + 8x + x + 2 = 0
Differentiating both sides, we get
(4x + 1) (x + 2) = 0
x = – 2, –1/4  dy 
x − y
y/x  dx 
∴ x = – 2, –4, –1/4 (a + bx).e .  2
y/x
 + be = 1
But, log exists only when, 6x2 + 23x + 21 > 0,  x 
4x2 + 12x + 9 > 0,  
2x + 3 > 0 and 3x + 7 > 0  dy 
⇒ x > –3/2 x − y
dx
∴ x = –1/4 is the only solution. ⇒ x.   + beb/x = 1, (using (1))
x2
14. Let f[(x + y)/2] = {f(x) + f(y)} / 2 for all real x and y, dy y
or – + bey/x = 1,
if f´(0) exists and equals –1 and f(0) = 1, find f(2). dx x
[ΙΙΤ−1992] Again differentiation both sides,
x+y f ( x ) + f ( y) dy  dy 
Sol. f   = ∀ x, y ∈ R (given) x −y x − y
d2y dx y/x  dx 
 2  2 – + be .   = 0 ..(2)
Putting y = 0, we get dx 2 x2  x 2

 
x f ( x ) + f (0) 1
f  = = [1 + f(x)] [Q f(0) = 1] dy
2
  2 2 x −y
d2y dx  dy y 
⇒ 2f(x/2) = f(x) + 1 from (2), –  −  =0
dx 2 x 2
 dx x 
⇒ f(x) = 2f(x/2) – 1 ∀ x, y ∈ R ...(1) 2
Since f´(0) = –1, we get d2y  dy 
⇒ x3 = x − y
f ( 0 + h ) − f ( 0) f (h ) − 1 dx 2  dx 
⇒ lim = – 1 ⇒ lim
h →0 h h → 0 h

XtraEdge for IIT-JEE 13 APRIL 2010


Physics Challenging Problems
Set #12

This section is designed to give IIT JEE aspirants a thorough grinding & exposure to variety
of possible twists and turns of problems in physics that would be very helpful in facing IIT
JEE. Each and every problem is well thought of in order to strengthen the concepts and we
hope that this section would prove a rich resource for practicing challenging problems and
enhancing the preparation level of IIT JEE aspirants.
By : Dev Sharma
So lutions g iven in same issue Director Academics, Jodhpur Branch

1. Four infinite thin current carrying sheets are placed in B


YZ plane. The 2D view of the arrangement is as shown
in fig. Direction of current has also been shown in the 2µ0I
(D)
figure. The linear current density. i.e. current per unit +µ0I
width in the four sheets are I, 2I, 3I and 4I, respectively. a X
2a 3a 4a 5a
–µ0I
Y I II III IV

2. Match the column


X Column – I Column – II
(A) a charge particle is (P) Velocity of the
moving in uniform particle may be
a a a a
electric and magnetic constant
The magnetic field as a function of x is best fields in gravity free
represented by space
(B) a charge particle is (Q) Path of the particle
B
moving in uniform may be straight line
5µ0I electric, magnetic
(A) 2µ I
0 and gravitational
µ0I fields
a 2a 3a 4a 5a X (C) a charge particle is (R) Path of the particle
moving in uniform may be circular
B magnetic and
3µ0I gravitational fields
(B) (where electric field
µ 0I is zero)
a X (D) A charge particle is (S) Path of the particle
2a 3a 4a 5a
–µ0I moving in only may be helical
uniform electric field
B
(T) None
4µ0I
(C) 3. Magnetic flux in a circular coil of resistance 10Ω
+µ0I
changes with time as shown in fig. Cross indicates a
a X
2a 3a 4a 5a direction perpendicular to paper inwards.
–µ0I
Match the following :

XtraEdge for IIT-JEE 14 APRIL 2010


φ(Magnetic flux) → ∧ ∧ → ∧ ∧
are given by unit vectors µ1 = a i + b j, µ 2 = c i + b j .
10
Then
8 10 14 16
2 6 t(s) a 4 a 3
(A) = (B) =
–10 c 3 c 4

× × × × × × b 4 b 3
(C) = (D) =
× × × × × × d 3 d 4
× × × × × ×
× × × × × ×
× × × × × × 6. Two converging lenses of the same focal length f are
× × × × × × separated by distance 2f. The axis of the second lens
Column – I Column – II is inclined at angle θ = 60º with respect to the axis of
the first lens. A parallel paraxial beam of light is
(A) At 1s, induced current is (P) Clockwise
incident from left side of the lens. Then
(B) At 5s, induced current is (Q) Anticlockwise
(C) At 9s, induced current is (R) Zero
(D) At 15s, induced current is (S) 2A 60º
(T) None 2f

(A) Final image after all possible refraction will


4. A conducting rod of length l is moved at constant
formed at optical centre of first lens
velocity v0 on two parallel, conducting, smooth, fixed
(B) Final image after all possible refraction will
rails, which are placed in a uniform constant magnetic
formed at optical centre of second lens
field B perpendicular to the plane of the rails as
(C) Final image after all possible refraction will
shown in figure. A resistance R is connected between
formed at distance f from second lens
the two ends of the rail. Then which of the following
(D) Final image after all possible refraction will
is/are correct ?
formed at distance f from first lens

B 7. If Cv for an ideal gas is given by Cv = (3 + 2T)R,


R ⊗ v0
⊗ where T is absolute temperature of gas, then the
equation of adiabatic process for this gas is
(A) VT2 = constant
(A) The thermal power dissipated in the resistor is
equal to the rate of work done by an external (B) VT3e-2T = C
person pulling the rod (C) VT2e2T = constant
(B) If applied external force is doubled, then a part (D) VT3e2T = constant
of the external power increases the velocity of
the rod 8. The pressure of one mole of ideal gas varies
(C) Lenz’s law is not satisfied if the rod is according to the law P = P0 – αV2 where P0 & α are
accelerated by an external force positive constant constants. The highest temperature
(D) If resistance R is doubled, then power required to that gas may attain -
maintain the constant velocity v0 becomes half 1/ 2 1/ 2
2P0  P0  3P0  P0 
(A)   (B)  
5. The x-z plane separates two media A & B of
3R  3α  2R  3α 
refractive indices µ1 = 1.5 & µ2 = 2. A ray of light 1/ 2 1/ 2
P0  P0  P0  P0 
travels from A to B. Its directions in the two media (C)
R  3α  (D)  
R  α 
 

XtraEdge for IIT-JEE 15 APRIL 2010


8 Solution
Set # 11

Physics Challenging Problems


Q u e s tio ns we r e Pub l is he d in Ma r ch I ss ue

1.[C] The given circuit as an R-L-C series circuit when 3[C] At frequency f1 current
frequency of the source varies the impedance of the I 1 V 1 200 20
± 0 = . = . = = 10 2 Amp.
R-L-C series circuit varies and correspondingly the 2 2 R 2 10 2
current in the circuit get varied Watt less current
Impedance variation and current variation are shown I µ = I sin φ
in figure.
1
I I µ = 10 2 . = 10 Amp.
2
I0
R 1
As φ = 45º because cos φ = R / Z = =
I0 R 2 2
R=Z I=
minimum 2
V V 200
4[A] At frequency current I = = = = 20 Amp.
Zm R 10
f1 f=fr f2 f f1 f=fr f2 f Potential difference across capacitor
∆f = f2-f1 ∆f = f2-f1 VC = IC.XC = 1.XC = 20.XC
Charge on capacitor QC = C.VC = C. 20XC
At frequency f1 XC > XL Power factor – leading
1
nature of circuit is capacitance = C. (2v)
ωC
At frequency f2 XL > XC Power factor – leading
20 20 20 1
nature of circuit is inducting = = = =
ω 2πf 2π(50) 5π
At frequency f1 and frequency f2 impedance Z= 2 .R
= (5π) −1 Coulomb
Because of the fact –
V V = (5π) −1 cb
As I0 = = ........(i)
Zhm R 5.[C] Longest wavelength
I0 V v + v s 350 × 0.8 × 5
±= = λ max = = = 0.59m
2 Z f 600
V/2 V v
⇒ = 6.[B] f max = f
2 Z v − vs
350
V V = = 607 Hz
⇒ = and R 2 350 − 0.8 × 5
R 2 Z
f 1 < f < f2 7.[A] 345.5/346.0 × 600 = 599 Hz
2.[A] ∆f = f 2 − f1 = B and width of R-L-C series circuit
dy 1 dy d 2 y 1 d2y
1 8.[A] =− & 2 =+ 2 2
= .R / L dx v dt dx v dt

XtraEdge for IIT-JEE 16 APRIL 2010


XtraEdge for IIT-JEE 17 APRIL 2010
8 Solution
Set # 12

Physics Challenging Problems


Questio ns Pub lished in this Issue

1.[C] Magnetic field due to infinite current carrying (C) This situation is similar to part (i)
µ J (D) In a uniform electric field, path can be only
sheet is given by B = 0 , where J is linear current
2
density. 3. A→Q B→R C→P D→Q
II III (A) At t = 1s, flux is increasing in the inward
direction, hence induced e.m.f. will be in
µ0J
µ0J µ0J anticlockwise direction.
2
2 2 (B) At t = 5s, there is no change in flux, so induced
µ0J
e.m.f. is zero
2
(C) At t = 9s, flux is increasing in upward direction
(a) (b) hence induced e.m.f. will be in clockwise direction.
Fig. (a) and (b) represent the direction of magnetic (D) At t = 15s, flux is decreasing in upward
field due to current carrying sheets. For x < a, direction, so induced e.m.f. will be in
µ J µ J (2J ) µ 0 (3J) µ 0 (4J) anticlockwise direction.
B resul tant = 0 − 0 − +
2 2 2 2 4.[A, B, D]
For a < x < 2a, Rate of work done by external agent is
µ J µ (2J) µ 0 (3J) µ 0 (4J)
B resul tant = 0 − 0 − + = −µ 0 J de/dt = BIL.dx/dt = BILv and thermal power
2 2 2 2
dissipated in resistor = eI = (BvL) I clearly both are
For 2a < x < 3a,
µ J µ (2J) µ 0 (3J ) µ 0 (4J ) equal, hence (A).
B resul tant = 0 + 0 − − =0 If applied external force is doubled, the rod will
2 2 2 2
So, the required curve is experience a net force and hence acceleration. As a
B result velocity increase, hence (B).
Since, I = e/R
On doubling R, current and hence required power
become half.
O X Since, P = BILv
a 2a 3a 4a 5a
Hence (D)

5.[A]
2. A → P,Q,S B → P,Q,R,S ^j
C → P,Q,R,S D→Q
(A) Velocity of the particle may be constant, if forces
of electric and magnetic fields balance each xz
other. Then, path of particle will be straight line.
Also, path of particle may be helical if magnetic n1sini = n2sinr
and electric fields are in same direction. But path → ∧ → ∧
of particle cannot be circular. Path can be 1.5(µ 1 × j) = 2(µ 2 × j)
∧ ∧ ∧ ∧ ∧ ∧
circular if only magnetic field is present, or if 1.5(a i + b j) × j = 2[(c i + d j) × j]
some other forces is present which can cancel the ∧ ∧
effect of electric field. 1.5a k = 2c k
(B) Here, all the possibilities are possible depending a 20 4
= =
upon the combinations of the three fields. c 1.5 3

XtraEdge for IIT-JEE 18 APRIL 2010


6.[A]
I2 f

f sin60º f cos60º +
f
• Saturn’s rings are made up of particles of ice, dust
I1
and rock. Some particles are as small as grains of
x sand while others are much larger than
u = -f cos60º skyscrapers.
f = +f
1 1 1 • Jupiter is larger than 1,000 Earths.
= −
f v − f cos 60º • The Great Red Spot on Jupiter is a hurricane-like
1 1 2
= + storm system that was first detected in the early
f v f 1600’s.
1 2 1
− =
f f v • Comet Hale-Bopp is putting out approximately
v = -f 250 tons of gas and dust per second. This is
f about 50 times more than most comets produce.
= cos 60º
x
f • The Sun looks 1600 times fainter from Pluto than
=x it does from the Earth.
cos 60º
x = 2f • There is a supermassive black hole right in the
∴ final image will formed at optical centre of first middle of the Milky Way galaxy that is 4 million
lens. times the mass of the Sun.
7.[C] Cv = (3 + 2T)R • Halley’s Comet appears about every 76 years.
dQ = dU + PdV
• The orbits of most asteroids lie partially between
adiabatic process dQ = 0
the orbits of Mars and Jupiter.
0 = Rn (3 + 2T)dT + PdV
nRT • Asteroids and comets are believed to be ancient
0 = Rn (3 + 2T )dT + dV remnants of the formation of our Solar System
V
dV  3 + 2T  (More than 4 billion years ago!).
∫−
V ∫
= 
 T 
dT
• Comets are bodies of ice, rock and organic
– log V = 3 logT + 2T + C compounds that can be several miles in diameter.
– logV – logT3 = 2T + C
• The most dangerous asteroids, those capable of
log VT3 = 2T + C
causing major regional or global disasters, usually
VT3 = e2T
impact the Earth only once every 100,000 years
VT3e-2T = C
on average.
8.[A] P = P0 − αV 2 • Some large asteroids even have their own moon.
PV = RT
RT • Near-Earth asteriods have orbits that cross the
= P0 − αV 2 Earth’s orbit. These could potentially impact the
V
Earth.
P V αV 3
T= 0 −
R R • There are over 20 million observable meteors
dT per day.
=0
dV
• Only one or two meteorites per day reach the
P0 3αV 2 surface of Earth.
− =0
R R
• The largest found meteorite was found in Hoba,
P0
V= Now put V in T. Namibia. It weighed 60 tons.

XtraEdge for IIT-JEE 19 APRIL 2010


Students' Forum
Expert’s Solution for Question asked by IIT-JEE Aspirants
PHYSICSS

1. A beam of length L, breadth b and thickness d when 2. A small glass ball is released from rest from the top
loaded by a weight Mg in the middle, a depression e of a smooth incline plane of constant base b. find the
is produced in it. By measuring this depression e, the angle of inclination of the plane for minimum time of
value of Young's modulus of the material of beam motion of the glass ball.
can be calculated by using the expression A
M g L3
Y=
4 b d3 e
Following are the values of different physical
quantities obtained in one set of observations on this θ
C B
experiment : b
M = 1000 gms, L = 200 cm, Sol. Let the angle of inclination be θ. If, the glass ball
b = 2.54 cm, d = 0.620 cm, e = 0.1764 cm. reaches the bottom B of the inclined plane after a
If M is measured by spring balance, L by metre scale, time, say t, the equation of motion along the plane is
b by vernier calipers, d by screw gauge and e by given as
spherometer, then what will be the maximum 1
possible percentage errors in Y ? AB = (VA)t + (g sin θ)t2
2
M g L3 A L = b sec θ
Sol. Given that Y =
4 b d3 e
Taking log on both sides of above equation, we get
log Y = log M + log g + 3 log L – log 4 g sin θ
– log b – 3 log d – log e θ
Differentiating above equations, we have : C g B
∆Y ∆M ∆L ∆b ∆d ∆e b
= +3 – –3 – Since the glass ball is released from rest VA = 0,
Y M L b d e
1
In order to calculate maximum possible error, we hence AB = (g sin θ)t2 ...(1)
shall convert negative sign into positive sign. 2
Putting AB = (BC) sec θ = b sec θ, in equation (1),
∆Y ∆M ∆L ∆b ∆d ∆e
∴ = +3 + +3 + we obtain
Y M L b d e
2b 4b
Now, least counts of the different measuring t= =
instruments used in the experiment are as under : g sin θ cos θ g sin 2θ
Least count of spring balance = 5 gm i.e. ∆M = 5gm For t to be minimum, sin 2θ is maximum
Least count of metre scale = 0.1 cm i.e. ∆L = 0.1 cm ∴ sin 2θ|max = 1
Least count of vernier callipers = 0.01 cm π π
i.e. ∆b = 0.001 cm or, 2θ = or, θ=
2 4
Least count of screw gauge = 0.001 cm
i.e. ∆d = 0.001 cm 3. Two particles, both of mass m, attract each other with
Least count of spherometer = 0.005 cm r r α
i.e ∆e = 0.005 cm the force F( r ) = – 2 r̂
∆Y 5 3 × 0.1 0.01 3 × 0.001 0.005 r
∴ = + + + + where α is a positive constant. At a certain moment
Y 1000 200 2.54 0.62 0.1764 (t = 0), the distance between the particles is R, and
= 0.005 + 0.0015 + 0.00393 + 0.00484 + 0.02834 their velocities are
= 0.0436 or 4.36%
Hence the maximum possible percentage error is v1 = v 0 x̂

4.36%. v 2 = −2 v 0 x̂

XtraEdge for IIT-JEE 20 APRIL 2010


Assuming the two-particle system reaches a The kinetic energy in the laboratory frame is,
minimum of kinetic energy at a certain moment and therefore :
at a certain finite distance between the particles (in 5 α 4α − 9mv 02 R
the laboratory frame), find the distance between the Kmin = mv02 – + α
2 R 4αR
particles at that moment and the value of that
minimal kinetic energy. 5 9 1
=  −  mv02 = mv02 ...(9)
Sol. For the sake of convenience, we will first solve the 2 4 4
problem in the frame of the centre of mass. Then, we Another way of finding the minimal kinetic energy is
will transform the results into the laboratory frame. by using the following formula :
We will determine the velocity of the center of mass 1 2
by : K = K´ + Mv cm ...(10)
mv 0 − 2mv 0 2
r 1
v cm = x̂ = – v 0 x̂ ...(1) where K´ is the kinetic energy in the centre of mass
2m 2 frame, K is the energy in the laboratory frame, and M
This velocity remains constant since there are no is the total mass of the system. In our case,
external forces acting on the whole system. In the 1 1
2
velocity of the centre of mass, the velocities of the K = 0 + (2m)Mv cm = Mv 02 ...(11)
particles are : 2 4
Note : Generally, in transforming from system S to
 r r r 3 r
 u1 = v1 − v cm = 2 v 0 x̂ system S´ with relative velocity V , the kinetic
r ...(2) energy is transformed as :
r r 3
u 2 = v 2 − v cm = − v 0 x̂ r 2
 2 p2 Mr p 
K´ = K – + V −  ..(12)
By definition of a centre of mass frame, the total 2M 2  M
momentum of the particles is zero. The kinetic r
where M is the total mass and p is the total
energy in this system at t = 0 is :
momentum in S. K´ is minimal in the center of mass
1 1 9 r r r
K´ = mu12 + mu22 = mv02 ...(3) p
2 2 4 frame if we choose V = v cm = . We obtain :
M
Therefore, the total energy in the center of mass
frame at t = 0 is p2
K´ = K = ...(13)
9 α 2M
E´ = K´ + U´ = mv02 – ...(4)
4 R
where we define R ≡ r(t = 0). Note that since the 4. A hot body is being cooled in air according to
r Newton's law of cooling, the rate of fall of
force is conservative, we have F = – ∇ u. The scalar
temperature being K times the difference of its
function is u = – α/r. temperature with respect to that of surroundings.
The advantage of using the center of mass frame is Calculate the time after which the body will lose half
evident when one inspects the moment of arrival at a the maximum heat it can lose. The time is to be
minimal distance, t0. At that moment, in this system, counted from the instant t = 0.
the two particles stop and reverse their directions. Sol. According to Newton's law of cooling, we have
The kinetic energy, therefore, vanishes at t0 in the

center of mass frame, or, = – K(θ – θ0)
dt
K´(t0) = K 'min = 0 ...(5)
where θ0 is the temperature of the surrounding and θ
α is the temperature of the body at time t. Suppose
Hence, E´(t0) = – = E´ ...(6)
R min θ = θ1 at time t = 0.
Plugging in the value of E´, we find : θ dθ t θ − θ0

Rmax =
4αR
...(7)
Then, ∫ θ1 θ − θ 0 ∫
= –K dt or, log
0 θ1 − θ 0
= – Kt

4α − 9mv 02 R or, θ – θ0 = (θ1 – θ0)e–Kt ...(1)


We now transform the centre of mass frame to the The body continues to lose heat till its temperature
laboratory frame. Since Rmax is the relative distance becomes equal to that of the surroundings. The loss
between the two particles, it is unchanged by the of heat in this entire period is dQm = ms(θ1 – θ0).
transformation. Recall that distance is an invariant This is the maximum heat the body can lose. If the
quantity of the transformations of displacement and / body loses half this heat, the decreases in its
or rotation. Therefore, temperature will be
α 1 1 α dQ m θ − θ0
Kmin – = E(0) = E = mv02 + m(2v0)2 – = 1
R max 2 2 R 2ms 2
....(8)

XtraEdge for IIT-JEE 21 APRIL 2010


If the body loses this heat in time t1, the temperature
at t1 will be
θ − θ0 θ + θ0
Interesting Science Facts
θ1 – 1 = 1
2 2 • The Universe contains over 100 billion galaxies.
Putting these values of time and temperature in (1) : • Wounds infested with maggots heal quickly and
θ1 + θ 0 without spread of gangrene or other infection.
– θ0 = (θ1 – θ0) e − Kt1
2 • More germs are transferred shaking hands than
1 log 2 kissing.
or, e − Kt1 = or t1 =
2 K • The longest glacier in Antarctica, the Almbert
glacier, is 250 miles long and 40 miles wide.
5. In a certain region surrounding the origin of the • The fastest speed a falling raindrop can hit you is
→ → →
–4
coordinates, B = 5 × 10 k T and E = k̂ V/m. A 18mph.
proton enters the fields at the origin with an initial • A salmon-rich, low cholesterol diet means that
→ Inuits rarely suffer from heart disease.
velocity v 0 = 2.5 × 105 î m/s. Describe the proton's
• Inbreeding causes 3 out of every 10 Dalmation
motion and give its position after three complete dogs to suffer from hearing disability.
revolutions.
→ • The world’s smallest winged insect, the Tanzanian
Sol. The z-component of the force F is a constant parasitic wasp, is smaller than the eye of a
electrical force. It produces a constant acceleration housefly.
along z-axis given as • If the Sun were the size of a beach ball then
1 1  eE  Jupiter would be the size of a golf ball and the
Z = at2 =   t2
2 2 m Earth would be as small as a pea.
• It would take over an hour for a heavy object to
E z B sink 6.7 miles down to the deepest part of the
ocean.
• There are more living organisms on the skin of
each human than there are humans on the
surface of the earth.
x • The grey whale migrates 12,500 miles from the
Artic to Mexico and back every year.
y • Quasars emit more energy than 100 giant galaxies.
The other component of the force F is a magnetic • Quasars are the most distant objects in the
force which provides the necessary centripetal force Universe.
to keep the proton in a circular path of radius r(say). • The Saturn V rocket which carried man to the
The period of revolution of the proton Moon develops power equivalent to fifty 747
2πr mv 02 jumbo jets.
T= As Fcp = = ev0B
v0 r • Koalas sleep an average of 22 hours a day, two
eBr 2πm hours more than the sloth.
∴ v0 = Hence, T = • Light would take .13 seconds to travel around the
m eB
Since the particle (proton) moves in circular path Earth.
having a period of revolution T in x, y plane and • Neutron stars are so dense that a teaspoonful
moves along z-axis with a constant acceleration would weigh more than all the people on Earth.
a = eE/m, the path of the proton is a helix. • One in every 2000 babies is born with a tooth.
After three revolutions, putting t = 3T, we obtain
• Every hour the Universe expands by a billion
1  eE  9 eET 2 miles in all directions.
z =   (3T)2 =
2 m  2 m • Somewhere in the flicker of a badly tuned TV set
Putting T = 2πm/eB, we get is the background radiation from the Big Bang.
18π 2 Em 18 × (22 / 7) 2 × 5 × 1.66 × 10 −27 • The temperature in Antarctica plummets as low
z= = as -35 degrees Celsius.
eB 2 1.610 − 6 × (5 × 10 − 4 ) 2
• Space debris travels through space at over 18,000
= 37m
mph.

XtraEdge for IIT-JEE 22 APRIL 2010


P HYSICS F UNDAMENTAL F OR IIT-J EE

Calorimetry, K.T.G., Heat transfer


KEY CONCEPTS & PROBLEM SOLVING STRATEGY

Calorimetry : where C´ is a constant that depends on the nature and


The specific heat capacity of a material is the amount extent of the surface exposed. Simplifying
of heat required to raise the temperature of 1 kg of it dθ C´
by 1 K. This leads to the relation = –C(θ – θ0) where C = = constant
dt ms
Q = ms θ
Kinetic theory of gases :
where Q = heat supplied, m = mass, θ = rise in
temperature. 1
The pressure of an ideal gas is given by p = µnC2
The relative specific heat capacity of a material is the 3
ratio of its specific heat capacity to the specific heat where µ = mass of each molecule, n = number of
capacity of water (4200 J kg–1K–1).
molecules per unit volume and C is the root square
Heat capacity or thermal capacity of a body is the speed of molecules.
amount of heat required to raise its temperature by 1
K. [Unit : J K–1] 1 2 1
p= ρC or pV = mC2
Thus heat capacity = Q/θ = ms 3 3
dθ 1 dQ where ρ is the density of the gas and m = mass of the
Also = × gas.
dt ms dt
i.e., the rate of heating (or cooling) of a body depends Root Mean Square Speed of Molecules :
inversely on its heat capacity. This is defined as
The water equivalent of a body is that mass of water
which has the same heat capacity as the body itself. C12 + C 22 + C 32 + ... + C 2N
C=
[Unit : g or kg] This is given by N
m×s where N = total number of molecules. It can be
W=
sw obtained through these relations
where m = mass of body, s = specific heat capacity of 3p 3RT
the body, sw = specific heat capacity of water. C= =
ρ M
Principle of Calorimetry : The heat lost by one
system = the heat gained by another system. Or, the Total Energy of an ideal gas (E) :
net heat lost or gainsed by an isolated system is zero. This is equal to the sum of the kinetic energies of all
It system with masses m1, m2, ...., specific heat the molecules. It is assumed that the molecules do not
capacities s1, s2, ...., and initial temperatures θ1, θ2, .... have any potential energy. This follows from the
are mixed and attain an equilibrium temperature θ assumption that these molecules do not exert any
then force on each other.
Σmsθ Σ sθ 1 3 m 3
θ= , for equal masses θ = E= mC2 = RT = pV
Σms´ Σs 2 2 M 2
Newton's law of cooling : 1 2
Thus, the energy per unit mass of gas = C
The rate of loss of heat from a body in an 2
environment of constant temperature is proportional
to the difference between its temperature and that of 3
The energy per unit volume = p
the surroundings. 2
If θ = temperature of the surroundings then 3 3
The energy per mole = pV = RT
dθ 2 2
– ms = C´(θ – θ0)
dt

XtraEdge for IIT-JEE 23 APRIL 2010


Perfect gas equation : velocities between c and c + dc per unit volume is
From the kinetic theory of gases the equation of an 2
dn = 4πna3 e − bc c2 dc where
ideal gas is
pV = RT for a mole m m
b= and a =
m 2kT 2πkT
and pV = RT for any mass m
M and the number of molecules with the velocity c per
Avogadro number (N) and Boltzmann constant (k) : unit volume is
The number of entities in a mole of a substance is 2

called the Avogadro number. Its value is nc = 4πna3 e − bc c2


6.023 × 1023 mol–1. The plot of nc and c is shown in the figure. The
The value of the universal gas constant per molecular velocity possessed by the maximum number of
is called Boltzmann constant (k). Its value is molecules is called the most probable velocity
1.38 × 10–23 J K–1.
Degrees of Freedom : Principle of equipartition of
energy :
The number of ways in which energy may be stored
by a system is called its degrees of freedom.
Principle of Equipartition of Energy : This α c Crms
principle states that the total energy of a gas in
α= 2kT / m
thermal equilibrium is divided equally among its
degrees of freedom and that the energy per degree of The mean velocity
freedom is kT/2 where T is the temperature of the
c = 8kT / mπ and vrms = 3kT / mπ
gas. For a monoatomic atom the number of degrees
of freedom is 3, for a diatomic atom it is 5, for a Conduction :
polyatomic atom it is 6. The transfer of heat through solids occurs mainly by
Hence the energy of a mole of a monoatomic gas is conduction, in which each particle passes on thermal
energy to the neighboring particle but does not move
 1  3
µ = N  3 × kT  = RT from its position. Very little conduction occurs in
 2  2 liquids and gases.
Which is the same as that given by the kinetic theory. θ1 θ2
For a mole of diatomic gas µ Q
 1  5 Q
= N  5 × kT  = RT
 2  2
For a mole of polyatomic gas µ d A
 1  Consider a slab of area A and thickness d, whose
= N  6 × kT  = 3RT opposite faces are at temperature θ1 and θ2 (θ1 > θ2).
 2 
Let Q heat be conducted through the slab in time t.
When the irrational degrees of freedom are also taken
into account, the number of degrees of freedom  θ − θ2 
Then Q = λA  1 t
= 6n – 6 for non-linear molecules  d 
= 6n – 5 for linear molecules where λ = thermal conductivity of the material.
where n = number of atoms in a molecule. This has a fixed value for a particular material, being
Kinetic Temperature : large for good conductors (e.g., Cu, Ag) and low for
The kinetic temperature of a moving particle is the insulators (e.g., glass, wood).
temperature of an ideal gas in thermal equilibrium Heat Current : The quantity Q/t gives the heat flow
whose rms velocity equals the velocity of the given per unit time, and is called the heat current.
particle. In the steady state, the heat current must be the same
Maxwellian distribution of velocities : across every cross-section. This is a very useful
In a perfect gas all the molecules do not have the principle, and can be applied also to layers or slabs in
same velocity, rather velocities are distributed among contact.
them. Maxwell enunciated a law of distribution of Q dθ dθ θ − θ2
= – λA where the quantity = 1 is
velocities among the molecules of a perfect gas. t dx dx d
According to this law, the number of molecules with called the temperature gradient.

XtraEdge for IIT-JEE 24 APRIL 2010


Unit of λ : Different units are used, Stefan-Boltzmann Law : If a black body at an
e.g., cal cm s ºC–1, cal m–1 s–1 ºC–1, jm´1 s–1 ºC–1. absolute temperature T be surrounded by another
Convection : black body at an absolute temperature T0, the rate of
loss of radiant energy per unit area is
It is a process by which heat is conveyed by the
actual movement of particles. Particles closest to the E = σ(T4 – T04)
source receive heat by conduction through the wall of where σ is a constant called Stefan constant and its
the vessel. They rise up-wards and are replaced by value is 5.6697 × 10–8 W m–2 K–4
colder particles from the sides. Thus, a circulation of
particles is set up – hot particles constitute the The total energy radiated by a black body at an
upward current and cold particles, the side and absolute temperature T is given by
downward current. E = σT4 × surface area × time
The transfer of heat by convection occurs only in Note : Remember that rate of generation of heat by
fluids, and is the main mode of heat transfer in them.
Most fluids are very poor conductors. V2
electricity is given by H = I2 R or or VI Js–1 or W.
Radiation : R
Thermal Radiation : Thermal radiations are
electromagnetic waves of long wavelengths.
Solved Examples
Black Body : Bodies which absorb the whole of the
incident radiation and emit radiations of all
wavelengths are called black bodies. 1. An earthenware vessel loses 1 g of water per second
It is difficult to realize a perfect black body in due to evaporation. The water equivalent of the
practice. However, a cavity whose interior walls are vessel is 0.5 kg and the vessel contains 9.5 kg of
dull black does behave like a black body. water. Find the time required for the water in the
Absorption : Every surface absorbs a part or all of vessel to cool to 28ºC from 30ºC. Neglect radiation
the radiation falling on it. The degree of absorption losses. Latent heat of vaporization of water in this
depends on the nature and colour of the surface. Dull, range of temperature is 540 cal g–1.
black surfaces are the best absorbers. Polished, white
Sol. Here water at the surface is evaporated at the cost of
surfaces absorb the least. The coefficient of
absorption for a surface is the water in the vessel losing heat.

radiation absorbed Heat lost by the water in the vessel


aλ = = (9.5 + 0.5) × 1000 × (30 – 20) = 105 cal
radiation incident
Let t be the required time in seconds.
The suffix λ denotes the wavelength of the radiation
being considered, Clearly, aλ = 1 for a black body, for Heat gained by the water at the surface
all values of λ. = (t × 10–3) × 540 × 103
Emission : Each surface emits radiation (radiates) (Q L = 540 cal g–1 = 540 × 103 cal kg–1)
continuously. The emissive power (eλ) is defined as
the radiation emitted normally per second per unit ∴ 105 = 540t or t = 185 s = 3 min 5s
solid angle per unit area, in the wave-length range λ
and λ + dλ. Clearly, the emissive power of a black 2. 15 gm of nitrogen is enclosed in a vessel at
body (denoted by Eλ) is the maximum. temperature T = 300 K. Find the amount of heat
required to double the root mean square velocity of
Kirchhoff's Law : According to this law, for the
same conditions of temperature and wavelength, the these molecules.
ratio eλ /aλ is the same for all surfaces and is equal to Sol. The kinetic energy of each molecule with mass m is
Eλ. This simply means that good absorbers are good given by
emitters. Hence, a black body is the best emitter, and 1 3
a polished white body, the poorest emitter. m v 2rms = kT ...(1)
2 2
Prevost's Theory of Exchanges : All bodies emit
radiations irrespective of their temperatures. They If we want to increase the r.m.s. speed to η times,
emit radiations to their environments and receive then the temperature has to be raised to T´. Then,
radiations from their environments simultaneously. In 1 3 1 3
the equilibrium state the exchange between a body mv 2rms = kT´ or mη2 v 2rms = kT´ ...(2)
and the environment of energy continues in equal 2 2 2 2
amounts. From eqs. (1) and (2), T´ = η2T ...(3)

XtraEdge for IIT-JEE 25 APRIL 2010


The internal energy of n molecules at temperature T Sol. The quantity of heat Q passing across the stone is
is given by given by
5 KA(T1 − T2 ) t
U = nRT Q=
2 d
5 Here A = 3600 sq. cm = 0.36 m2
Similarly, U´ = nRT´
2 d = 10 cm = 0.10 m, (T1 – T2) = 100 – 0 = 100ºC and
5 t = 1 hour = 3600 sec.
∴ Change in internal energy ∆U = nR[T´ – T] K × 0.36 × 100 × 3600
2 ∴ Q= kilo-calories ...(1)
5 0.10
or ∆U = nRT[η2 – 1] Now heat gained by the ice in one hour
2
= mass of the ice × latent heat of ice
5 m 2
=   RT[η – 1] = 4.8 × 80 kilo calories ...(2)
2 M
From eqs. (1) and (2)
5  15  4 K × 0.36 × 100 × 3600
=   (8.31) (300) [4 – 1] = 10 J 4.8 × 80 =
2  28  0.10
4.8 × 80 × 0.10
or K =
3. 10 gm of oxygen at a pressure 3 × 105 N/m2 and 0.36 × 100 × 3600
temperature 10ºC is heated at constant pressure and
= 3 × 10–4 kilo cal m–1(ºC)–1s–1
after heating it occupies a volume of 10 litres (a) find
the amount of heat received by the gas and (b) the
energy of thermal motion of gas molecules before 5. A flat bottomed metal tank of water is dragged along
heating. a horizontal floor at the rate of 20m/sec. The tank is
Sol. (a) The states of the gas before and after heating are of mass 20 kg and contains 1000 kg of water and all
the heat produced in the dragging is conducted to the
M M
PV1 = RT1 and PV2 = RT2 water through the bottom plate of the tank. If the
µ µ bottom plate has an effective area of conduction 1 m2
Solving these equations for T2, we have and the thickness 5 cm and the temperature of water
µV2 P 32 × (10 × 10 −3 )(3 × 10 5 ) in the tank remains constant at 50ºC, calculate the
T2 = = = 1156 K temperature of the bottom surface of the tank, given
MR (10 × 10 −3 )(8.31× 10 3 ) the coefficient of friction between the tank and the
Now T2 – T1 = 1156 – 283 = 873 K floor is 0.343 and K for the material of the tank is 25
The amount of heat received by the gas is given by cal m–1 s–1 K–1.
M Sol. Frictional force = µ m g
∆Q = CP(T2 – T1) = 0.343 × (1000 + 20) × 9.81 = 3432 N
µ
The rate of dragging, i.e., the distance travelled in
(10 × 10 −3 )29.08 × 10 3 × 873 one second = 20 m.
=
32 ∴ Work done per second
= 7.9 × 103 J = (3432 × 20) Nm/sec.
(b) The energy of the gas before heating This work done appears as heat at the bottom plate of
M i the tank. Hence
E1 = × × RT1
µ 2 3432 × 20
H= cal/sec
where i = number of degrees of freedom 4.18
= 5 (for oxygen) KA(T1 − T2 )
But H= (Q t = 1 sec)
(10 × 10 −3 )5 × (8.31× 10 −3 )(283) d
=
2 × 32 3432 × 20 25 × 1× (T1 − T2 )
Now =
= 1.8 × 10 J3 4.18 0.05
3432 × 20 × 0.05
4. A slab of stone of area 3600 sq cm and thickness 10 ∴ T1 – T2 = = 32.84
4.18 × 25 × 1
cm is exposed on the lower surface of steam 100ºC.
A block of ice at 0ºC rests on upper surface of the Temp. of bottom surface T1 = 50 + 32.84
slab. In one hour 4800 gm of ice is melted. Calculate = 82.84ºC
the thermal conductivity of the stone.

XtraEdge for IIT-JEE 26 APRIL 2010


P HYSICS F UNDAMENTAL F OR IIT-J EE

Atomic Structure, X-Ray & Radio Activity


KEY CONCEPTS & PROBLEM SOLVING STRATEGY

Atomic Structure : The maximum number of electrons that can be


According to Neil Bohr's hypothesis is the angular accommodated in an orbit is 2n2.
momentum of an electron is quantised. X-rays :
 h  h When fast moving electron strikes a hard metal,
mvr = n   or L = n
 2π  2π X-rays are produced. When the number of electrons
2πr = nλ striking the target metal increases, the intensity of X-
rays increases. When the accelerating voltage/kinetic
h  c  z –1
vn = Zn =  × ms energy of electron increases λmin decreases. X-rays
2πmr  137  n
have the following properties :
 h2  n2 n2 1 (a) Radiations of short wavelength (0.01 Å – 10Å);
rn =  2 
 Z = 0.529 Å where k =
 4π mke
2
 Z 4 πε 0
high pentrating power; having a speed of 3 × 108 m/s
in vacuum.
 ke 2  15
fn =   × 1 = 6.58 × 10 Hz Continuous spectrum
 hr  n n
 
Intensity
(Varies & depends on
accelerating voltage) Characteristic spectrum
1 ke 2 Z − ke 2 ke 2 Kα
(fixed for a target material)
K.E. = ; P.E. = × Z; T.E. = – ×Z Kβ
2 r r 2r Lγ Lβ

− 13.6 Z 2
T.E. = ev/atom where –13.6
n2
= Ionisation energy λmin λ
+ P.E. hc hc 12400
⇒ +T.E. = = – K.E. (b) λmin = = = Å
2 eV K.E V
Note : If dielectric medium is present then εr has to
1  1 
be taken into consideration. (c) = R(Z – b)2 1 − 2 
λ  n 
v 1 me 4 z 2  1 1 
= =v= 2 3  2
− 2 b = 1 for k-line transfer of electron
c λ 8ε 0 h c  n1 n 2 
(d) Moseley law ν = a(z – b)
1 2 1  p mv
= RZ  2 − 2  = = R = R0A 1/3
where R0 = 1.2 × 10–15 m
 n1 n 2  h h
R = radius of nucleus of mass number A.
n=∞
n=7 * Nucleus density is of the order of 1017 kg/m3
n=6
n=5 –0.85 eV Isomers are nuclides which have identical atomic
Kδ Lγ Pfund
n=4 number and mass number but differ in their energy
Kγ Lβ Brackett (I.R.)
n=3 (I.R.) –1.5 eV states.
Paschen

Kβ (I.R.) Nuclear binding energy ∆mc 2
n=2 –3.4 eV =
Balmer Nucleon A
(Visible)
where ∆m = mass defect

n=1
Limiting line of Lyman series
–13.6 eV [ Zm p + (A − Z)m n − M ]c 2
Lyman Series
=
A
(U.V. rays)

XtraEdge for IIT-JEE 27 APRIL 2010


* The binding energy per nucleon is small for small (f) In a nuclear fusion reaction small nuclei fuse to
nuclei. give big nuclei whereas in a nuclear fusion reaction a
* For 2 < A < 20, there are well defined maxima big nuclei breaks down.
which indicate that these nuclei are more stable. Thermal neutrons produce fission in fissile nuclei.
* For 30 < A < 120 the average B.E./A is 8.5 MeV / Fast moving neutrons, when collide with atoms of
nucleon with a peak value of 8.8 MeV for Iron. comparable masses, transfer their kinetic energy to
colliding particle and slow down.
* For A > 120, there is a gradual decreases in
B.E./nucleon. ∆λ v
According to Doppler's effect of light =
* More the B.E./A, more is the stability. λ c

Radioactivity : E nhν nhc


Power, P = = =
t t λt
β particles are electrons emitted from the nucleus.
(n → p + β) out put
η=
In put
(a) N = N0e–λt
−dN dN
(b) = λN where = activity level
dt dt
Solved Examples
t
n
1  1  T1 / 2
(c) N = N0   = N0  
2 2 1. The energy of an excited hydrogen atom is –3.4 eV.
n Calculate the angular momentum of the electron
1
⇒ A = A0   where A = activity level according to Bohr theory.
2
Sol. The energy of the electron in the nth orbit is
0.693 13.6
(d) T1/2 = En = – eV
λ
n2
1 13.6
(e) τ = Here, – = –3.4
λ n2
(f) τ = 1.4 T1/2 or n=2
2.303 N 2.303 A nh 2 × 6.63 × 10 −34
(g) t = log10 0 = log10 0 Angular momentum = =
λ N λ A 2π 2 × 3.14
2.303 m = 2.11 × 10–34 Js.
= log 0
λ m
2. The wavelength of the first member of the Balmer
(h) If a radioactive element decays by simultaneous
series in the hydrogen spectrum is 6563 Å. Calculate
−dN the wavelength of the first member of the Lyman
emission of two particle then = λ1N + λ2N
dt series.
The following parameters remain conserved during a Sol. For the first member of the Balmer series
nuclear reaction
1 1 1 5R
(a) linear momentum = R 2 − 2  = ...(1)
λ  2 3  36
(b) Angular momentum
For the first member of the Lyman series
(c) Number of nucleons
1 1 1 3R
(d) Charge = R 2 − 2 = ...(2)
λ´ 1 2  4
(e) The energy released in a nuclear reaction
Dividing Eq. (1) by Eq. (2)
X+P→Y+Z+Q
λ´ 5× 4 5
Q = [mx + mp) – (my + mz)]c2 = ∆m × c2 = =
λ 36 × 3 27
Q = ∆m × 931 MeV
5 5
or λ´ = λ= × 6563 = 1215 Å
27 27

XtraEdge for IIT-JEE 28 APRIL 2010


3. Hydrogen atom in its ground state is excited by Sol. Initial kinetic energy of the electron = 50.0 keV
means of a monochromatic radiation of wavelength Energy of the photon produced in the first collision,
970.6 Å. How many different wavelengths are E1 = 50.0 – 25.0 = 25.0 keV
possible in the resulting emission spectrum ? Find the
Wavelength of this photon
longest wavelength amongst these.
Sol. Energy the radiation quantum hc 6.6 × 10 −34 × 3 × 108
λ1 = =
E1 1.6 × 10 −19 × 12.5 × 10 3
hc 6.6 × 10 −34 × 3 × 108
E = hv = = = 0.99 × 10–10 m
λ 970.6 × 10 −10 × 1.6 × 10 −19
= 12.75 eV = 0.99 Å

Energy of the excited sate Kinetic energy of the electron after third collision = 0

En = – 13.6 + 12.75 = – 0.85 eV Energy of the photon produced in the third collision ,
E3 = 12.5 – 0 = 12.5 keV
13.6
Now, we know that En = – This is same as E2. Therefore, wavelength of this
n2
photon, λ3 = λ2 = 0.99 Å
13.6 −13.6
or n2 = – = = 16
En − 0.85 5. In an experiment on two radioactive isotopes of an
or n = 4 elements (which do not decay into each other), their
The number of possible transition in going to the mass ratio at a given instant was found to be 3. The
ground state and hence the number of different rapidly decaying isotopes has larger mass and an
wavelengths in the spectrum will be six as shown in activity of 1.0 µCi initially. The half lives of the two
the figure. isotopes are known to be 12 hours and 16 hours.
What would be the activity of each isotope and their
n
4 mass ratio after two days ?
3 Sol. We have, after two days, i.e., 48 hours,
4
1
N1 = N10   = N10 /16
2 2
3
1
N2 = N 02   = N 02 /8
2
1
N1 N0 8 3× 8 3
The longest wavelength corresponds to minimum Mass ratio = = 10 . = =
N2 N 2 16 162 2
energy difference, i.e., for the transition 4 → 3.
13.6 Now, A10 = λ1 N10 = 1.0 µCi
Now E3 = – = – 1.51 eV
32 After two days,
hc A1 = λ1N1 = λ1 N10 /16 = A10 /16 = (1/16)µCi
= E4 – E3
λ max A2 = λ2N2 = λ2 N 02 /8
6.6 × 10 −34 × 3 × 108 λ2 T 12 3
or λmax = But = 1 = =
(1.51 − 0.85) × 1.6 × 10 −19 λ1 T2 16 4
= 18.75 × 10–7m = 18750 Å 3
or λ2 = λ1
4
4. X-rays are produced in an X-ray tube by electrons 3  1  1
A2 =  λ1  ×  N10  ×
accelerated through a potential difference of 50.0 kV.  4   3  8
An electron makes three collisions in the target
before coming to rest and loses half its kinetic energy 1 1
= λ1 N10 = A10
in each of the first two collisions. Determine the 32 32
wavelengths of the resulting photons. Neglect the = (1/32) µCi
recoil of the heavy target atoms.

XtraEdge for IIT-JEE 29 APRIL 2010


KEY CONCEPT

Organic AROMATIC
Chemistry
Fundamentals HYDROCARBON
Halogenation of Benzene : The function of the Lewis acid can be seen in step 1.
Benzene does not react with bromine or chlorine The ferric bromide reacts with bromine to produce a
unless a Lewis acid is present in the mixture. (As a positive bromine ion, Br+ (and FeBr4–). In step 2 this
consequence, benzene does not decolorize a solution Br+ ion attacks the benzene ring to produce an
of bromine in carbon tetrachloride.) When Lewis arenium ion. Then, finally in step 3 a proton is
acids are present, however, benzene reacts readily removed from the arenium ion by FeBr4–. This results
with bromine or chlorine, and the reactions give in the formation of bromobenzene and hydrogen
bromobenzene and chlorobenzene, respectively, in bromide the products of the reaction. At the same
good yields : time this step regenerates the catalyst, FeBr3.
Cl
FeCl3 Nucleophilic Aromatic Substitution through an
+ Cl2 + HCl
25ºC Elimination – Addition Mechanism : Benzyne
Chlorobenzene (90%)
Although aryl halides such as chlorobenzene and
Br bromobenzene do not react with most nucleophiles
FeBr3
+ Br2 + HBr under ordinary circumstances, they do react under
heat
highly forcing conditions. Chlorobenzene can be
Bromobenzene (75%)
converted to phenol by heating it with aqueous
The Lewis acids most commonly used to effect sodium hydroxide in a pressurized reactor at 350ºC .
chlorination and bromination reactions are FeCl3,
FeBr3, and AlCl3, all in the anhydrous form. Cl ONa OH
A mechanism for the reaction :
Electrophillic Aromatic Bromination : + NaOH 350ºC
H2O H3O+
Step 1
+
Phenol

Br – Br : + FeBr3 → :Br – Br – FeBr3 Bromobenzene reacts with the very powerful base,
NH2– , in liquid ammonia :

→ :Br+ + :Br – FeBr3 Br NH2
Bromine combines with FeBr3 to form a complex that dissociates
+ – -33ºC
to form a positive bromine ion and FeBr4–
+ K :NH2 + KBr
Step 2 NH3
+ slow Aniline
+ Br:
These reactions take place through an elimination –
H H + H addition mechanism that involves the formation of an
Br: Br: Br: interesting intermediate called benzyne (or
+ + dehydrobenzene). We can illustrate this mechanism
with the reaction of bromobenzene and amide ion.
Arenium ion In the first step, the amide ion initiates an elimination
The positive bromine ion attacks benzene to by abstracting one of the ortho protons because they
form an arenium ion are the most acidic. The negative charge that
Step 3 develops on the ortho carbon is stabilized by the
– inductive effect of the bromine. The anion then loses
H :Br – FeBr3 a bromide ion. This elimination produces the highly
Br: Br: unstable, and thus highly reactive, benzyne. Benzyne
+ H – Br: + FeBr3
+ then reacts with any available nucleophile (in this
case, an amide ion) by a two-step addition reaction to
A proton is removed from the arenium ion
to become bromobenzene produce aniline.

XtraEdge for IIT-JEE 30 APRIL 2010


The Benzyne Elimination – Addition Mechanism : CF3
Elimination NH2

Br Br CF3 4
(–NH3) (–Br–) –
:NH2 Less stable carbanion

H CF3 CF3
Benzyne
– 2
:NH2 (or dehydrobenzene) – :NH3
– –
:NH2 NH2 NH2 +:NH2
3
NH2 More stable carbanion
(The negative charge is closer to the
– electronegative trifluoromethyl group)
Addition
Carbanion 3 then accepts a proton from ammonia to
:NH3 form m-(trifluoromethyl) aniline.
NH2 – Carbanion 3 is more stable than 4 because the carbon
atom bearing the negative charge is closer to the
+ :NH2 highly electronegative trifluoromethyl group. The
H trifluoromethyl group stabilizes the negative charge
Evidence for an elimination-addition mechanism : through its inductive effect. (Resonance effects are
not important here because the sp2 orbital that
When 14C-labeled (C*) chlorobenzene is treated with
contains the electron pair does not overlap with the π
amide ion in liquid ammonia, the aniline that is orbitals of the aromatic system.)
produced has the label equally divided between the 1 The Birch Reduction :
and 2 positions. Benzene can be reduced to 1, 4-cyclohexadiene by
* NH2 treating it with an alkali metal (sodium, Lithium, or
potassium) in a mixture of liquid ammonia and an
* Cl K+NH – * NH2– alcohol.
2
(50%)
NH3 Na
*
NH3, EtOH
NH2 Benzene 1, 4-cyclohexadiene
Elimination Addition (50%)
A Mechanism for the Reaction :
When the ortho derivative 1 is treated with sodium Brich Reduction :
amide, the only organic product obtained is Na
m-(trifluoromethyl) aniline : etc.
– –
CF3 CF3 Benzene Benzene radical anion
Cl
NaNH2 The first electron transfer produces a delocalized
benzene radical anion.
NH3
NH2 EtOH
1
m-(Trifluoromethy)aniline
H H etc.
This result can also be explained by an elimination –
addition mechanism. The first step produces the H H
benzyne 2 : Cyclohexadienyl radical
Protonation produces a cyclohexadienyl radical
CF3 CF3 (also a delocalized species)
Cl Na
NaNH2 H
+ Cl– –
NH3 H
H etc. EtOH
1 2 H H
H –
H H
This benzyne then adds an amide ion in the way that Cyclohexadienyl anion 1,4-Cyclohexadiene
produces the more stable carbanion 3 rather than the Transfer of another electron leads to the formation of a delocalized
cyclohexadienyl anion, and protonation of this produces the
less stable carbanion 4 : 1,4-cyclohexadiene.

XtraEdge for IIT-JEE 31 APRIL 2010


KEY CONCEPT

Physical
Chemistry
SOLUBILITY
Fundamentals
PRODUCT
Solubility : representing the dissociation of the salt at a given
The amount of a solute, dissolved in a given volume temperature when the solution is saturated.
of a solvent (water) in 100 mL or in 1L to form a Solubility product is not the ionic product under all
saturated solution at a given temperature is termed as conditions but only when the solution is saturated.
the solubility of the solute. Ionic product has a broad meaning since it is
Solubility Product : applicable to all types of solutions, may be saturated
Salts like AgI, AgCl, PbI2, BaSO4, PbSO4 etc. are or unsaturated.
ordinarily considered insoluble but they do possess Relationship between solubility and solubility product :
some solubility. These are sparingly soluble salts. A The equilibrium for a saturated solution of any
saturated solution of sparingly soluble salt contains a sparingly soluble salt be expressed as :
very small amount of the dissolved salt. It is assumed Ax B y x Ay+ + y Bx–
that whole of the dissolved salt is present in the form
Thus, solubility product, Ksp = [Ay+]x [Bx–]y .
of ions, i.e., it is completely dissociated. Consider a
sparingly soluble salt like AgCl, the following Let 's' mole per litre be the solubility of the salt, then
equilibrium occurs between the undissolved solid salt Ax B y x Ay+ + y Bx–
and the silver and chloride ions in the saturated xs ys
solution. So KSP = [xs]x [ys]y = xx.yy(s)x+y
AgCl (s) Ag+ (aq) + Cl–(aq) Since the solubility of a salt varies with temperature,
Applying the law of mass action to the ionic the numerical value of Ksp for a salt changes with
equilibrium, temperature; values usually recorded at 25ºC.
[Ag + ][Cl − ] Common Ion Effect :
K= The suppression of the degree of dissociation of a
[AgCl(s)]
weak acid or a weak base by the addition of a strong
or K[AgCl(s) = [Ag+] [Cl–] electrolyte containing a common ion. The common
The concentration of solid AgCl in the solid state i.e. ion effect play an important role in the qualitative
[AgCl(s)] is constant at a particular temperature, no analysis.
matter how much solid is present in contact with the Application of solubility product in qualitative analysis:
solution. It follows that
Precipitation of sulphides of group II. Sulphides of
[AgCl(s)] = K´ = constant group II are precipitated by passing H2S gas through
Hence, [Ag+] [Cl–] = KK´ = Ksp (constant) the solution of these cations in presence of dil HCl.
Ksp is termed as the solubility product. It is defined as H2S being a weak electrolyte ionizes only sligthtly,
the product of the concentration of ions in a saturated while HCl being a strong electrolyte is almost
solution of a salt at a given temperature. Consider, in completely ionized.
general, the salt of the type AxBy which is dissociated H2S 2H+ + S2– ; HCl → H+ + Cl–
as : Thus, the common ion effect takes place. As a result,
AxBy x Ay+ + y Bx– the degree of dissociation of H2S decreases
Applying law of mass action, sufficiently and the concentration of S2– ions in the
solution becomes very small. But with this low
[A y + ]x [B x − ] y
=K concentration of second group and the sulphide ions
[A x B y ] exceeds the very low solubility products of their
when the solution is saturated, corresponding sulphides. Therefore, the cations of
group II get precipitated as their insoluble sulphides.
[Ax By] = K´ (constant)
On the other hand, the sulphides of the cations of the
or [Ay+]x [Bx–]y = K[AxBy] = KK´ = Ksp (constant)
other groups (III, IV, V and Mg) are not precipitated
Thus, solubility product is defined as the product of under these conditions because their solubility
concentrations of the ions raised to a power equal to products are quite high.
the number of times the ions occur in the equation

XtraEdge for IIT-JEE 32 APRIL 2010


Precipitation of the hydroxides of group III : However, under these conditions, Mg salts do not get
Hydroxides of group III are precipitated by adding an precipitated as MgCO3 since its solubility product is
excess of solid NH4Cl to the solutions of these comparatively high and thus requires a high
cations followed by the addition of excess of concentration of CO32– ions for precipitation. The
NH4OH. Being a weak electrolyte, NH4OH is only carbonates of Na+, K+ and NH +4 ions are also not
slightly ionised, whereas NH4Cl, being a strong
precipitated because they are quite soluble.
electrolyte, ionizes almost completely to give at large
The necessity of adding NH4OH arises due to the fact
concentration of NH4+ ions.
that (NH4)2CO3 solution usually contains a large
NH4OH NH4+ + OH–; NH4Cl → NH4+ + Cl– amount of NH4HCO3. Thus, the cations of group V
Due to the common ion effect, the degree of will form not only insoluble carbonates but soluble
dissociation of NH4OH gets suppressed and hence the bicarbonates as well. As a result, the precipitation
concentration of OH– ions in solution decreases will not be complete. In order to convert NH4HCO3
appreciably. But even with this low conc. of OH– to (NH4)2CO3, NH4OH is always added.
ions, the ionic products of the cations of group III and NH4HCO3 + NH4OH → (NH4)2CO3 + H2O
OH–ions exceed the low values of the solubility Preferential precipitation of Salts :
products of their corresponding metal hydroxides. As A solution contains more than one ion capable of
a result, the cations of group III get precipitated as forming a precipitate with another ion which is added
their insoluble hydroxides. to the solution. For example, in a solution containing
On the other hand, cations of groups IV, V and Mg, Cl–, Br–, and I– ions, if Ag+ ions are added, then out
which require a large conc. of OH– ions due to their of the three, the least soluble silver salt is precipitated
high solubility products will not be precipitated. first. If the addition of Ag+ ions is continued,
Precipitation of sulphides of group IV. The eventually a stage is reached when the next lesser
sulphides of group IV are precipitated by passing H2S soluble salt starts precipitating along with the least
through ammoniacal solution of these cations. soluble salt and so on. If the stoichiometry of the
Both H2S and NH4OH, being weak electrolytes, precipitated salts is the same, then the salt with the
ionize only slightly as : minimum solubility product (and hence also the
H2S 2H+ + S2– minimum solubility) will precipitate first followed by
the salt of next higher solubility product and so on.
NH4OH NH +4 + OH– If the stoichiometry of the precipitated salts is not the
The H+ ions and OH– ions combine to produce same, then, from the solubility product data alone, we
practically unionised molecules of water cannot predict which ion will precipitate first. Take,
for example, a solution containing Cl– and CrO42–.
H+ + OH– → H2O
Both these ions form precipitates with Ag+. Through
As a result, the above dissociation equilibrium the solubility product product of AgCl is larger than
reactions get shifted in the forward direction, so that that of Ag2CrO4, yet it is AgCl (lesser soluble) which
the concentration of S2– ions goes on increasing. precipitates first when Ag+ ions are added to the
Ultimately, the ionic product of the cations of group solution. In order to predict which ion (Cl– or CrO42–)
IV and S2– ions exceed the solubility products of their precipitates first, we have to calculate the
corresponding metal sulphides and hence get concentration of Ag+ ions needed to start the
precipitated. precipitation through the solubility product data and
Precipitation of carbonates of group V : The the given concentration of Cl– or CrO42–. Since square
carbonates of group V are precipitated by adding root is involved in the expression for computing Ag+
(NH4)2CO3 solution to the solution of these cations in for silver chromate, the quantity of Ag+ needed to
the presence of NH4Cl and NH4OH. (NH4)2CO3, start the precipitation of CrO42– is larger than that for
being a weak electrolyte ionises only slightly to give Cl–. Hence, as AgNO3 is added to the solution, the
a small concentration of CO32– ions. minimum of the two concentrations of Ag+ to start
(NH4)2CO3 2NH4+ + CO32– the precipitation will be reached first and thus the
On the other hand, NH4Cl being a strong electrolyte, corresponding ion (Cl– in this case) will be
ionises almost completely to give a large precipitated in preference to the other. During the
course of precipitating, concentration of Cl–
concentration of NH +4 ions. Due to the common ion decreases and the corresponding concentration of Ag+
effect, the dissociation of (NH4)2CO3 is suppressed to start the precipitation increases. Its concentration
and hence the concentration of CO32– ions in the eventually becomes equal to the value required for
solution decreases considerably. But even with this CrO42–. At this stage, practically the whole of Cl–
low concentration of CO32– ions, the ionic products of ions have been precipitated. The addition of more of
these cations and CO32– ions exceed the low values of AgNO3 causes the precipitation of both the ions
the solubility products of their corresponding metal together.
carbonates and thus get precipitated.

XtraEdge for IIT-JEE 33 APRIL 2010


UNDERSTANDING
Organic Chemistry

1. An organic compound (A), C4H9Cl, on reacting with Hence (D) is acetone and (E) is formaldehyde.
aqueous KOH gives (B) and on reaction with Therefore, alkene (C) is 2-methyl propene.
alcoholic KOH gives (C) which is also formed on CH3 – C = CH2
passing vapours of (B) over heated copper. The
compound (C) readily decolourise bromine water. CH3
Ozonolysis of (C) gives two compounds (D) and (E). (C)
Compound (D) reacts with NH2OH to give (F) and (D) reacts with hydroxyl amine (NH2OH) to form
the compound (E) reacts with NaOH to give an oxime (F).
alcohol (G) and sodium salt (H) of an acid. (D) can CH3 –H2O CH3
also be prepared from propyne on treatment with C = O + H2 NOH C = NOH
CH3 CH3
water in presence of Hg++ and H2SO4. Identify (A) to (D) (F)
(H) with proper reasoning.
OH Cl
Sol. | |
Alc. KOH Thus, (B) is CH 3 − C − CH 3 and (A) is CH 3 − C − CH 3
C4H9Cl C4H8 | |
(A) ∆;– HCl (C) CH 3 CH 3
(Alkyl halide) (Alkene) Reactions :
Cl OH
Aq. KOH Cu
C4H9OH Aq. KOH
CH3 – C – CH3 CH3 – C – CH3
∆; –KCl (B) ∆; –H2O ∆; –KCl

Cu/300ºC
–H2O
(Alcohol) CH3 CH3
We know that p-alcohol on heating with Cu gives (A) (B)
aldehyde while s-alcohol under similar conditions CH3 – C = CH2 + H2O
gives ketone. Thus, (B) is a t-alcohol because it, on
heating with Cu gives an alkene (C). Since a CH3 (C)
t-alcohol is obtained by the hydrolysis of a t-alkyl
halide, hence (A) is t-butyl chloride. Alc. KOH/∆
Thus, (A) is CH3 – C = CH2
–KCl; –H2O
Cl OH CH3
| |
CH 3 − C − CH 3 and (B), is CH 3 − C − CH 3 (C)
| | O
CH 3 CH 3 (I) O3 CH3
The alkene (C) on ozonolysis gives (D) and (E), CH3 – C = CH2 C = O + H –C – H
(II) H2O/Zn CH3
hence (C) is not symmetrical alkene. In these (D) (E)
compound (E) gives Cannizaro's reaction with CH3
NaOH. So, (E) is an aldehyde which does not contain (C)
α - H atom. Hence it is HCHO. Compound (D) can
also be prepared by the hydration of propyne in the CH3 ∆ CH3
C = O + H2NOH C = NOH
presence of acidic solution and Hg++ CH3 –H2O
CH3
(D) (F)
Hg++
CH3 – C ≡ CH + H2O CH3 – C = CH2
H+
OH 2HCHO + NaOH → CH3OH + HCOONa
(E) (G) (H)
CH3 – C – CH3 O
Hg++
O CH3 – C ≡ CH + H2O CH3 – C – CH3
(D) H+
(D)

XtraEdge for IIT-JEE 34 APRIL 2010


2. An organic compound (A) C7H15Cl on treatment with Thus, both (I) and (II) give main product as 2-methyl
alcoholic KOH gives a hydrocarbon (B) C7H14. (B) hexene-2, hence (A) is either (I) or (II).
on treatment with O3 and subsequent hydrolysis gives Cl
acetone and butyraldehyde. What are (A) and (B) ?
Explain the reactions. (A) CH3 – C – CH2CH2CH2CH3
Sol. In numerical, following data are given : CH3
O
C 7 H15 Cl Alc
. KOH
→ C 7 H14 →
3
C 7 H14 O 3 2-chloro-2-methyl hexane

(A) ( B) Ozonide
Alkyl halide Alkene or CH3 – CH – CH – CH2CH2CH3
H 2 O / Zn
 → CH3COCH3 + CH3CH2CH2CHO CH3 Cl

The alkene contains seven carbon atoms. The 3-chloro-2-methyl hexane
position of C=C double bond can be located (B) CH3 – C = CH.CH2CH2CH3
as follows : CH3
H 2-methyl hexene-2
CH3 –2[O]
C = O + O = C.CH2CH2CH3 3. An unsaturated hydrocarbon (A), C6H10 readily gives
CH3
(B) on treatment with NaNH2 in liquid NH3. When
CH3 – C = CH.CH2CH2CH3
(B) is allowed to react with 1-chloro propane, a
CH3 compound (C) is obtained. On partial hydrogenation
Thus, alkene (B) is 2-methyl hexene-2 in the presence of Lindlar catalyst (C) gives (D),
The ozonolysis reaction is as follows : C9H18. On ozonolysis (D) gives 2, 2-dimethyl
O3 propanal and butanal. Give structures of (A), (B), (C)
CH3 – C = CH.CH2CH2CH3 and (D) with proper reasoning.
(B) O O Sol. The structure of compound (D) can be obtained by
CH3 joining the products of ozonolysis.
CH3 – C CH – CH2CH2CH3 CH3
O –2[O]
CH3 CH3 – C – CH = O + O = CH.CH2CH2CH3
H2O/Zn Butanal
CH3 – C = O + O = C – CH2CH2CH3 CH3

2,2-dimethyl propanal CH3
CH3 H
Since alkene (B) is produced by the removal of one CH3 – C – CH = CH.CH2CH2CH3
mol of HCl from alkyl halide (A) and thus (A) can be CH3
either (I) or (II). 2,2-dimethyl heptene-3 (D)
Cl Ozonolysis equation of (D) is :
or CH3
(I) CH3 – C – CH2CH2CH2CH3
(I) O3
CH3 – C – CH = CHCH2CH2CH3
CH3 (II) H2O/Zn
(II) CH3 – CH – CH – CH2CH2CH3 CH3
(D) CH3
CH3 Cl
The dehydro halogenation reaction by(I) or (II) yeilds CH3 – C – CHO + CH3CH2CH2CHO
Cl
CH3
KOH
CH3 – C – CH2CH2CH2CH3 Alkene (D) is obtained by the partial hydrogenation
alc.
of (C), thus (C) contains a – C≡C – triple bond at C3.
CH3
(I). CH3 – C = CH.CH2CH2CH3
CH3
CH3 H2
Main product (Saytzeff's rule) CH3 – C – C ≡ C – CH2CH2CH3
Lindlar catalyst
KOH
CH3 – CH – CH – CH2CH2CH3 CH3
Alc.
(C) CH3
CH3 Cl
(II) CH3 – C = CH.CH2CH2CH3 CH3 – C – CH = CHCH2CH2CH3
CH3 CH3
Main product (Saytzeff's rule) (D)

XtraEdge for IIT-JEE 35 APRIL 2010


The starting compound (A) reacts with NaNH2 in Sol. Determination of empirical formula of (A) :
presence of liquid NH3. It means it contains one
–C≡CH at the terminal carbon, and, therefore gives a Atomic Relative no.
mono sodium derivative. Element % Simplest ratio
wt. of atoms
NaNH
C 6 H10  
2
→ C4H9 – C ≡ C.Na
(A)
NH 3 ( B) 90.56 7.55
C 90.56 12 = 7.55 = 1 or 4
Compound (B) reacts with 1-chloro propane to give 12 7.55
compound (C) as follows : 9.44
∆ 9.44 = 1.25
C4H9 – C ≡ C – Na + Cl – CH2CH2CH3 H 9.44 1 = 9.44 7.55
–NaCl 1
(B) 1-chloro propane or 5

C4H9 – C≡C – CH2CH2CH3 The empirical formula of (A) = C4H5


(C)
Empirical formula weight = 48 + 5 = 53
CH3 Molecular weight = V.D. × 2 = 53 × 2 = 106
But, (C) is CH3 – C – C ≡ C – CH2CH2CH3 Molecular wt. 106
Hence, n = = =2
CH3 Empirical wt. 53
Now, putting the value of C4H9 as a t-butyl radical, Molecular formula = 2 × C4H5 = C8H10
we have : The given equation may be outlined as follows :
Vigrous oxidation
COOH
CH3 CH3 C8H10 C6H4 + 2H2O
6[O]
(A) (B) COOH
CH3 – C – C≡C – H NaNH2 CH3 – C – C≡CNa + NH3
Meq. of dicarboxylic acid = Meq. of NaOH
CH3 CH3 0.1× 1,000
(A) (B) = 24.1 × 0.05
E
Equivalent of acid = 83
Hence, Molecular wt. = Basicity × Equivalent weight
CH3 = 2 × 83 = 166
(A) CH3 – C – C≡CH Since (B) on heating with soda-lime gives benzene,
the C6H4 represents to benzene nucleus having two
CH3 side chains, thus (B) is a benzene dicarboxylic acid.
(3,3-dimethyl butyne-1)
There are three benzene dicarboxylic acids.
CH3 COOH COOH
(B) CH3 – C – C≡CNa COOH
COOH COOH
CH3 COOH
Phthalic acid Isophthalic acid Terphthalic acid
CH3
All the above three acids are obtained by the
(C) CH3 – C – C≡C – CH2CH2CH3 oxidation of respectively xylenes.
CH3 CH3 6[O] COOH
+ 2H2O
CH3 CH3 COOH
o-xylene
(D) CH3 – C – CH = CHCH2CH2CH3
CH3 6[O] COOH
CH3 + 2H2O
CH3 COOH
4. A hydrocarbon (A) [C = 90.56%, V.D. = 53] was m-xylene
subjected to vigrous oxidation to give a dibasic acid CH3 COOH
(B). 0.10 g of (B) required 24.10 ml of 0.05 N NaOH 6[O]
for complete neutralization. When (B) was heated + 2H2O
strongly with soda-lime it gave benzene. Identify (A) CH3 COOH
and (B) with proper reasoning and also give their p-xylene
structures.

XtraEdge for IIT-JEE 36 APRIL 2010


All the above three acids on heating with soda-lime Sol. (i) Acetic acid on heating with C2H5OH gives
yields only benzene. original compound (A).
COOH COOH CH3COOH + C2H5OH H2SO

 4
→ CH 3COOC 2 H 5
, , ∆ (A)
COOH
COOH + H2O
COOH
(ii) CH3COOC2H5 (A) on heating with C2H5ONa
NaOH + CaO

+ 2CO2 undergoes Claisen condensation to give (B), which is
COOH aceto acetic ester.
C2H5ONa
CH3CO OC2H5 + H CH2COOC2H5
Of the three acids, one which on heating gives an Reflux
(A)
anhydride, is o-isomer. + C2H5OH + CH3COCH2COOC2H5
COOH (B)
∆ CO
O (iii) (B) on heating in acidic solution gives (C) and
COOH –H2O CO
ethyl alcohol.
+
One acid which on nitration gives a mono nitro CH 3COCH 2 COOC 2 H 5 + HOH H→
compound is p-dicarboxylic acid. ( B)

COOH COOH CH 3COCH 2 COOH + C2H5OH


HNO3 NO2 (C)
∆; H2SO4
COOH (iv) (C) on decarboxylation gives acetone (D).
COOH

CH 3COCH 2 COOH → CH 3COCH 3
− CO 2
One acid which on nitration gives three mono nitro (C) ( D)

compounds will be the m-isomer. (v) (D) reacts with NaNH2 to form sodium salt (E),
COOH COOH COOH which on heating with CH3I gives butanone (F).
HNO3 NO2 ∆
COOH H2SO4 COOH COOH CH 3COCH 3 + NaNH2 → CH 3COCH 2 Na
− NH 3
( D) (E)
NO2
COOH
CH I
 3
– NaI
→ CH 3COCH 2 CH 3
NO2 COOH ( F)


(vi) CH 3COCH 2 CH 3 + 3I2 + 4NaOH →
5. Two moles of an anhydrous ester (A) are condensed ( F)
in presence of sodium ethoxide to give a β-keto ester
CHI3 + CH3CH2COONa + 3NaI + 3H2O
(B) and ethanol. On heating in an acidic solution

compound (B) gives ethanol and a β-keto acid (C). (vii) CH 3COCH 3 + 3I2 + 4NaOH →
( D)
(C) on decarboxylation gives (D) of molecular
formula C3H6O. Compound (D) reacts with sodamide CHI3 + CH3COONa + 3NaI + 3H2O
to give a sodium salt (E), which on heating with CH3I CH3COONa HCl
→ CH3COOH + NaCl
gives (F), C4H8O, which reacts with phenyl hydrazine
Thus, (A) CH3COOC2H5
but not with Fehling reagent. (F) on heating with I2
and NaOH gives yellow precipitate of CHI3 and (B) CH3COCH2COOC2H5
sodium propionate. Compound (D) also gives (C) CH3COCH2COOH
iodoform, but sodium salt of acetic acid. The sodium (D) CH3COCH3
salt of acetic acid on acidification gives acetic acid
(E) CH3COCH2Na
which on heating with C2H5OH in presence of conc.
H2SO4 gives the original ester (A). What are (A) to (F) CH3COCH2CH3
(F) ?

XtraEdge for IIT-JEE 37 APRIL 2010


Set

`tà{xÅtà|vtÄ V{tÄÄxÇzxá 12
This section is designed to give IIT JEE aspirants a thorough grinding & exposure to variety
of possible twists and turns of problems in mathematics that would be very helpful in facing
IIT JEE. Each and every problem is well thought of in order to strengthen the concepts and
we hope that this section would prove a rich resource for practicing challenging problems and
enhancing the preparation level of IIT JEE aspirants.
By : Shailendra Maheshwari
So lu tions pub lished in this issue Joint Director Academics, Career Point, Kota

1. Prove that, if n is a positive integer, 6. A number is chosen at random from the set
a {1, 2, 3, …......, 2006}. What is the probability that
∫e
−x
x n dx =
0 it has no prime factor in common with
 10 ! ?
 a2 a n 
n ! 1 − e −a 1 + a + + ... + 
  2! n ! 

7. Two vertices of a triangle are a – î + 3 ĵ and 2î + 5 ĵ

Also, deduce the value of ∫ 0
e − x x n dx and its orthocenter is at ( î + 2 ĵ ). Find the position
vector of third vertex.
2. Let A ≡ (6, 5), B ≡ (2, –3) and C ≡ (–2, 1) be the
8. Show that an equilateral triangle is a triangle of
vertices of a triangle. Find the point P in the interior
maximum area for a given perimeter and a triangle of
of the triangle such that ∆PBC is an equilateral
minimum perimeter for a given area.
triangle.

9. The bottom of a tank with a capacity of 300 litres is


3. If Sn = nC1 + 2.nC2 + 3.nC3 + ....... + n nCn then find
covered with a mixture of salt and some insoluble
n

∑S n . Also prove that substance. Assuming that the rate at which the salt
n =1 dissolves is proportional to the difference between
n +1
n
C1 . (nC2)2 . (nC3)3.... (nCn)n ≤  2n 
C2
. the concentration at the given time and the
 
 n +1
  concentration of a saturated solution (1 kg of salt per
3 litres of water) and that the given quantity of pure
4. Let z1, z2, z3 be three distinct complex numbers water disolves 1/3 kg of salt in 1 minute. Find the
satisfying quantity of salt in solution at the expiration of one
|z1 – 1| = |z2 – 1| = |z3 – 1|; Let A, B and C be the hour.
points represented in the argand plane corresponding
to z1, z2 and z3 respectively. Prove that z1 + z2 + z3 = 10. An isosceles triangle with its base parallel to the
3 if and only if ∆ABC is an equilateral triangle. x2 y2
major axis of the ellipse + = 1 is
9 3
5. Let A ≡ (r, 0) be a point on the circle x2 + y2 = r2 and
circumscribed with all the three sides touching
D be a given point inside the circle. If BC be any
the ellipse. The least possible area of the
arbitrary chord of the circle thorugh point D. Prove
triangle is.
that the locus of the centroid of triangle ABC is a
circle whose radius is less than r/3.

XtraEdge for IIT-JEE 38 APRIL 2010


MATHEMATICAL CHALLENGES
SOLUTION FOR MARCH ISSUE (SET # 11)

1. Let z = x + iy so given 1


1 ≤ x or 1 ≤ r cos θ , if z = ∑eiθNow, Now, as g(x) = x2 f  
x
1 − x − iy (1 − x − iy)(1 + x − iy)
= 1 1
1 + x + iy (1 + x ) 2 + y 2 = (1 + x) f(1) + (x2 – 1) f(0) + (1 – x) f(–1)
2 2
(12 − x 2 ) − y 2 so 2 | g(x) | ≤ |x + 1| + 2 |1 – x2| + |1 – x|
Real part ≤ 0 as x ≥ 1 given and
(1 + x ) 2 + y 2 2 | g(x) | ≤ x + 1 + 2(1 – x2) + 1 – x
imaginary part as x ∈ [–1, 1]
2 |g(x)| ≤ –2x2 + 4 ≤ 4
−(1 − x ) y − (1 + x ) y −2 y
2 2
= ≤0 |g(x)| ≤ 2
(1 + x ) + y (1 + x ) 2 + y 2
as y ≥ 0 given 1  2y 
1− z 4. 1− y´
so ∩ 0 is true. a  2 a 2 − y2 
1+ z  
1 −2 y.y´ 1
2. As ∠ POQ = 90º = . – y´
P
2
a+ a −y 2 2
2 a −y 2 y

1  y y 1
= y´  − − 
a  a a 2 − y 2 (a + a 2 − y 2 ) a 2 − y 2 y 
O 90–θ  
θ
C y´ y 2 (a + a 2 − y 2 ) − y 2 a − a (a + a 2 − y 2 ) a 2 − y 2 
 
= 
Q 2 2
ay(a + a − y ) a − y 2 2

so CP = (OC) . tan (90 – θ)


= (OC) cot θ & CQ = (OC) tan θ y(a + a 2 − y2 ) a 2 − y2
so CP. CQ = (OC)2 = r2
when r is the radius of circle. = y´[y2a + y2 a 2 − y 2 – y2a – a2 a 2 − y 2 – a(a2 – y2)]
3. f(0) = c y (a + a 2 − y2 ) a 2 − y 2 = –y´(a2 – y2) (a + a 2 − y2 )
f(1) = a + b+ c
& f(–1) = a – b + c y
y´ = –
1 a − y2
2
solving these, a = [f(1) + f(–1) – 2f(0)]
2 −β
1 y´M =
b = [f(1) – f(–1)] & c = f(0) a − β2
2
2
x ( x + 1) x ( x − 1) −β
so f(x) = f(1) + (1 – x2) f(0) + f(–1) y= x ...(1)
2 2 a − β2
2
2 | f(x) | ≤ |x| |x + 1| + 2 |1 – x2| + |x| |x – 1|; as
y=β ...(2)
|f(1)|, |f(0)|, |f(–1)| ≤ 1.
locus of intersection of these two lines is
2 |f(x)| ≤ |x| (x + 1) + 2(1 – x2) + |x| (1 – x) as
−y
x ∈ [–1, 1] y= x
5 a − y2
2
so 2 |f(x)| ≤ 2(|x| + 1 – x2) ≤ 2 .
4 a2 – y2 = x2
5 x2 + y2 = a2
so |f(x)| ≤
4

XtraEdge for IIT-JEE 39 APRIL 2010


n n
Cr . 2 8,9,10. g (f(x)); z → z1
5. ∑
r =0
(−2) r
(r + 2) (r + 1) g(f(x)) = 1 ; x ∈ even integer
=0 ; otherwise
n
1
=
2(n + 1)(n + 2) ∑ (−2)
r =0
r n+2
Cr+2 (–2)2
Hence g(f(x)) is many one onto, periodic and even
n function
1
=
2(n + 1)(n + 2) ∑ (−2)
r =0
r + 2 n+2
Cr+2 Now, f(g(x)) = p ;
=1 ;
if x is perfect square
otherwise
1 Hence f(g(x)) = p has infinitely many roots.
= [(1 – 2)n + 2 –1 + 2(n + 2)]
2(n + 1)(n + 2)
1
= [(–1) n + 2 + 2n + 3]
2(n + 1)(n + 2)

=
1
if n is odd
Do you know
n+2
1
=
n+2
if n is even • The largest telescope in the world is currently
being constructed in northern Chile. The
1
telescope will utilize four - 26 ft. 8 in. (8.13
 x n +1 tan −1 x  1 x n +1 meters) mirrors which will gather as much light as
6. In = 

 n +1
 –

0
∫ 0 (n + 1)(1 + x 2 )
dx ...(1) a single 52 ft. 6 in. (16 meters) mirror.
• The Hubble Space Telescope weighs 12 tons
π 1  1 x n −1 ( x 2 + 1 − 1)  (10,896 kilograms), is 43 feet (13.1 meters) long,
=
4(n + 1)
– 
n + 1  ∫0 1+ x2
dx 
 and cost $2.1 billion to originally build.
π 1 1 1 1 x n −1
• The longest living cells in the body are brain cells
= –
4(n + 1) n + 1 ∫ 0
x n −1dx +
n +1 ∫ 1+ x
0 2
dx which can live an entire lifetime.

1
• The largest flying animal was the pterosaur which
π 1  x n 
 + 1
1 x n −1 lived 70 million years ago. This reptile had a wing
= –
4(n + 1) n + 1  n 
0 n +1 ∫ 1+ x
0 2
dx ...(2) span of 36-39 feet (11-11.9 meters) and weighed
190-250 pounds (86-113.5 kilograms).
1 x n +1 π • The Atlantic Giant Squid's eye can be as large as
from (1) ∫ 1+ x
0 2
dx =
4
– (n + 1) In
15.75 inches (40 centimeters) wide.
1 x n −1 π • Armadillos, opossums, and sloth's spend about
so ∫ 1+ x
0 2
dx =
4
– (n – 1)In–2
80% of their lives sleeping.
use it in (2) • The starfish species, Porcellanaster ivanovi, has been
π 1 1 π  found to live in water as deep as 24,881 feet
In =
4(n + 1)

(n + 1)n
+
n +1  4 − (n − 1)I n − 2  (7,584 meters).
 
π 1 • The tentacles of the giant Arctic jellyfish can reach
(n + 1) In + (n – 1) In – 2 = – 120 feet (36.6 meters) in length.
2 n
• The greatest tide change on earth occurs in the
Bay of Fundy. The difference between low tide
7. diff. partially w. r.t. x
and high tide can be as great as 54 ft. 6 in. (16.6
x+y 1 f ´(x ) meters).
f ´  . =
 3  3 3
• The highest temperature produced in a laboratory
Let x=0 was 920,000,000 F (511,000,000 C) at the
f ´(y/3) = f ´ (0) Tokamak Fusion Test Reactor in Princeton, NJ,
so f ´(x) = f ´(0) USA.
hence f ´(x) is a constant so • The fastest computer in the world is the CRAY
f ´ (x) = 2 (as given) Y-MP C90 supercomputer. It has two gigabytes of
so f(x) = 2x + c; since f(0) = 2 central memory and 16 parallel central processor
so f(x) = 2x + 2 units.
Hence f(2) = 6

XtraEdge for IIT-JEE 40 APRIL 2010


MATHEMATICAL CHALLENGES
SOLUTION FOR THIS ISSUE (SET # 12)

∫ 0 e x dx = (e − x x n )0 + n ∫ 0 e x dx
a −x n a a − x n −1 n
1. In = 3. Sn = ∑ r . nCr = n . 2n–1
r =0
= – e–a an + nIn – 1
n n
In = –e–a an + n[–e–a an–1 + (n–1)In–2] so S = ∑ S1 = ∑ n.2 n −1
n =1 n =1
= –e–a[an + nan–1 +n(n – 1)an–2 + n(n – 1)(n – 2)an–3
S = 1 + 2.21 + 3.22 + 4.23 + .... + n . 2n–1
+ ..... + n(n – 1)..... 2a] + n I0 2S = 2 + 2.22 + 3.23 + ..... + (n – 1). 2n–1 + n.2n
(1 – 2)S = (1 + 2 + 22 + 23 + .... + 2n–1) – n . 2n
 
(
= n  − e−x ) a
0
 a n
− e −a  +
 n
a n −1
n −1
+ .... +
a2
2
+ a 
 =1.
2n −1
– n . 2n
 2 −1
 S = n . 2n – 2n + 1 = (n – 1) 2n + 1
 a2 a n 
In = n 1 − e − a 1 + a + + ..... +  Now A.M. ≥ G. M.
  2 n  n
C1 + 2.n C 2 + 3.n C 3 + .... + n.n C n
∞ ≥
Now, I = ∫ 0 e − x x n = Lt In = n 1 + 2 + 3 + ..... + n
a →∞ 1

2. Mid pt. M of BC = (0, –1)  1



( )( ) ( )
 n C . n C 2 . n C 3 ...... n C n  1+ 2 +...+ n
2 3 n 

n ( n +1)
  2
A(6, 5)  n.2 n −1 
 n (n + 1)  ≥ nC1 (nC2)2 ..... (nCn)n
 
C P  2 
60º  
(–2, 1) n +1
45º C2
 2n 
M so C1 . ( C2) ..... ( Cn) ≤ 
n n 2 
n n
 n +1
 
B(2, –3) 4. Let P be a point represented by 1.
−3 − 1 so as |z1 – 1| = |z2 – 1| = |z3 – 1| so P is the
slope of BC = = –1
2+2 z + z 2 + z3
circumcentre of ABC. Its centroid is 1
so slope of altitude of ∆PBC is = 1. 3
length BC = 16 + 16 = 4 2 If the ∆ABC is equilateral then circumcentre = centroid
z + z 2 + z3
Now altitude PM = 4 2 sin 60º so 1= 1
3
3 so z1 + z2 + z3 = 3
= 4 2 . = 2 6
2 Now if z1 + z2 + z3 = 3 then centroid of ∆ABC is 1
eqn. of PM line is which is point P and P is already the circumcentre of
x − 0 y +1 ∆ABC. So now if they are same then ∆ABC is
= =r (as its slope is 1) equilateral.
1 1
2 2 5. let the centroid of ∆ABC be (h, k) then
3h = r cos α + r cos β + r
r r
x= & y= –1 3h α+β α −β
2 2 – 1 = 2 cos cos ...(1)
r 2 2
for req. pt. P take r = 2 6 . 3k
& = sin α + sin β
2 6 2 6  r
So pt. P ≡  , − 1 = (2 3 , 2 3 − 1) α −β α+β
 2 2  = 2 cos sin ...(2)
 
2 2

XtraEdge for IIT-JEE 41 APRIL 2010


α+β α +β α −β A∪B∪C∪D
line BC, x cos + y sin = r cos
2 2 2 =A+B+C+D–A∩B–A∩C–A∩D
Let point D be (a, b) – B ∩C – B ∩D – C ∩D + A ∩B ∩C
+ A ∩C∩D + B ∩C∩D + A ∩B∩D
–A∩B∩C∩D
C(α)
P(A ∪ B ∪ C ∪ D)
D
B(β) 1
= [ 1003 + 668 + 401 + 286 – 334 – 200
2006
A(r, 0) – 143 – 133 – 95 – 57 + 66 + 28 + 19 + 47 – 9]
1547
=
2006
α +β α+β α −β
then a cos + b sin = r cos 7. Line BC
2 2 2 A
....(3) a
α −β
Multiply (3) by cos
2
α −β α+β α +β α −β
a cos cos + b sin cos H
2 2 2 2
2 α −β
= r cos
2 B C
D
use (1) & (2) b = –i + 3j c = 2i + 5j
a  3h  3k α −β →
 −1 + b = r cos2 ...(4) r = – î + 3 ĵ + t (3î + 2 ĵ) ...(1)
2 r  2 r 2
any pt. D on it = (3t – 1) i + (3 + 2t) j
square & add (1) & (2)
2 As HD ⊥ BC,
α −β  3h  9k 2 so ((3t – 1 – 1) i + (3 + 2t – 2) j). (3i + 2j) = 0
4 cos2 =  − 1 + 2 ...(5)
2  r  r 3(3t – 2) + 2(2t + 1) = 0
from (4) & (5) 4
2
13 t – 4 = 0 ⇒ t=
1  3h  9k 2 a  3h  b3k 13
 − 1 + =  −1 +
4 r  4r 2
2 r  r  2r 2 i 47 j
so point D = – +
so req. locus is 13 13
(3x – r)2 + 9y2 = 2a(3x – r) + 6.b.y →  i 47 j 
9x2 + 9y2 – 6rx + r2 = 6ax – 2ar + 6.b.y Now line HD ⇒ r = i + 2j + s´  − + − i − 2 j
 13 13 
2 2 r2 2ar = i + 2j + s(–14i + 21j)
x2 + y2 – (r + a)x – by + + =0
3 3 9 9 →
2 2 ⇒ r = i + 2j + λ(–2i + 3j)
 1   1  Any point A on it = (1 – 2λ)i + (2 + 3λ)j
 x − 3 (r + a ) +  y − 3 b
    Now as AC ⊥ BH
(r + a ) 2 − r 2 − 2ar + b 2 a 2 + b2 so [ (1 – 2λ – 2)i + (2 + 3λ – 5)j] . [2i – j] = 0
= =
9 9 2 (–1 – 2λ) – (3λ – 3) = 0
1
a 2 + b2 ⇒ –7λ = 2 – 3 ⇒ λ =
It is a circle and radius is 7
3
Since point D is interior of circle so a2 + b2 < r2, 5i 17 j
so pt. A = +
so radius of this circle is less than r/3. 7 7

8. If A is the area of the triangle with sides a, b, and c,


6. Prime factors dividing 10 are 2, 3, 5, 7. As required
then A2 = s(s – a) (s – b) (s – c);
the number chosen should not be divisible by 2 or 3 where 2s = a + b + c
or 5 or 7. Define, the events as using AM – GM inequality for s – a, s – b, s – c, we
A : divisible by 2 have
B : divisible by 3 3
C : divisible by 5  (s − a ) + (s − b) + (s − c) 
A2 ≤ s  
D : divisible by 7  3 

XtraEdge for IIT-JEE 42 APRIL 2010


 3s − 2s 
3
s4 at t = 60 min.
A2 ≤ s   = 3   299  60 
 3  3 x = 100 1 −    kg
s2   300  
A≤
3 3
p2 10. Let P (3 cos θ, 3 sin θ)
Let 2s = p, then A ≤
12 3 line BC : y = – 3 ; line AC :
A
p2
Amax = ,
12 3
As condition of equality holds iff
s – a = s – b = s – c which happen if a = b = c
so Amax = ; for a = b = c
Now again p ≥ 12 3A P

pmin. = 12 3A ;
and again equality holds if a = b = c. B C
x y
9. Let the amount of salt dissolved at any time t is x kg. cos θ + sin θ = 1
3 3
x
So concentration is
300  3(1 + sin θ) 
pt. C  , − 3
dx 1 x   100 − x   cos θ 
so = k −  = k 
dt  3 300   300  pt. A (0, 3 cosec θ )
dx k 1 3(1 + sin θ)
= dt Area A = .2. . ( 3 cosec θ + 3)
100 − x 300 2 cos θ

– ln (100 – x) =
k
t+C 3 3 (1 + sin θ) 2 6 3 (1 + sin θ) 2
= =
300 sin θ cos θ sin 2θ
at t = 0, x = 0 so C = –ln 100 dA 6 3 (2(1 + sin θ) sin 2θ cos θ − 2(1 + sin θ) 2 cos 2θ)
kt 100 =
so = ln 100 – ln (100 – x) = ln dθ sin 2 2θ
300 100 − x
at t = 1 min., x = 1/3 12 3 (1 + sin θ)(sin 2θ cos θ − cos 2θ − sin θ cos 2θ)
=
k 100 sin 2 2θ
so = ln
300 1 12 3 (1 + sin θ)(sin(2θ − θ) − cos 2θ)
100 − =
3 sin 2 2θ
k 100 12 3 (1 + sin θ)(sin θ − 1 + 2 sin 2 θ)
so = ln =
300 299 sin 2 2θ
3
12 3 (1 + sin θ)(2 sin θ − 1)(sin θ + 1)
k 300 =
so = ln sin 2 2θ
300 299
12 3 (1 + sin θ) 2 (2 sin θ − 1)
300 =
so k = 300 ln sin 2 2θ
299
π
300 100 Amin at θ =
so ln . t = ln 6
299 100 − x
2
t  1
 300  100 6 3 1 + 
so   =  2 9
 299  100 − x so Amin = = 12 .
t 3 4
 299  2
so 100 – x = 100 .  
 300  = 27sq. units.
  299  t  = 0027 Ans.
so x = 100 1 −   
  300  

XtraEdge for IIT-JEE 43 APRIL 2010


Students' Forum
Expert’s Solution for Question asked by IIT-JEE Aspirants
MATHS
1. Let f : R → R and f(x) = g(x) + h(x), where g(x) is a 85
polynomial and h(x) is a continuous and Now AC0 = r
5
differentiable bounded function on both sides, then
f(x) is onto if g(x) of odd degree and f(x) is into if AC 0 .(AB × C 0 D) 5r
and shortest distance = = unit.
g(x) is of even degree. Then check whether f(x) is | AB × C 0 D | 3
one one, many one, onto or into.
(i) f(x) = a1x + a3x3 + a5x5 + .... + a2n + 1x2n + 1 – cot–1x
where 0 < a1 < a2 < a3 ....... < a2n + 1 3. Let f(x) is a polynomial one-one function such that
f(x).f(y) + 2 = f(x) + f(y) + f(xy) ∀ x, y ∈ R–{0},
x ( x 4 + 1)( x + 1) + x 4 + 2
(ii) f(x) = x x
x2 + x +1 f(1) ≠ 1, f´(1) = 3. Let g(x) = (f(x) + 3) –
4 0 ∫
f ( x ) dx,
Sol. (i) f(x) = odd degree polynomial + bounded function
then prove that g(x) is an identity for all given
cot–1x also f´(x) > 0
x ∈ R –{0}.
⇒ y = f(x) will be one one and onto
Sol. putting x = y = 1 in given condition we get
( x 4 + 1)( x 2 + x + 1) 1 f(1)2 + 2 = f(1) + f(1) + f(1)
(ii) f(x) = = x4 + 1 + 2
x2 + x +1 x + x +1 ⇒ {f(1)}2 – 3{f(1)} + 2 = 0
= even degree polynomial ⇒ f(1) = 1 or 2 ⇒ f(1) = 2
 4 Now put y = 1/x,
+ bounded function ∈  0, 
 3 1 1
f(x) . f   + 2 = f(x) + f   + f(1)
also f´(x) = 0 has at least one root real which is not x
  x
repeated since f´(x) is a polynomial of degree 7. ⇒ f(x) = 1 ± xn
⇒ f(x) = 0 has at least one point of extrema. According to given conditions, f(x) = 1 + x3
⇒ many one & Into x x
Now, g(x) = [1 + x3 + 3] –
4 0 ∫(1 + x3) dx = 0
2. A rectangle ABCD of dimensions r and 2r is folded
⇒ g(x) = 0 for ∀ x ∈ R –{0}
along the diagonal BD such that planes ABD and
CBD are perpendicular to each other. Let the position 4. Let three normals are drawn to the parabola y2 = 4ax
of the vertex A remains unchanged and C0 is the new at three points P, Q and R, from a fixed point A. Two
position of C, then find the distance of C0 from A
circles S1 and S2 are drawn on AP and AQ as
and shortest distance between the edges AB & C0D. diameter. If slope of the common chord of the circles
(2r, r) S1 and S2 be m1 and the slope of the tangent to the
(0, r) D C parabola at the point R be m2, then prove that
m1 . m2 = 2.
Sol. Let A(h, k) be a fixed point
N at3 + (2a – h)t – k = 0
{Q three normals are drawn from (h, k)}
A Let feet of normals P,Q and R are three points with
(0, 0) B(2r, 0) parameters t1, t2 and t3.
Sol. Let the rectangle ABCD lies on the plane xy. After Common chord of S1 and S2 = S1 – S2 = (t1 + t2)x +
folding the rectangle along the BD co-ordinates of 2y – h(t1 + t2) – 2k = 0
points in 3-D are- Tangent to the parabola at R = t3y = x + at32
A : (0, 0, 0), B : (2r, 0, 0), C : (2r, r, 0), D(0, r, 0)
t +t  1 1
 2r r   2r r 2 r  m1 . m2 = –  1 2  . =
and N :  , ,0  and C0  , ,  ,  2  3 t 2
 5 5   5 5 5 (Q t1 + t2 + t3 = 0 for co-normal points).

XtraEdge for IIT-JEE 44 APRIL 2010


5. Let z1, z2 and z3 are unimodular complex numbers
then find the greatest value of |z1 – z2|2 + |z2 – z3|2 +
|z3 – z1|2.
Sol. |z1 – z2|2 + |z2 – z3|2 + |z3 – z1|2
Science Facts
= 2[|z1|2 + |z2|2 + |z3|2] – [z1 z 2 + z1 z2 + z1 z3 Skin Deep Storage
+ z 3 z1 + z2 z 3 + z 2 z3]
Chip implants that keep
= 6 – [z1 z 2 + z2 z1 + z1 z 3 + z3 z1 +z2 z 3 + z3 z 2 ] track of personal
...(1) information seem like a
Now |z1 + z2 + z3|2 ≥ 0
novelty but do they have a
⇒ z1 z 2 + z2 z 3 + z2 z1 + z3 z 2 + z1 z 3 + z1 z3 ≥ –3
more useful future?
...(2)
From (1) & (2) These days, some people are following their
maxm value of |z1 – z2|2 + |z2 – z3|2 + |z3 – z1|2 pets and getting tagged. Radio frequency identification
= 6 – (–3) = 9 (RFID) chips are the size of a grain of rice and can be
loaded up with personal information like passwords
6. Consider following two infinite series in real θ and r and implanted under the skin. Instead of having to
remember a login code, an RFID reader can be set up
r 2 cos 2 2θ r 3 cos 3θ
C = 1 + r cos θ + + + .... to automatically detect it and grant you access to a
2! 3! range of things from your computer to your front
r 2 sin 2θ r 3 sin 3θ door. It seems like it could be useful to people with
S = r sin θ + + + ..... exceptionally poor memories, but right now these
2! 3!
chips are being snapped up by technology geeks like
If a remains constant and r varies the prove that Amal Graafstra. The 29-year-old businessman from
dC dS Vancouver, Canada, is one of the first people to have
(i) C +S = (C2 + S2) cos θ
dr dr an RFID implant and so far is happy with the results. "I
2 2 just don't want to be without access to the things that
 dC   dS  2 2 I need to get access to. In his chip, he has stored a
(ii)   +   =C +S
 dr   dr  unique identification number which can be used to log
Sol. We have him into various electronic devices. It didn't cost him
iθ an arm and a leg either: he got the whole set-up on
C + iS = e re ...(1) the internet for about $50 (£30), including the $2 cost
re −iθ of the chip itself.
C – iS = e ...(2)
Now, The procedure to implant the chip is quite
iθ 2 2 simple and painless. Amal's chip was implanted under
C2 + S2 = e re = e r cos θ .e i r sin θ = e2r cos θ ...(3) the skin of his left hand while he was under a local
anesthetic. It is possible to inject the chips using a
Differentiating (1) w.r.t.r, we get large enough needle, but in Amal's case the chip was
dC dS iθ
inserted by simply cutting through his skin with a
+i = eiθ . e r.e ....(4)
dr dr scalpel. Other than complaining of sensitivity in the
2 2 area of the implant, Amal said that it doesn't hurt and
 dC   dS  iθ r.e iθ
∴   +   = e .e = e2r cos θ he expects that eventually the chip will be completely
 dr   dr  unobtrusive.
= C2 + S2 (Form 3) A hand implanted with an
multiply (2) and (4) RFID chip and the chip reader.
 dC dS  iθ −iθ The chip is made of silicon and is
 + i  (C – iS) = eiθ . e r.e . e r.e digitally encoded with information..
 dr dr  A RFID reader, which is installed in a computer or an
iθ − iθ
= eiθ [e r ( e + e ) ] electronic device like a reader by a front door, emits a
radio signal of a particular frequency, just like radio
= eiθ . e2r cosθ
stations each broadcast on their own frequency. The
Now equating real parts in both sides
chip acts passively when it is within 3 inches of the
dC dS reader: the right incoming radio signal induces just
C +S = (C2 + S2) cosθ
dr dr enough energy in the antenna of the chip for a circuit
Hence proved. in the chip to power up and produce a response. The
reader can then access the information on the chip and
pass it on to the computer or device that requires it.

XtraEdge for IIT-JEE 45 APRIL 2010


MATHS
CALCULUS Mathematics Fundamentals

Function, Limits, Continuity & Differentiability : If f(x) and g(x) both are not differentiable at x = a
If the domain of the function is in one quadrant then then the product function f(x).g(x) can still be
the trigonometrical functions are always one-one. differentiable at x = a.
If trigonometrical function changes its sign in two If f(x) is differentiable at x = a and g(x) is not
consecutive quadrants then it is one-one but if it does differentiable at x = a then the sum function f(x) +
not change the sign then it is many one. g(x) is also not differentiable at x = a.
In three consecutive quadrants tigonometrical If f(x) and g(x) both are not differentiable at x = a,
functions are always many one. then the sum function may be a differentiable
function.
Any continuous function f(x), which has at least one
local maximum, is many-one. Differentiation and Applications of Derivatives :
Any polynomial function f : R → R is onto if degree dy d d
is (y) in which is simply a symbol of
of f is odd and into if degree of f is even. dx dx dx
An into function can be made onto by redefining the operation and not 'd' divided by dx.
cordomain as the range of f is even. If f´(x0) = ∞, the function is said to have an infinite
An into function can be made onto by redefining the derivative at the point x0. In this case the line tangent
codomain as the range of the original function. to the curve of y = f(x) at the point x0 is
perpendicular to the x-axis.
1
If f(x) is periodic with period T then is also Of all rectangles of a given perimeter, the square has
f (x)
the largest area.
periodic with same period T.
All rectangles of a given area, the squares has the
If f(x) is periodic with period T, f ( x ) is also least perimeter.
periodic with same period T. A cone of maximum volume that can be inscribed in
Period of x – [x] is 1. Period of algebraic functions 4r
a sphere of a given radius r, is of height .
x , x2, x3 + 5 etc. does't exist. 3
If lim f ( x ) does not exist, then we can not remove A right circular cylinder of maximum volume that
x →a can be inscribed in a square of radius r, is of height
this discontinuity. So this become a non-removable 2r
discontinuity or essential discontinuity. .
3
If f is continuous at x = c and g is discontinuous at x
= c, then If at any point P(x1, y1) on the curve y = f(x), the
tangent makes equal angle with the axes, then at the
(a) f + g and f – g are discontinuous π 3π dy
(b) f.g may be continuous point P, ψ = or . Hence, at P tan ψ = = ±1
4 4 dx
If f and g are discontinuous at x = c, then f + g, f – g
Indefinite Integral :
and fg may still be continuous.
If F1(x) and F2(x) are two antiderivatives of a
Point functions (domain and range consists one value
function f(x) on an interval [a, b], then the difference
only) is not a continuous function.
between them is a constant.
If a function is differentiable at a point, then it is
The signum function has an antiderivative on any
continuous also at that point.
interval which does not contain the point x = 0, and
i.e., Differentiability ⇒ Continuity, but the converse does not possess an anti=derivative on any interval
need not be true. which contains the point.
If a function 'f' is not differentiable but is continuous The antiderivative of every odd function is an even
at x = a, it geometrically implies a sharp corner or function and vice-versa.
kink at x = a.

XtraEdge for IIT-JEE 46 APRIL 2010


x n e ax n sin n x
If In = ∫ x n .e ax dx, then In =
a
– In–1
a
Reduction formulae for I(n,m) = ∫ cos m
x
dx is

If In = ∫ (log x) dx , then I n = x log x – x I(n,m) =


1
.
sin n −1 x
m − 1 cos m −1

(n − 1)
(m – 1)
.I(n–2, m – 2)
x
1
If In = ∫ log x dx, then Definite Integral and Area Under Curves :
1 b

In = log(logx) + logx +
(log x ) 2
+
(log x ) 3
+ ...
The number f(c) =
(b − a ) a ∫
f ( x ) dx is called the
2.(2!) 3.(3!) mean value of the function f(x) on the interval [a, b].
If m and M are the smallest and greatest values of a
∫ (log x)
n
If In = dx ; then In = x(logx)n – n.In–1
function f(x) on an interval [a, b], then m(b – a) ≤
Successive integration by parts can be performed b

when one of the functions is xn (n is positive integer) ∫a


f ( x ) dx ≤ M(b – a).
which will be successively differentiated and the
If f2(x) and g2(x) are integrable on [a, b], then
other is either of the following sin ax, cos ax, e–ax, (x
1/ 2 1/ 2
+a)m which will be successively integrated. b  b   b 
Chain rule : ∫ a
f ( x ) g ( x ) dx ≤ 
 ∫ a
f 2 ( x ) dx 


 ∫
a
g 2 ( x ) dx 

∫ u.v dx = uv 1 – u´v2 + u"v3 – u"'v4 + .... Change of variables : If the function f(x) is
continuous on [a, b] and the function x = φ(t) is
+ (–1)n – 1un–1vn + (–1)n u n .v dx ∫ continuously differentiable on the interval [t1, t2] and
a = φ(t1), b = φ(t2), then
n th
where u stands for n differential coefficient of u b t2
and vn stands for nth integral of v. ∫a
f ( x ) dx = ∫t1
f (φ( t )φ´(t ) dt
ax
xe
∫ xe Let a function f(x, α) be continuous for a ≤ x ≤ b and
ax
sin(bx + c)dx = [a sin(bx + c) – b
a 2 + b2 c ≤ α ≤ d. Then for any α ∈[c, d], if I(α) =
e ax b b
cos(bx + c)] –
(a + b ) 2 2 2
[(a2 – b2)sin (bx + c) – 2ab
∫a
f ( x, α) dx , then I´(α) = ∫ a
f ´(x, α) dx , where

cos (bx + c)] + k I´(α) is the derivative of I(α) w.r.t. α and f´(x, α) is
the derivative of f(x, α) w.r.t α, keeping x constant.
x.e ax
∫ xe ax cos(bx + c)dx =
a 2 + b2
[a cos(bx + c) – b

b
f ´(x ) dx = (b – a)
1
∫ f [(b − a)t + a] dt
a 0
ax
e 2 2
sin(bx + c)] – [(a – b )cos (bx + c) – 2ab b f ( x )dx 1
(a 2 + b 2 ) 2 ∫a f ( x ) + f (a + b − x )
= (b – a)
2
sin (bx + c)] + k
The area of the region bounded by y2 = 4ax, x2 = 4by
∫ xe ax sin(bx + c)dx
is
16ab
sq. unit.
3
ax
= [(loga)sin(bx + c) – b cos(bx + c)] + k The area of the region bounded by y2 = 4ax and y =
(log a ) 2 + b 2
8a 2
mx is sq. unit.
∫ xe
ax
cos(bx + c)dx 3m 3
The area of the region bounded by y2 = 4ax and its
ax
= [(loga)cos(bx + c) + b sin(bx + c)] + k 8a 2
(log a ) 2 + b 2 latus-rectum is sq. unit.
3
a cos x + b sin x The area of the region bounded by one arch of sin(ax)
∫ c cos x + d sin x dx or cos (ax) and x-axis is 2/sq. unit.
ac + bd ad − bc Area of the ellipse (x2/a2) + (y2/b2) = 1 is πab sq. unit.
= x+ log |c cos x + d sinx| + k.
c2 + d2 c2 + d 2 Area of region bounded by the curve y = sin x, x-axis
and the line x = 0 and x = 2π is 4 sq. unit.

XtraEdge for IIT-JEE 47 APRIL 2010


MATHS

ALGEBRA Mathematics Fundamentals

Complex Numbers : (z1 – z2)2 + (z2 – z3)2 + (z3 – z1)2 = 0


|z| ≥ |Re(z)| ≥ Re(z) and |z| ≥ |1m(z) | ≥ 1m(z) or z12 + z 22 + z 32 = z1z2 + z2z3 + z3z1
z 1 1 1
is always a unimodular complex number if z ≠ 0 or + + =0
|z| z1 − z 2 z2 − z3 z 3 − z1
|Re(z) | + |1m(z) | ≤ 2 |z| If z1, z2 z3 are the vertices of an isosceles triangle,
1 right angled at z2 then z12 + z 22 + z 32 = 2z2(z1 + z2)
If z + = a, the greatest and least values of |z| are
z If z1, z2, z3 are the vertices of a right-angled isosceles
2 2 triangle, then (z1 – z2)2 = 2(z1 – z3)(z3 – z2).
a+ a +4 −a + a +4
respectively and If z1, z2, z3 be the affixes of the vertices A, B, C
2 2 respectively of a triangle ABC, then its orthocentre is
|z1 + z12 − z 22 | + |z2 – z12 − z 22 | = |z1 + z2| + |z1 – z2| a (sec A)z1 + b(sec B)z 2 + (c sec C)z 3
If z1 = z2 ⇔ |z1| = |z2| or arg z1 = arg z2 a sec A + b sec B + c sec C
|z1 + z2| = |z1 – z2| ⇔ arg (z1) – arg(z2) = π/2. For any a, b ∈ R
If |z1| ≤ 1, |z2| ≤ 1 then (i) a + ib + a − ib = 2{ a 2 + b 2 + a}
(i) |z1 + z2|2 ≤ (|z1| – |z2|)2 + (arg (z1) – arg (z2))2
(ii) |z1 + z2|2 ≥ (|z1| + |z2|)2 – (arg (z1) – arg(z2))2 (ii) a + ib – a − ib = 2{ a 2 + b 2 − a}
|z1 + z2|2 = |z1|2 + |z2|2 + 2||z2| cos(θ1 – θ2). The sum and product of two complex numbers are
|z1 – z2|2 = |z1|2 + |z2|2 – 2|z1||z2| cos(θ1 – θ2). real simultaneously if and only if they are conjugate
If z1 and z2 are two complex numbers then to each other.
z1 r If ω and ω2 are the complex cube roots of unity, then
|z1 z2| = r1r2; arg(z1z2) = θ1 + θ2 and = 1 , (i) (aω + bω2)(aω2 + bω) = a2 + b2 – ab
z2 r2
(ii) (a + b (aω + bω2)(aω2 + b2ω) = a3 + b3
z  (iii) (a + bω + cω2)(a + bω2 + cω)
arg  1  = θ1 – θ2 and where |z1| = r1, |z2| = r2,
 z2  = a2 + b2 + c2 – ab – bc – ca
arg(z1) = θ1 and arg(z2) = θ2. (iv) (a + b + c) (a + bω + cω2) (a + bω2 + cω)
The area of the triangle whose vertices are z, iz and = a3 + b3 + c3 – 3abc
1 If three points z1, z2, z3 connected by relation
z + iz is |z|2.
2 az1 + bz2 + cz3 = 0 where a + b + c = 0, then the three
The area of the triangle with vertices z, wz and points are collinear.
3 2 If three complex numbers are in A.P. then they lie on
z + wz is |z |. a straight line in the complex plane.
4
Progression :
If z1, z2, z3 be the vertices of an equilateral triangle
and z0 be the circumcentre, then If Tk and Tp of any A.P. are given, then formula for
T − Tk Tp − Tk
z12 + z 22 + z 32 = 3 z 02 . obtaining Tn is n = .
n−k p−k
If z1, z2, z3 ....... zn be the vertices of a regular
polygon of n sides and z0 be its centroid, then If pTp = qTq of an A.P., then Tp + q = 0.
If pth term of an A.P. is q and the qth term is p, then
z12 + z 22 + .......+ z 2n = n z 02 . Tp+q = 0 and Tn = p + q – n.
If z1, z2, z3 be the vertices of a triangle, then the If the pth term of an A.P. is 1/q and the qth term is 1/p,
triangle is equilateral if then its pqth term is 1.

XtraEdge for IIT-JEE 48 APRIL 2010


The common difference of an A.P. is given by Permutations and Combinations :
d = S2 – 2S1 where S2 is the sum of first two terms n
C0 = nCn = 1, nC1 = n
and S1 is the sum of first term or the first term. n
Cr + nCr–1 = n+1Cr
If sum of n terms Sn is given then general term n
Cx = nCy ⇔ x = y or x + y = n
Tn = Sn – Sn–1, where Sn–1 is sum of (n – 1) terms of
A.P. n. n-1Cr–1 = (n – r + 1)nCr–1
If for an A.P. sum of p term is q and sum of q terms If n is even then the greatest value of nCr is nCn/2.
n
is p, then sum of (p + q) terms is {–(p + q)}. C n +1
If n is odd then the greatest value of nCr is or
If for an A.P., sum of p term is equal to sum of q 2
terms, then sum of (p + q) terms is zero. n
C n −1
If the pth term of an A.P. is 1/q and qth term is 1/p, .
2
1
then sum of pq terms is given by Spq = (pq + 1). n n
2 Cr = .n–1Cr–1.
r
Sum of n A.M.'s between a and b is equal to n times
the single A.M. between a and b. Number of selection of zero or more things out of n
different things is, nC0 + nC1 + bC2 + ... + nCn = 2n.
a+b n
C0 + nC2 + nC4 + .... = nC1 + nC3 + nC5 + .... = 2n–1.
i.e. A1 + A2 + A3 + ...... + An = n  .
 2  Gap method : Suppose 5 moles A, B, C, D, E are
If Tk and Tp of any G.P. are given, then formula for arranged in a row as × A × B × C × D × E ×. There
1 1 will be six gaps between these five. Four in between
 T  n −k  Tp  p − k and two at either end. Now if three females P, Q, R
obtaining Tn is  n  =  
are to be arranged so that no two are together we
 Tk   Tk 
shall use gap method i.e., arrange them in between
Product of n G.M.'s between a and b is equal to nth these 6 gaps. Hence the answer will be 6P3.
power of single geometric mean between a and b
Together : Suppose we have to arrange 5 persons in
i.e. G1G2G3 .... Gn = ( ab )n. a row which can be done in 5! = 120 ways. But if two
The product of n geometric means between a and 1/a particular persons are to be together always, then we
is 1. tie these two particular persons with a string. Thus
1
we have 5 – 2 + 1 (1 corresponding to these two
 b  n +1 together) = 3 + 1 = 4 units, which can be arranged in
If n G.M.'s inserted between a and b then r =   4! ways. Now we loosen the string and these two
a particular can be arranged in 2! ways. Thus total
Quadratic Equations and Inequations : arrangements = 24 × 2 = 48.
An equation of degree n has n roots, real or If we are given n different digits (a, a2, a3 ..... an) then
imaginary. sum of the digits in the unit place of all numbers
If f(α) = 0 and f´(α) = 0, then α is a repeated root of formed without repetition is (n – 1)!(a1 + a2 + a3 + ....
the quadratic equation f(x) = 0 and f(x) = a(x – α)2. + an). Sum of the total numbers in this case can be
obtatined by applying the formula (n – 1)!(a1 + a2 +
In fact α = –b/2a.
a3 + ..... + an). (1111 ......... n times).
If α is repeated common root of two quadratic
Binomial Theorem & Mathematical Induction :
equations f(x) = 0 and φ(x) = 0, then α is also a
The number of terms in the expansion of (x + y)n are
common root of the equations f´(x) = 0 and φ´(x) = 0.
(n + 1).
In the equation ax2 + bx + = 0 [a, b, c ∈R], if If the coefficients of pth, qth terms in the expansion of
a + b + c = 0 then the roots are 1, c/a and if a – b + c (1 – x)n are equal, then p + q = n + 2.
= 0, then the roots are –1 and – c/a. For finding the greatest term in the expansion of
If one root of the quadratic equation ax2 + bx + c = 0 (x + y)n. we rewrite the expansion in this form
is equal to the nth power of the other, then n
 y
(x + y)n = xn 1 +  . Greatest term in (x + y)n = xn.
( ) ( )
1 1
ac n n +1 + a nc n +1 + b = 0.  x
n
If one root is k times the other root of the quadratic  y
Greatest term in 1 +  .
(k + 1) 2 b 2  x
equation ax2 + bx + c = 0, then = .
k ac There are infinite number of terms in the expansion
If an equation has only one change of sign, it has one of (1 +x)n, when n is a negative integer or a fraction.
+ve root and no more. The number of term in the expansion of
(x1 + x2 + .... + x2)n = n+r-1Cr–1.

XtraEdge for IIT-JEE 49 APRIL 2010


Learning the MIghTy way
When you hear... you forget. When you see... you remember. When you do... you understand. – Confucius

This sounds very much the way we learnt While focusing on innovation, MIT has taken
things in our childhood. Our parents did not tell us care of students by getting them absorbed into the
how to walk, but made us walk. They did not show best industries, through quality industry projects and
us how to ride a bicycle, they made us ride it. And collaborations with universities and companies.
that’s how learning goes naturally: deriving ‘Practice School’ is one such concept, in sync with
inspiration by doing things, ‘Inspired Learning’. the idea of ‘Inspired learning’. This was introduced
at MIT in 2005, through which students are trained to
In this era of information overload,
effectively link the theory learnt in classrooms with
educational institutions hardly devote time and
the practice in the industry. This training helps
resources to make students follow this natural path of
reduce the cost for a company, on in-house training
learning. An exception to this is Manipal Institute of
of students who are their prospective employees. Due
Technology (MIT), where the ‘been there, done that’
to such efforts placement statistics have been moving
spirit has helped students make a mark worldwide.
up since 2005 reaching 95% in 2008. Even in the
While in other colleges, students were seeing recession hit 2008-09, 56 companies turned up
how a car is built, students at MIT designed and offering jobs to 948 students, with the best
manufactured a high performance car which made its remuneration reaching 10.75 lakhs per annum. MIT
way into the ‘Formula Student 2009’ (Student now stands 7th among the private Engineering
edition of Formula One), at Silverstone, UK. MIT colleges in India, and 22nd overall.
got 65th position among teams from 126 universities
A student, Gaurav Sinha (4th year, Electrical
of 23 countries. Several other feathers of innovation
and Electronics) quotes “MIT has provided me with
have been added to MIT’s achievement hat within
excellent opportunities to develop to my full
the last 2 years like: 1st position in “Train Blazer-
potential. The environment is conducive for our
2008”; 1st position in “GE Edison Innovation
growth and along with the state-of-art infrastructure
Challenge -2008”; 1st and 2nd position in “Schneider
facilities available helps bring out the best in each of
Electric Innovation Challenge” 2008; 1st prize for an
us. The platform and exposure that I got at MIT has
Innovative proposal on “Solar Power embedded
groomed my personality and prepared me to face any
Street Lamp” and many more.
challenge in future”. Already having produced strong
The foundation of this hub of thinking and personalities of industry like Rajeev Chandrasekhar
innovation was laid 52 years ago, and since then is (Chairman & CEO of Jupiter Capital; FICCI
being strengthened by constructive effort and hard President; Founder & former-CEO of BPL mobile),
work. The quality of technical education is evident Ravi Bapna (Executive Director of CITNE &
in the research activities going on in the campus: 6 Professor, Indian School of Business) , Amit Behl
patents filed in the current year, 9 grants received for (Director, Intel India), etc. the excellent
various activities and, several papers, books and infrastructure coupled with innovative learning and
awards added to its research portfolio. A major course content, holds promise for a much brighter
landmark is ‘Manipal University Technology future for the country.
Business Incubator’, which is being established with
– MIT, Manipal
a huge funding from the Department of Science and
Technology, Government of India.

XtraEdge for IIT-JEE 50 APRIL 2010


XtraEdge for IIT-JEE 51 APRIL 2010
MOCK TEST FOR IIT-JEE
PAPER - I
Time : 3 Hours Total Marks : 240

Instructions :
• This question paper contains 60 questions in Chemistry (20), Mathematics (20) & Physics (20).
• In section -I (8 Ques) of each paper +3 marks will be given for correct answer & –1 mark for wrong answer.
• In section -II (4 Ques) of each paper +4 marks will be given for correct answer –1 mark for wrong answer.
• In section -III contains 2 groups of questions (2 × 3 = 6 Ques.) of each paper +4 marks will be given for each
correct answer & –1 mark for wrong answer.
• In section -IV (2 Ques.) of each paper +8(2×4) marks will be given for correct answer & No Negative marking for
wrong answer.

CH3
CHEMISTRY
(D) CH = CH
SECTION – I
Straight Objective Type CH3

This section contains 8 multiple choice questions. Each 3. Identify product D in the following reaction
question has 4 choices (A), (B), (C) and (D), out of sequence:
which ONLY ONE is correct.
CH 3
1. A bis-aldol dimerization of 1-phenyl-1,2- |
K 2Cr2 O 7 ,H + SOCl 2
H3C– C — CH2CH2OH   → A  → B
propanedione (C6H5COCOCH3) gives which of the | H 2O, Heat
following ? CH 3
O (CH ) NH I. LiAlH , ether
O 32→ C   4  → D
C6H5 C6H5 II. H 2O
(A) (B)
C6H5 C6H5 CH 3
|
O
(A) H3C– C — CH2C ≡ N
O O |
C6H5
(C) (D) C6H5 CH 3
C6H5
C6H5 CH 3 N(CH 3 ) 2
O
| |
2. An alkene (A), C16H16 on ozonolysis gives only one (B) H3C– C — CH2 CHN (CH 3 ) 2
|
product B(C8H8O). Compound (B) on reaction with CH 3
NH2OH, H2SO4, ∆ gives N-methyl benzamide. The
CH 3 O
compound 'A', is – | ||
H (C) H3C – C — CH2 C N(CH3)2
CH3 |
(A) C=C
H CH3 CH 3
CH 3
(B) C=C |
CH3 CH3 (D) H3C– C — CH2CH2N(CH3)2
|
CH 3
(C) CH2–CH = CH–CH2

XtraEdge for IIT-JEE 52 APRIL 2010


4. Consider the following reactions : (A) PT = constant is correct, because volume remain
Reaction I : same in both the laws
O O O O (B) PT = constant is incorrect, because volume
+ remain same at the constant temperature and at
CH CH OH
H 3C O CF3 3 
r1
→
2
H 3C O F3C OH the constant pressure
(C) PT = constant is correct, because volume at
Reaction II : constant temperaute and volume at constant
O O pressure are not same
CH CH OH (D) PT = constant is incorrect, because volume at
H 3C O CF3 3  →
2
r2 constant temperature and volume at constant
O O pressure for the same amount of gas are different
+
H 3C OH F3C O 7. Which of the following statements is correct ?
Which of the following is a correct comparison of the (A) (n – 1) d subshell has lower energy than ns
rate of reactions I and II ? subshell
(A) rI = rII (B) r1 > rII (B) (n – 1) d subshell has higher energy than ns
(C) r1 < rII (D) None subshell
(C) (n + 1) d subshell has lower energy than nf
5. A container containing HCl(g) is fitted with a funnel
and a long capillary tube as shown below. Capillary subshell
is immersed in the water. When few drops of water is (D) nf subshell has lower energy than (n + 2) s
introduced into subshell
the container through the funnel then –
A 8. (A), (B) and (C) are elements in the third short
period. Oxide of (A) is ionic , that of (B) is
HCl(g) amphoteric and of (C) a giant molecule. (A), (B) and
(C) have atomic numbers in the order of -
(A) (A) < (B) < (C) (B) (C) < (B) < (A)
(C) (A) < (C) < (B) (D) (B) < (A) < (C)

SECTION – II
Multiple Correct Answers Type
(A) there is a pressure drop in the container and liquid This section contains 4 multiple correct answer(s) type
level in the capillary rises questions. Each question has 4 choices (A), (B), (C) and
(B) there is a pressure drop in the container because (D), out of which ONE OR MORE is/are correct.
HCl(g) effuse out more rapidly than the air effuse
H
in ⊕
(C) HCl(g) undergoes spontaneous dissociation to 9. + Ph3C⊕BF4– → BF4– + Ph3 CH
H2(g) & Cl2(g), hence number of moles decreases, H
resulting pressure drop inside the container. H
Water level in the capillary rises
which of the following statements is/are correct -
(D) HCl(g) spontaneously mixed with water through
the capillary therefore, water level in the capillary (A) the cation in reactant side is approximately
remain same 1011 times more stable than product side
(B) cation in reaction side is non planer
6. As per Boyle's law V ∝ 1/P at constant temperature, (C) it is acid base reaction
As per charles law V ∝ T at constant pressure.
Therefore, by combining, one concluded that T ∝ 1/P (D) reaction must be exothermic
hence, PT = constant

XtraEdge for IIT-JEE 53 APRIL 2010


10. Which of the following is/are correct ? Paragraph # 1 (Ques. 13 to 15)
(A) The efficiency of a solid catalyst depends A pleasant smelling optically active compound,
upon its surface area monoester 'F' has molecular weight 186. It doesn't
(B) Catalyst operates by providing alternate path react with Br2 in CCl4. Hydrolysis of 'F' gives two
for the reaction that involves lower activation optically active compounds 'G', which is soluble in
energy NaOH and 'H'. H gives a positive iodoform test, but
(C) Catalyst lowers the activation energy of on warming with conc. H2SO4 gives I with no
forward reaction only without affecting the disastereomers. When the Ag+ salt of 'G' is reacted
activation energy of backward reaction with Br2, racemic 'J' is formed. Optically active J is
(D) Catalyst does not affect the overall enthalpy formed when 'H' is treated with tosyl chloride (TsCl),
change of the reaction and then with NaBr.

11. For the three elements P, Q & R, ionization 13. The pleasant smelling optically active compound, F
enthalpy (IE) and electron gain enthalpies (∆eg H) is -
O
are given in the following table - ||
Element IE in kJ/mol ∆eg H in kJ/mol (A) (CH3)2CH– CHC – O– CH – CH(CH3)2
| |
P 1680 –340 CH 3 CH 3
Q 1100 –120 O
R 500 –20 ||
(B) (CH3)3C–CH2 C –O– CH – CH(CH3)2
|
(A) P is the highest electronegative element CH 3
among P, Q and R O
(B) R is the least electronegative among P, Q & R ||
(C) CH3CH2CH2 CH – C –O– CH – CH2CH2CH3
(C) Electro negativity of P is approximately equal to 4 | |
(D) R may be chlorine CH 3 CH 3
O
12. Which of the following is/are correct ? ||
(D) CH3CH2 CH – CH2– COCH 2 – CHCH 2 –CH3
 dH  | |
(A) For the incompressible liquid   is CH 3 CH 3
 dP  T
approximately equal to volume of liquid 14. How would be the structure of F if I exists as
diastereomers ?
 dH  O
(B) For ideal gas   is equal to zero
 dP  T ||
(A) (CH3)2CH CHCOCHCH (CH 3 ) 2
 dE  | |
(C) For real gas if   = 0 then not CH 3 CH 3
 dV  T
O
 dH  ||
necessarily   is equal to zero (B) (CH3)3CCH2 C O CHCH (CH 3 ) 2
 dP  T |
CH 3
(D) None of these
O
||
(C) CH3CH2CH2 CHCOCHCH 2 CH 2 CH 3
SECTION – III | |
Comprehension Type CH 3 CH 3
O
This section contains 2 groups of questions. Each group ||
has 3 multiple choice questions based on a paragraph. (D) CH3CH2 CHCH 2 C OCH2CHCH 2 CH 3
Each question has 4 choices (A), (B), (C) and (D) for its | |
CH3 CH 3
answer, out of which ONLY ONE is correct.

XtraEdge for IIT-JEE 54 APRIL 2010


15. What would be the structure of F if H gives negative If the correct matches are A – p, s and t; B – q and r;
idoform test ? C – p and q; and D – s and t; then the correct
O darkening of bubbles will look like the following.
|| p q r s t
(A) (CH3)2 CHCH 2 COCHCH (CH 3 ) 2
| | A q r
CH 3 CH 3
B p s t
O
|| C r s t
(B) CH3(CH2)3CH2 COCHCH CH(CH3)2
| D p q r
CH 3
O 19. Match the following :
|| Column-I Column-II
(C) CH3CH2CH2 CHCOCHCH 2 CH2CH3
| | (A)CH3CH2CH2NH2 (p)Treatment of NaNO2, HCl
CH 3 CH 3 gives N-nitroso compound
O (B) CH3CH2NHCH3 (q)Treatment of NaNO2, HCl
|| gives diazonium chloride
(D) CH3CH2 CHCH 2 COCH2 CHCH 2CH3
| | (C)CH3– N − CH3 (r) Treatment of
CH 3 CH 3 | CH3I (excess)
CH 3 followed by AgOH;
Paragraph # 2 (Ques. 16 to 18) heat gives out alkene
–2
To the 100 ml of 10 (M) aqueous solution of HCl
–2 (D) NH2 (s) Treatment of HCl,
0.1 (M) HA (Ka = 10 ) is added in such a way so
that the final pH of the solution become 1.7 heat gives dealkylation
Given log 2 = 0.3 (t) Treatment with CHCl3/
–2 Alc.KOH gives isocyanide
16. What was the pH of 10 (M) aqueous solution of
HCl ? 20. Match the following :
(A) pH = 2 (B) pH < 2
Column-I Column-II
(C) pH > 2 (D) pH = 1.7

OH , Br (1 eq )
(A) C6H5CH2CHO   
2
→ (p) Redox products
17. What volume of 0.1 (M) HA was required to add in
OH −
aqueous HCl to reduced the final pH equal to 1.7 ? (B) CH3CHO → (q) Enantiomeric
(A) 175 ml (B) 100 ml ( 0 −5 º C )
products
4
(C) 10 ml (D) 75 ml
OH − K
(C) CH3CH2–CH = O → (r) α→
D
< 1(Primary
( 25º C) K αH
18. Which of the following solution is isohydric with
0.1(M) aqueous solution of HA ? isotopic effect)
–2
(A) 0.01(M) aqueous solution of HB (Ka = 10 ) Conc. OH −
–1 (D) CH3– CH – C – H  → (s) Diastereomeric
(B) 0.01(M) aqueous solution of HC (Ka=2×10 ) | || products
(C) 0.01(M) aqueous solution of HNO3 CH 3 O
–3
(D) 1(M) aqueous solution of HD (Ka = 10 ) (t) Disproportionation

SECTION – IV
Matrix – Match Type
This section contains 2 questions. Each question MATHEMATICS
contains statements given in two columns, which have
to be matched. The statements in Column I are labeled
SECTION – I
A, B, C and D, while the statements in Column II are
labeled p, q, r, s and t. any given statement in Column I Straight Objective Type
can have correct matching with ONE OR MORE This section contains 8 multiple choice questions. Each
statements (s) in column II. The appropriate bubbled question has 4 choices (A), (B), (C) and (D), out of
corresponding to the answers to these questions have to which ONLY ONE is correct.
be darkened as illustrated in the following example :

XtraEdge for IIT-JEE 55 APRIL 2010


1. Let F denote the set of all onto functions from A = y+3
{a1, a2, a3, a4} to B = {x, y, z}. A function f is chosen (C) log ((y + 3)2 + (x + 2)2) + 2 tan–1 =C
x+2
at random from F. The probability that f –1 (x)
y+3
consists of exactly two elements is (D) log ((y + 3)2 + (x + 2)2) – 2 tan–1 =C
(A) 2/3 (B) 1/3 (C) 1/6 (D) 0 x+2

2. A letter is known to have come from either SECTION – II


TATANAGAR or CALCUTTA. On the envelope,
Multiple Correct Answers Type
just two consecutive letters TA are visible . The
probability that the letter has come from This section contains 4 multiple correct answer(s) type
CALCUTTA is questions. Each question has 4 choices (A), (B), (C) and
(A) 4/11 (B) 1/3 (C) 5/12 (D) None (D), out of which ONE OR MORE is/are correct.
3. A person writes 4 letters and addresses 4 envelopes. 9. If n > 1, then (1 + x)n – nx – 1 is divisible by -
If the letters are placed in the envelops at random, the (A) x2 (B) x3
probability that not all letters are placed in correct 4
(C) x (D) x5
envelopes is
(A) 1/24 (B) 11/24 (C) 5/8 (D) 23/24
1 − sin 4A + 1
4. The value of x for which the matrix 10. If y = , then one of the values of y is -
1 + sin 4A − 1
 2 0 7 (A) – tan A (B) cot A
A =  0 1 0 is inverse of
π  π 
 1 − 2 1 (C) tan  + A  (D) – cot  + A 
4  4 
− x 14 x 7x 

B=  0 1 0  is 11. Equation of a common tangent to the circles
 x − 4x − 2x  x2 + y2 – 6x = 0 and x2 + y2 + 2x = 0 is -
(A) x = 1 (B) x = 0
1 1 1 1
(A) (B) (C) (D) (C) x + 3 y + 3 = 0 (D) x – 3 y + 3 = 0
2 3 4 5
5. The largest term in the expansion of (3 + 2x)50, where x2 t 2 − 5t + 4
x = 1/5, is
(A) 5th (B) 6th (C) 8th (D) 9th
12. The points of extremum of ∫ 0 2 + et
are -

(A) x = – 2 (B) x = 1
2 (C) x = 0 (D) x = – 1
6. If z − = 2, then the greatest value of | z | is
z
SECTION – III
(A) 1 + 2 (B) 2 + 2
Comprehension Type
(C) 3 +1 (D) 5 +1
This section contains 2 groups of questions. Each group
16 has 3 multiple choice questions based on a paragraph.
∫ tan
−1 Each question has 4 choices (A), (B), (C) and (D) for its
7. The value of x − 1 dx is
1
answer, out of which ONLY ONE is correct.
16π 4 Paragraph # 1 (Ques. 13 to 15)
(A) + 2 3 (B) π– 2 3
3 3 Consider the region S0 which is enclosed by the curve
4 16 y ≥ 1− x 2 and max. {|x|, |y| } ≤ 4. If slope of a
(C) π + 2 3 (D) π– 2 3
3 3 family of lines is defined as
m(t) = cos t where point (t, 2t + 0.4) lies inside the
8. The solution of region S0. Any member of this family of lines is called
(y + x + 5)dy = (y – x + 1) dx is L1= 0 if it passes through (π, max {t}) and L2 = 0 if it
y+3 passes through the (π, minm {t}).
(A) log ((y + 3)2 + (x + 2)2) + tan–1 =C
x+2 13. Area of region of S0 is -
2 2 –1 y−3 (A) 8 + π/2 sq. units (B) 8 – π/2 sq. units
(B) log ((y + 3) + (x – 2) ) + tan =C
x−2 (C) 8 sq. units (D) 8 + π sq. units

XtraEdge for IIT-JEE 56 APRIL 2010


14. If line lies inside the region S0, then - p q r s t
(A) t ∈ (0, 0.28) (B) t ∈ (0, 1) A q r
(C) t ∈ (0.28, 1) (D) t ∈ [.28, 1]
B p s t
15. L1 = 0 having maximum slope, is -
C r s t
y −1 y −1
(A) = cos 1 (B) = cos (0.28) D p q r
x−π x−π
y−π
(C) = cos (1) (D) None of these 19. Match the following-:
x −1
Column- I Column- II
Paragraph # 2 (Ques. 16 to 18)
Two circles S1 = 0 and S2 = 0 are touching to each (A) The sum of the series (p) – 10C5
other externally at point T, with centre 10
C1, C2 and radii r1 and r2 respectively.
If P and Q be the points of contact of a direct
∑ r =0
20
C r is

common tangent to the two circles and PQ meets the


line joining C1, C2 in S. Tangent at common point T (B) The coefficient of x53 in (q) 100C4
100
is intersecting to the tangent PQ at R point and to other
direct tangent at V point. Let S1= x2 + y2 – 6x = 0 and ∑ 100
C r ( x − 3)100− r 2 r is
S2= x2 + y2 + 2x = 0. r =0

(C) (10C0)2 –(10C1)2 +……. 1 20


16. Angle between the two direct tangents is– (r) 219+ C10
(A) 90º (B) 30º …..–(10C9)2 + (10C10)2 equals 2
(C) 60º (D) None of these
(D) The value of (s) – 100C53
17. Direct tangents are– 5

(A) y = 3x+ 3,y=– 3x+ 3


95
C4 + ∑j=1
100 − j
C 3 is

x −x
(B) y = – 3,y= – 3 (t) 100C47
3 3
x −x
(C) y = + 3,y= – 3
3 3 20. Match the following -
(D) None of these
Column -I Column -II
18. A circle S = 0 of radius 1 units rolls on the outside of x −2 y−3 z −4
(A) If the lines = = (p) 0
the circle S2 = 0, touching it externally, locus of the 1 1 λ
centre of this outer circle is – x −1 y−4 z −5
(A) Circle (B) Ellipse and = =
λ 2 1
(C) Parabola (D) None of these
intersect at (α, β, γ) then λ =

SECTION – IV π  x +1  (q) –1
(B) If lim 4x  − tan −1    =
Matrix – Match Type x →∞ 4  x + 2 
This section contains 2 questions. Each question y2 + 4y + 5 then y =
contains statements given in two columns, which have
to be matched. The statements in Column I are labeled (C) If chord x + y + 1= 0 of parabola (r) –2
A, B, C and D, while the statements in Column II are y2 = ax subtends 90º at (0, 0)
labeled p, q, r, s and t. any given statement in Column I then a =
can have correct matching with ONE OR MORE
statements (s) in column II. The appropriate bubbled (D) If a = î + ĵ + k̂ , a . b = 1 (s) 1
corresponding to the answers to these questions have to
be darkened as illustrated in the following example : and a × b = ĵ – k̂ , then | b | is
If the correct matches are A – p, s and t; B – q and r; equal to
C – p and q; and D – s and t; then the correct (t) –3
darkening of bubbles will look like the following.

XtraEdge for IIT-JEE 57 APRIL 2010


PHYSICS r1

r2
SECTION – I
Straight Objective Type
2 r22 g 2 r12 g
(A) (σ − ρ) (B) (σ − ρ)
This section contains 8 multiple choice questions. Each 9 η 9 η
question has 4 choices (A), (B), (C) and (D), out of 3 3
2 (r13 − r23 ) (σ − ρ)g
which ONLY ONE is correct. (C) 2 (r1 + r2 ) (σ − ρ)g (D)
9 r1 + r2 η 9 r1 − r2 η
1. ABCD is a smooth horizontal fixed plane on which
4. A block of mass m is attached to an ideal spring and
mass m1 = 0.1 kg is moving in a circular path of
system lies in vertical plane as shown. Initially the
radius r = 1 m. It is connected by an ideal string supporting plane is placed so that spring remains in
which is passing through a smooth hole and connects its natural length then the plane is moved very slowly
of mass m2 = 1/ 2 kg at the other end as shown. m2 downwards. The graph showing variation of normal
reaction applied by mass on supporting plane with
also moves in a horizontal circle of same radius of 1
distance travelled by block is –
m with a speed of 10 m/s. If g = 10 m/s2 then the
speed of m1 is-
A B
m1
M
D C Supporting plane
N N
m2 mg
(A) (B) mg
(A) 10 m/s (B) 10 m/s
x x
1
(C) m/s (D) None of these N
10 mg

2. A L shaped rod whose one rod is horizontal and other (C) (D) None of these
is vertical is rotating about a vertical axis as shown x
with angular speed ω. The sleeve shown in figure has
5. A massless container is filled with liquid of density
mass m and friction coefficient between rod and ρ. It contains two holes as shown in figure. Container
sleeve is µ. The minimum angular speed ω for which rests on ground. Area of the two holes are A each.
sleeve cannot sleep on rod is – Container is filled with liquid upto height H. Then –
ω

m sleeve H
3H/4
H/4
l (A)Torque produced by normal force between
g µg container & ground about center of gravity is
(A) ω = (B) ω =
µl l ρAgH 2
into the plane of paper
l 2
(C) ω = (D) None of these (B) Torque produced by friction about center of
µg
gravity is ρAgH2 out of the plane of paper
(C) Net torque produce by thrust force & friction
3. Two solid spherical balls of radius r1 & r2
ρAgH 2
(r2 < r1), of density σ are tied up with a string and force about center of gravity is into the
4
released in a viscous liquid of lesser density ρ and plane of paper
coefficient of viscosity η, with the string just taut as (D) Torque produced by normal force between
shown. The terminal velocity of spheres is - container and ground about centre of gravity is
zero

XtraEdge for IIT-JEE 58 APRIL 2010


A
6. A B
ψ
l
Two containers A & B contain ideal gases helium R
and oxygen respectively. Volume of both containers O B
are equal and pressure is also equal. Container A has
twice the number of molecules than container B then l
if vA & vB represent the rms speed of gases in
containers A & B respectively, then - 1 1
(A) 2 (B) (C) 2 2 (D)
vA vA 2 2 2
(A) = 2 (B) =4
vB vB

(C)
vA
=2 (D)
vA
= 8 SECTION – II
vB vB Multiple Correct Answers Type

7. A capacitor is composed of three parallel conducting This section contains 4 multiple correct answer(s) type
plates. All three plates are of same area A. The first pair questions. Each question has 4 choices (A), (B), (C) and
of plates are kept a distance d1 apart and the space (D), out of which ONE OR MORE is/are correct.
between them is filled with a medium of a dielectric ε1.
The corresponding data for the second pair are d2 & ε2 9. A body moves in a circular path of radius R with
respectively. What is the surface charge density on the deceleration so that at any moment of time its
middle plate ? tangential and normal acceleration are equal in
d1 d2 magnitude. At the initial moment t = 0, the velocity
of body is v0 then the velocity of body at any time
ε1 ε2 will be –
v0
(A) v = at time t
 v0 t 
1 + 
 R 

V0 S

(B) v = v 0 e R after it has moved S meter
–SR
(C) v = v0e after it has moved S meter
(D) None of these
ε ε  ε ε 
(A) ε 0 V  1 + 2  (B) − ε 0 V  1 + 2 
 d1 d 2   d1 d 2  10. A cylinder block of length L = 1m is in two
ε ε  ε ε  immiscible liquids. Part of block inside liquid(1) is
(C) 2ε 0 V  1 + 2  (D) − 2ε 0 V  1 + 2  1 1
 d1 d 2   d1 d 2  m and in liquid (2) is m. Area of cross-section
4 4
8. The mirror of length 2l makes 10 revolutions per of block is A. Densities of liquid (1) & (2) are ρ and
minute about the axis crossing its mid point O and 2ρ respectively –
perpendicular to the plane of the figure There is a
light source in point A and an observer at point B of
the circle of radius R drawn around centre
O (∠AOB = 90º) ρ. Liquid (1)
R
What is the proportion if the observer B first sees
l
2ρ. Liquid (2)
the light source when the angle of mirror
ψ = 15º ?

XtraEdge for IIT-JEE 59 APRIL 2010


(A) Density of block is 3ρ/4 Paragraph # 1 (Ques. 13 to 15)
(B) Force exerted by liquid (1) on block is ρAg/4 In the shown arrangement, both springs are relaxed.
(C) Block is depressed so that it is just completely The coefficient of friction between m2 and m1 is µ.
immersed in liquid (1) and released. A initial There is no friction between m1 and surface. If the
acceleration of block is 4/3 g blocks are displaced slightly they perform SHM
(D) In case (C) force exerted by liquid (2) on block is together
3/2 ρAg

k2
11. R = 10Ω & E = 13 V and voltmeter & Ammeter are m2
k1
ideal then - m1
a

8V R 13. If the small displacement of blocks is x then


V acceleration of m2 is-
6V k2x (k1 + k 2 ) x
(A) (B)
b c m2 m2
3Ω
E  k + k2 
A (C)  1  x (D) None of these
 m1 + m 2 
(A) Reading of Ammeter is 2.4 A
(B) Reading of Ammeter is 8.4 A 14. The condition in which frictional force on m2 acts in
(C) Reading of voltmeter is 8.4 V the direction of its displacement from mean position
is –
(D) Reading of voltmeter is 27 V
m1 k 1 m 2 k1
(A) > (B) >
m2 k 2 m1 k 2
12. A parallel plate air capacitor is connected to a
battery. If plates of the capacitor are pulled further m1 k 1
(C) = (D) None of these
apart, then which of the following statements are m2 k 2
correct -
(A) Strength of electric field inside the capacitor 15. If the condition obtained in Q.15 is met, then the
remain unchanged, if battery is disconnected maximum amplitude of oscillation is –
before pulling the plate.
µm 2 g (m1 + m 2 ) µm 2 g(m1 + m 2 )
(B) During the process, work is done by an external (A) (B)
m1k 2 − m 2 k1 m1 k 1 − m 2 k 2
force applied to pull the plates whether battery is
disconnected or it remain connected. µm 2 g (m1 + m 2 )
(C) (D) None of these
(C) Potential energy in the capacitor decreases if the m1k 2 + m 2 k1
battery remains connected during pulling plates
apart.
Paragraph # 2 (Ques. 16 to 18)
(D) None of the above A conducting rod PQ of mass M rotates without
friction on a horizontal plane about Ο on circular rails
SECTION – III of diameter 'l'. The centre O and the periphery are
Comprehension Type connected by resistance R. The system is located in a
uniform magnetic field perpendicular to the plane of
This section contains 2 groups of questions. Each group the loop. At t = 0, PQ starts rotating clockwise with
has 3 multiple choice questions based on a paragraph. angular velocity ω0. Neglect the resistance of the rails
Each question has 4 choices (A), (B), (C) and (D) for its and rod, as well as self inductance.
answer, out of which ONLY ONE is correct.

XtraEdge for IIT-JEE 60 APRIL 2010


⊗ B Q 19. A uniform solid cube is floating in a liquid as shown in
the figure, with part x inside the liquid. Some changes in
O ω0 parameters are mentioned in Column I. Assuming no
other changes, match the following -
R P

16. Magnitude of current as a function of time


Bω0 l 2 −αt Bω0l 2 −2αt
(A) e (B) e Column I Column II
2R 16R
Bω0 l 2 −αt Bω0l 2 −2αt (A) If density of the liquid (p) Increase
(C) e (D) e
8R 8R decreases, x will
3B 2l 2 (B) If height of the cube is (q) Decreases
Where α =
8RM increased keeping base area

17. Total charge flow through resistance till rod PQ stop and density same, x will
rotating . (C) If the whole system is (r) Remain same
ωM ωM ωM ωM accelerated upward, then
(A) 0 (B) 0 (C) 0 (D) 0
8B 3B 6B 9B x will
18. Heat generated in the circuit by t = ∞ (D) If the cube is replaced (s) May increase
Ml 2
ω02 Ml 2
ω02 by another cube of same or decrease
(A) (B)
24 8 size but lesser density, x will
2
Ml ω02 Ml 2
ω02 (t) none
(C) (D)
3 32
20. A block of mass m = 1 kg is at rest with respect to a
SECTION – IV rough wedge as shown in figure.
Matrix – Match Type
m
This section contains 2 questions. Each question
contains statements given in two columns, which have µ
a
to be matched. The statements in Column I are labeled
A, B, C and D, while the statements in Column II are θ
labeled p, q, r, s and t. any given statement in Column I
can have correct matching with ONE OR MORE The wedge starts moving up from rest with an
statements (s) in column II. The appropriate bubbled acceleration of a = 2m/s2 and the block remains at res
corresponding to the answers to these questions have to with respect to wedge then in 4 sec. of motion of wedge
be darkened as illustrated in the following example :
work done on block (assume angle of inclination of
If the correct matches are A – p, s and t; B – q and r;
C – p and q; and D – s and t; then the correct wedge is θ = 30º and g = 10 m/s2) –
darkening of bubbles will look like the following. Column I Column II
p q r s t
(A) By gravity (p) 144 J (in magnitude)
A q r
(B) By normal reaction (q) 32 J
B p s t
(C) By friction (r) 160 J
C r s t
(D) By all the forces (s) 48 J
D p q r
(t) none

XtraEdge for IIT-JEE 61 APRIL 2010


MOCK TEST FOR IIT-JEE
PAPER - II
Time : 3 Hours Total Marks : 240

Instructions :
• This question paper contains 57 questions in Chemistry (19,) Mathematics (19) & Physics (19).
• In section -I (4 Ques) of each paper +3 marks will be given for correct answer & –1 mark for wrong answer.
• In section -II (5 Ques) of each paper +4 marks will be given for correct answer & –1 mark for wrong answer
• In section -III (2 Ques.) of each paper +8(2×4) marks will be given for correct answer. No Negative marking for
wrong answer.
• In section -IV (8 Ques.) of each paper +4 marks will be given for correct answer & –1 mark for wrong answer.

The correct structures of 'A' and 'B' are,


CHEMISTRY respectively–
(A) CH2=CH COOH ;
SECTION – I
Straight Objective Type CH=CHCOOH
This section contains 4 multiple choice questions. Each COOH
question has 4 choices (A), (B), (C) and (D), out of
(B) CH2=C ; CH=CH–COOH
which ONLY ONE is correct.

NOBr (C) CH=CHCOOH ; CH2=CH COOH


1. Product

(D) CH=CH–COOH ;
NH2 C=CH2
The main product is – COOH
+ Enantiomer
(A) 3. If H-He undergoes dissociation, which of the
follwoing product most expected to occur –
Br 2HHe products
+ Enantiomer (A) He2 + 2H (B) H2 + 2He
(B)
(C) D2 + He2 (D) 2H + 2He
Br
4. 100 ml solution (I) of buffer containing 0.1(M) HA
Br –
and 0.2 (M) A , is mixed with another solution (II) of
(C) –
+ Enantiomer 100 ml containing 0.2(M) HA and 0.3(M) A .After
mixing what is the pH of resulting solution ?( Given
NO pKa of HA = 5)
(D) (A) 5 – log 5/3 (B) 5 + log 5/3
+ Enantiomer
(C) 5 + log 2/5 (D) 5 – log 5/2

SECTION – II
2. There are two isomeric carboxylic acids– 'A' and 'B'
Multiple Correct Answers Type
C9H8O2. reacts with H2/Pd giving compounds,
C9H10O2. 'A' gives a resolvable product and 'B' gives This section contains 5 multiple correct answer(s) type
a non-resolvable product. Both isomers could by questions. Each question has 4 choices (A), (B), (C) and
oxidised to PhCOOH. (D), out of which ONE OR MORE is/are correct.

XtraEdge for IIT-JEE 62 APRIL 2010


5. Dopamine of a drug used in the treatment of 9. Consider the cell :
parkinson's disease. Ag | AgCl (s) | KCl (1M) | Hg2Cl2(s) | Hg (l) | Pt. The
NH2 cell potential :
HO CH2–CH –
COOH (A) increases on increasing concentration of Cl ions
HO –
Dopamin (B) decreases on decreasing concentration of Cl ions

Which of the following statements about this (C) is independent of concentration of Cl ions
compound are correct ? (D) is independent of amounts of AgCl (s) and
(A) It can exist in optically active forms. Hg2Cl2 (s)
(B) One mole will react with three moles of sodium
hydroxide to form a salt SECTION – III
(C) It can exist as a Zwitter ion in the aqueous Matrix - Match Type
solution This section contains 2 questions. Each question
(D) It gives nitroso compound on treatment with contains statements given in two columns, which have
HNO2. to be matched. The statements in Column I are labeled
A, B, C and D, while the statements in Column II are
6. Which of the following method(s) would be useful
labeled p, q, r, s and t. any given statement in Column I
for preparing ketones ?
can have correct matching with ONE OR MORE
(A) Friedel-Crafts reaction of an acyl chloride with
statements (s) in column II. The appropriate bubbled
benzene (AlCl3 catalyst)
corresponding to the answers to these questions have to
(B) Reaction of methyllithium with the lithium salt of
be darkened as illustrated in the following example :
carboxylic acid, followed by hydrolysis
If the correct matches are A – p, s and t; B – q and r;
(C) Reaction of R2CuLi with an acyl choride in ether
C – p and q; and D – s and t; then the correct
at low temperature
darkening of bubbles will look like the following.
(D) Reaction of Grignard reagents with acyl chloride
in ether followed by hydroylsis p q r s t
A q r
7. In the given table types of H bonds and some H bond
energies are given and other H bond energies are not B p s t
given. You are to perdit the unknown H-bond C r s t
energies.
Types of H-bonds H-bond energies in KJ/mol D p q r
……..
(I) F–H O – 10. Match the following
……..
F–H F 30 Column I Column II
(II) O – H
……..
O – (A) HCOOH (p) Decarboxylation
…….. on heating
O–H F 15 (B) CH3COOH (q) Reation with Br2
…….. – OH
(III) F – H F – (C) (r) Cu2+(alkaline)→Cu2O
…….. COOH
(IV) N – H N –
Correct prediction are –
(D) PhCH2COOH (s) Decarbonylation or
(A) H-bond energy for (I) may be 20 kJ/mol
decarboxylation on
(B) H-bond energy for (II) may be 25 kJ/mol treatment with
(C) H-bond energy for (III) may be 113 kJ/mol conc. H2SO4
(D) H-bond energy for (IV) may be 12 kJ/mol (t) Reaction with I2+NaOH

8. During conductance measurement of an electrolyte 11. Match the following


based on the wheatstone bridge principle alternating Column I Column II
current is used because direct current produces – (A) XeO4 (p) Non polar
(A) polymerisation (B) XeF4 (q) Having no lone pair of
(B) Ionisation electron in the central atom
(C) Electrolysis resulting in the chnge of (C) SO3 (r) Planar
concentration and in consequencae the resistance (D) BF3 (s) Having lone pair of
electrons in the central atom
(D) Polarisation at the electrodes resulting in the (t) Tetrahedral
change of resistance

XtraEdge for IIT-JEE 63 APRIL 2010


SECTION – IV 17. The stopcock, connecting the two bulbs of volumes
Integer answer type 5 litres and 10 litres containing an ideal gas at 9 atm
and 6 atm respectively, is opened. What is the final
This section contains 8 questions. The answer to each of pressure (in atm) in the two bulbs if the temperature
the questions is a single-digit integer, ranging from 0 to remained the same ?
9. The appropriate bubbles below the respective
question numbers in the ORS have to be darkened. For 18. Sum of lone pairs present in XeOF4 and XeO3 is ….. .
example, if the correct answers to question numbers X,
19. Total number of isomers are possible in [Co(en)2Cl2]
Y, Z and W (say) are 6, 0, 9 and 2, respectively, then
and [Co(C2O4)2(NH3)2]– is.
the correct darkening of bubbles will look like the
following :
X Y Z W MATHEMATICS
0 0 0 0
1 1 1 1
2 2 2 2 SECTION – I
Straight Objective Type
3 3 3 3
4 4 4 4 This section contains 4 multiple choice questions. Each
5 5 5 5 question has 4 choices (A), (B), (C) and (D), out of
6 6 6 6 which ONLY ONE is correct.
7 7 7 7 n n

8 8 8 8 1. Let ∑
r =1
r 4 = f (n ) , then ∑ (2r − 1)
r =1
4
is equal to
9 9 9 9 (A) f(2n) – 16 f(n) (B) f (2n) – 7f (n)
(C) f(2n – 1) – 8 f(n) (D) None of these
12. 3 ampere current was passed through an aqueous
solution of an unknown salt of Pd for one hour,  α α
2. If fr(α) =  cos 2 + i sin 2  ×
2.977g of Pdn+ was deposited at cathode. Find n.  r r 
(At. wt. of Pd = 106.4)
 2α 2α   α α
13. Consider the following reaction sequence  cos 2 + i sin 2  ….  cos + i sin 
 r r   r r
NH 2 NH 2 O3 / H 2O then lim f n (π) equals
O KOH
 → W  → X
/ glycol n →∞
(A) –1 (B) 1
Ca ( OH ) 2 C H COOOH (C) – i (D) i
 → Y    → Z
6 5

∆ γ
How many carbons are present in the final product Z? 3. If α, β, γ, δ are four complex numbers such that is
δ
14. For the reaction in the plant cells α + βt
real and αδ – βγ ≠ 0, then z = ,
6CO2(g) + 12H2O (l) → C6H12O6(s) + 6O2(g) γ + δt
+ 6H2O(l) ∆rGº = 3000 kJ/mol t ∈ R represents a
ATP → ADP + PO43– ∆rGº = – 30 kJ/mol (A) circle (B) parabola
Glucose is stored in the plant cell as starch, (C) ellipse (D) straight line
(C6H10O5)n. To produce 162 g of starch how many
moles of ATP are minimum required ? Give your 4. The inflection points on the graph of function
answer after divide actual answer by 100. x

∫ (t − 1)(t − 2)
2
y= dt are
15. The relative lowering of the vapour pressure of an
0
aqueous solution containing a non-volatile solute is
(A) x = –1 (B) x = 3/2
0.0125. What is the molality of the solution. Give
(C) x = 4/3 (D) x =1
your answer after multiplying actual answer by 10.
16. A polyvalent metal weighing 0.1g and having atomic SECTION – II
weight 51 reacted with dil. H2SO4 to give 43.9 ml of
Multiple Correct Answers Type
hydrogen at STP. The solution containing the metal
in this lower oxidation state(X), was found to require This section contains 5 multiple correct answer(s) type
58.8 mL of 0.1 N KMnO4 for complete oxidation. questions. Each question has 4 choices (A), (B), (C) and
What is the higher oxidation state (Y) of the metal ? (D), out of which ONE OR MORE is/are correct.

XtraEdge for IIT-JEE 64 APRIL 2010


5. If A and B are two invertible matrices of the same order, 10. Match the following
then adj (AB) is equal to - Column-1 Column-II
(A) adj (B) adj (A) (B) |B| |A| B–1A–1 Equation General Solutions
–1 –1
(C) |B| |A| A B (D) |A| |B| (AB)–1 π
(A) 2 sin θ – 3=0 (p) nπ + (–1)n
3
 cos α sin α 0  π
(B) 2 sin 2θ + 3 (q) 2nπ –
If A (α , β) = − sin α cos α 0  , then
3
6.
 0 0 e β  = 2 sin θ + 2 3 cos θ
π
(A) A (α, β)′ = A (– α, β) (C) sin 2θ + cos 2θ + 4 sin θ (r) 2nπ +
3
(B) A (α, β)–1 = A (–α, –β) = 1 + 4 cos θ
(C) Adj (A (α, β)) = e–β A (–α, –β) 1 π
(D) cos2θ = (s) nπ –
(D) A (α, β)′ = A (α, – β) 4 4
π
(t) nπ +
7. For a positive integer n, if the expansion of 4
 5
n 11. Match the following :
4
 2 +x  has a term independent of x, then n can
x  Column- I Column- II
be (A) If three unequal numbers a, b, c (p) 1/3
(A) 18 (B) 21 are in A.P. and b – c,
(C) 27 (D) 99 c – b, a are in G.P., then
a 3 + b3 + c3
8. If k is odd then kCr is maximum for r equal to is equal to
3abc
1 1
(A) (k – 1) (B) (k + 1) (B) Let x be the arithmetic mean (q) 1
2 2 and y,z be two geometric
(C)k – 1 (D) k means between any two
positive numbers, then
9. Let an = (111... 1) , then 3
y +z 3
1424 3 is equal to
n times
xyz
(A) a912 is not prime (B) a951 is not prime
(C) a480 is not prime (D) a91 is not prime (C) If a, b, c be three positive (r) 2
number which form three
SECTION – III successive terms of a G.P. and
Matrix - Match Type c> 4b –3a, then the common
ratio of the G.P. can be equal to
This section contains 2 questions. Each question
contains statements given in two columns, which have  n  1 
to be matched. The statements in Column I are labeled
A, B, C and D, while the statements in Column II are
n →∞ ∑
(D) lim tan  tan −1  2
 r =1  2r


(s) 3

labeled p, q, r, s and t. any given statement in Column I is equal to


can have correct matching with ONE OR MORE
statements (s) in column II. The appropriate bubbled (t) 1/2
corresponding to the answers to these questions have to
be darkened as illustrated in the following example :
If the correct matches are A – p, s and t; B – q and r; SECTION – IV
C – p and q; and D – s and t; then the correct Integer answer type
darkening of bubbles will look like the following.
This section contains 8 questions. The answer to each of
p q r s t
the questions is a single-digit integer, ranging from 0 to
A q r 9. The appropriate bubbles below the respective
p s t question numbers in the ORS have to be darkened. For
B
example, if the correct answers to question numbers X,
C r s t Y, Z and W (say) are 6, 0, 9 and 2, respectively, then
the correct darkening of bubbles will look like the
D p q r following :

XtraEdge for IIT-JEE 65 APRIL 2010


X Y Z W What does the value of capacitance, if both the plate
0 0 0 0 of capacitor is shortened ?
1 1 1 1 (A) C (B) 2C
(C) 3C (D) 4C
2 2 2 2
3 3 3 3 2. White light is incident normally on a glass surface
4 4 4 4 (n = 1.52) that is coated with a film of
5 5 5 5 mg F2 (n = 1.38). For what minimum thickness of the
6 6 6 6 film will yellow light of wavelength 550 nm (in air)
7 7 7 7 be missing in the reflected light ?
8 8 8 8
9 9 9 9 mgF2
glass
12. If z ≠ 0 and 2 + cos θ + i sin θ = 3/z, then find the
(A) 99.6 nm (B) 49.8 nm
value of 2 (z + z ) – |z|2.
(C) 19.6 nm (D) 10.6 nm
13. Find the sum of all the integral roots of
(log5 x)2 + log5x (5/x) = 1. 3. A circuit element is placed in a closed box. At time
14. Four different integers form an increasing A.P. such t = 0, a constant current generator supplying a current of I
that one of them is the square of the remaining amp is connected across the box. Potential diff. across the
numbers. Find the largest numbers. box varies according to graph as shown in the figure. The
element in the box is -
15. If n is a positive integer, and
E = 2n + 1C1 + 2n + 1C2 + … + 2n + 1Cn – 2n + 1C2n + 1
– 2n + 1C2n – … – 2n + 1Cn + 1. Find |E| 8
Volts
16. Find the number of values of t for which the system (V)
of equations 2
(a + 2t)x + by + by + cz = 0 3 t(sec)
bx + (c + 2t)y + az = 0 Time
cx + ay + (b + 2t) z = 0 (A) a resistance of 2Ω (B) a battery of emf 6V
has non-trivial solutions. (C) an inductance of 2H (D) a capacitance
17. A function f(x) is defined for x > 0 and satisfies 4. Consider a usual set-up of Young's double slit
f(x2) = x3 for all x > 0. Then the value of f ′(4) is ___. experiment with slits of equal intensity as shown in
18. If A is the area formed by the positive x-axis, and the the figure. Take 'O' as origin and the Y axis as
normal and tangent to the circle x2 + y2 = 4 at indicated. If average intensity between y1 = λD/4d
and y2 = λD/4d equals n times the intensity of
(1, 3 ) then A/ 3 is equal to _____.
maximum, then n equal is (take average over phase
19. The area bounded by the curves x = y2 and difference) -
x = 3 – 2y2 is ______. S1 y

PHYSICS d
S2
O

D
SECTION – I
Straight Objective Type 1 2  2
(A) 1 +  (B) 2 1 + 
This section contains 4 multiple choice questions. Each 2 π  π
question has 4 choices (A), (B), (C) and (D), out of  2 1 2
which ONLY ONE is correct. (C) 1 +  (D) 1 − 
 π 2 π
1. A thin conducting plate is inserted in half way between
the plates of a parallel plates capacitor of capacitance C. SECTION – II
Multiple Correct Answers Type
Conducting plate
This section contains 5 multiple correct answer(s) type
questions. Each question has 4 choices (A), (B), (C) and
(D), out of which ONE OR MORE is/are correct.

XtraEdge for IIT-JEE 66 APRIL 2010


5. A long round conductor of cross-sectional area A is 9. Binding energy per nucleon vs mass number curve
made of a material whose resistivity depends on the for nuclei is shown in the figure. W, X, Y and Z are
radial distance r from the axis of the conductor as ρ four nuclei indicated on the curve. The process that
α would release energy is -
= 2 , α is a constant. The total resistance per unit
r
length of the conductor is R and the electric field Y
strength in the conductor due to which a current I 8.5 X

Nucleon in MeV
Binding Energy/
8.0 W
flows in it is E. 7.5
2πα 4πα
(A) R = 2 (B) R = 2
A A 5.0 Z
2παI 4παI
(C) E = 2 (D) E = 2
A A
6. Two infinite plates carry j ampere of current out of 0 30 60 90 120
the age per unit width of the plate as shown. BP and Mass number of nuclei
BQ represent magnitude of field at points P and Q (A) Y → 2Z (B) W → X + Z
respectively.
(C) W → 2Y (D) X → Y + Z

P SECTION – III
Matrix - Match Type
Q
This section contains 2 questions. Each question
(A) BP = 0 (B) BP = µ0j
(C) BQ = 0 (D) BQ = µ0j contains statements given in two columns, which have
to be matched. The statements in Column I are labeled
7. A bar magnet M is allowed to fall towards a fixed A, B, C and D, while the statements in Column II are
conducting ring C. If g is the acceleration due to
labeled p, q, r, s and t. any given statement in Column I
gravity, v is the velocity of the magnet at t = 2 s and s
is the distance traveled by it in the same time then, can have correct matching with ONE OR MORE
M statements (s) in column II. The appropriate bubbled
corresponding to the answers to these questions have to
be darkened as illustrated in the following example :
If the correct matches are A – p, s and t; B – q and r;
3g C – p and q; and D – s and t; then the correct
darkening of bubbles will look like the following.
p q r s t
C
A q r
(A) v > 2g (B) v < 2g
(C) s > 2g (D) s < 2g B p s t
C r s t
8. In the network shown, the capacitor C is initially
uncharged. The time constant of the circuit is τ and D p q r
the charge on C at time t after the switch S is closed
is q. 10. Capillary rise and shape of droplets on a plate due to
R3 surface tension are shown in column II.
Column I Column II
A
R1
B
R2
S C (A) Adhesive forces is (p)

greater than cohesive forces


 R 2R 3 
(A) τ = CR1 (B) τ = C  R 1 + 
 R 2 + R 3  A
(B) Cohesive forces is (q)
CVR 2 CVR 1
(C) q = (1 − e − t / τ ) (D) q = (1 − e − t / τ ) B
R2 + R3 R1 + R 2 greater than adhesive forces

XtraEdge for IIT-JEE 67 APRIL 2010


(C) Pressure at A > pressure (r) A mercury drop A
at B is pressed between
two parallel plates v1
of glass
v2
B A
30º
13. Portion AB of the wedge shown in figure is rough
B A
(D) Pressure at B > Pressure (s) and BC is smooth. A solid cylinder rolls without
at A slipping from A to B. If AB = BC, then find the value
(t) none of ratio of translational kinetic energy to rotational
kinetic energy, when the cylinder reaches to point C.
11. Column I Column II A
(A) The coefficient of (p ) with decrease
volume expansion at in pressure B
constant pressure is
(B) Mean free path of (q) at all temperatures
D C
molecule increases
(C) An ideal gas obeys (r) Same for all gases 14. In what minimum time after its motion begins will a
Boyle’s and Charle’s 1
Law particle oscillating according to the law x = 7sin πt
2
(D) A real gas behaves (s) At high temperature move from the position of equilibrium to the position
as an ideal gas at low x = – 7/2 units ?
pressure and
(t) none 15. A satellite is revolving in a circular equatorial orbit of
radius R = 2 × 104 km from east to west. Calculate
SECTION – IV the interval (in hrs) after which it will appear at the
Integer answer type same equatorial town. Given that the radius of the
earth = 6400 km and g (acceleration due to gravity)
This section contains 8 questions. The answer to each of = 10 ms–2.
the questions is a single-digit integer, ranging from 0 to
9. The appropriate bubbles below the respective 16. Three capillary tubes of same radius 1 cm but of
question numbers in the ORS have to be darkened. For lengths 1m, 2m and 3m are fitted horizontally to the
bottom of a long cylinder containing a liquid at
example, if the correct answers to question numbers X,
Y, Z and W (say) are 6, 0, 9 and 2, respectively, then constant pressure and flowing through these tubes.
the correct darkening of bubbles will look like the What is the length of a single tube which can replace
the three capillaries.
following :
17. Equations of a stationary and a traveling waves are as
X Y Z W follows : y1 = a sin kx cos ωt and y2 = a sin (ωt – kx)
0 0 0 0 π
The phase difference between two points x1 = and
1 1 1 1 3k
2 2 2 2 3π
x2 = are φ1 and φ2 respectively for the two waves.
3 3 3 3 2k
4 4 4 4 φ
Then find the value of 1 .
5 5 5 5 φ2
6 6 6 6 18. A coil, a capacitor and an AC source of voltage 24 V
7 7 7 7 (rms) are connected in series. By varying the
8 8 8 8 frequency of the source, a maximum rms current of
6A is observed. If this coil is connected to a battery
9 9 9 9
of emf 12 V and internal resistance 4 Ω, then find the
value of current flow through it.
12. A ball A is falling vertically downwards with
velocity v1. It strikes elastically with a wedge moving 19. A potential difference of 103 V is applied across an
horizontally with velocity v2 as shown in figure. Find X-ray tube. Calculate the value of ratio of the
v de-Broglie wavelength of the incident electrons to the
the value of 1 , so that the ball bounces back in
v2 shortest wavelength of X-rays produced.
vertically upward direction relative to the wedge.

XtraEdge for IIT-JEE 68 APRIL 2010


MOCK TEST - AIEEE PATTERN
SYLLABUS : Physics : Full syllabus Chemistry : Full syllabus Mathematics : Full syllabus

Time : 3 Hours Total Marks : 432

Instructions :
• Part A – Physics (144 Marks) – Questions No. 1 to 2 and 9 to 30 consist FOUR (4) marks each and Question No.
3 to 8 consist EIGHT (8) marks each for each correct response.
Part B – Chemistry (144 Marks) – Questions No. 31 to 39 and 46 to 60 consist FOUR (4) marks each and
Question No. 40 to 45 consist EIGHT (8) marks each for each correct response.
Part C – Mathematics (144 Marks) – Questions No.61 to 82 and 89 to 90 consist FOUR (4) marks each and
Question No. 83 to 88 consist EIGHT (8) marks each for each correct response
• For each incorrect response, ¼ (one fourth) of the weightage marks allotted of the would be deducted.

4. Statement-1: Temperature of the body is lowered


PHYSICS considerably if we put wet clothes.
Statement -2: Specific heat of water is high.
1. An P-N-P transistor circuit is arranged as shown. It is (A) Both statement -1 and statement -2 are true
a– and statement -2 is a correct explanation of
the statement -1
P RL = 10 K (B) Both statement -1 and statement -2 are true
N V but statement -2 is not a correct explanation
P of the statement -1
V (C) Both statement -1 and statement -2 are
false
(D) statement -1 is false but the statement -2 is
(A) common base amplifier circuit true.
(B) common-emitter amplifier circuit
(C) common-collector circuit 5. An observer starts moving with uniform acceleration a
(D) None toward a stationary sound source emitting a whistle of
frequency n. As the observer approaches source, the
2. A tuning fork and an air column in resonance tube apparent frequency n' heard by the observer varies
whose temperature is 51°C produces 4 beats in 1 with time as –
second when sounded together. When the n' n'
temperature of the air column decreases, the number
of beats per second decreases. When the temperature
(A) (B)
remains 16°C, only 1 beat per second is produced.
Then the frequency of the tuning fork is - time time
(A) 55 Hz (B) 50 Hz
(C) 68 Hz (D) none n' n'

3. For wave propagation wrong statement is - (C) (D)


(A) The wave intensity remains constant for a plane time
time
wave
(B) The wave intensity decreases as the inverse of the 6. The main scale of a spectrometer is divided into
distance from the source for a spherical wave 720 divisions in all. If the vernier scale consists of
(C) The wave intensity decreases as the inverse 30 divisions, the least count of the instrument is
square of the distance from the source for (30 division of vernier scale coincide with
spherical wave 29 division of main scale) -
(D) Total intensity of the spherical wave over the
(A) 0.1° (B) 1''
spherical surface centered at the source remains
constant at all times (C) 1' (D) 0.1''

XtraEdge for IIT-JEE 69 APRIL 2010


7. Experimental verification of Newton's law of cooling v(cm)
v(cm)
is valid for -
(A) large temperature difference i.e.
30°C to 85°C between hot liquid and surrounding (C) (D)
u(cm) O
(B) very large temperature difference i.e. 5°C to 95°C O u(cm)
between hot liquid and surrounding
(C) small temperature difference i.e. 30°C to 35°C 13. Two concave mirror each of focal length f. A point
between hot liquid and surrounding source is placed at a point midway between two
(D) any temperature difference mirror. The minimum value of d for which only one
8. While studying the dissipation of energy of a simple image of s is formed –
pendulum by plotting a graph between square of
amplitude and time which of the following apparatus
is not essential ? s
(A) Ticker timer (B) Meter scale
(C) Vernier calliper (D) Stop watch
d
9. An object is weighed on a balance whose pans are (A) f (B) 2f
not equal in masses when placed in the left pan, the (C) 3f (D) 4f
object appears to weigh 10.30g but when placed in
the right pan, it appears to weigh 12.62g. What is the
correct mass of the object ? 14. In YDSE, if the intensity of central maxima is IO then
(A) 10.30 g (B) 12.62 g I
the y-coordinate of point where the intensity is O ,
(C) 11.46 g (D) Can not find 2
d = 0.1 mm, D = 1m,
10. When jockey is put at two ends of the potentiometer λ = 5000A°
wire, the galvanometer gives diflections in opposite
directions. It means that apparatus can -
P
(A) not give a null point
(B) give a null point y
d
(C) be faulty O

(D) be used after making some changes in the circuit


D
11. A student performs an experiment to determine the
Young's modulus of a wire exactly 2cm long, by Searle's
method. In a particular reading, the student measures the (A) 1.5 mm (B) 2 mm
extension in the length of the wire to be 0.8 mm with an (C) 1.75 mm (D) 1.25 mm
uncertainty of ± 0.05 mm at a load of exactly 1.0 kg.
The student also measures the diameter of the wire to
15. Two charges of –4µC and +4µC are placed at points
be 0.4 mm with an uncertainty of ± 0.01 mm. Take
g = 9.8 m/s2 (exact). The Young's modulus obtained A (1,0,4) and B (2, –1,5) located in an electric field
from the reading is - →
E = 0.20 î V/cm. The torque acting on the dipole
(A) (2.0 ± 0.3) × 1011 N/m2
is-
(B) (2.0 ± 0.2) × 1011 N/m2
(C) (2.0 ± 0.1) × 1011 N/m2 (A) 8 × 10–5 N-m (B) 8/ 2 × 10–5 N-m
(D) (2.0 ± 0.05) × 1011 N/m2 (C) 8 2 × 10–5 N-m (D) 2 2 × 10–5 N-m
12. A student measures the focal length of a convex lens
by putting an object pin at a distance 'u' from the lens 16. Three concentric spherical metallic shells A, B and C
and measuring the distance 'v' of the image pin. The of radii a, b and c (a < b < c) have surface charge
graph between 'u' and 'v' plotted by the student should
look like - densities σ, – σ and σ respectively. If the shells A
v(cm)
and C are at same potential, then the correct relation
v(cm)
between a, b and c is -
(A) a + c = b (B) b + c = a
(A) (B)
O u(cm) O (C) a – b = c (D) a + b = c
u(cm)

XtraEdge for IIT-JEE 70 APRIL 2010


17. Five identical plates of equal area A are placed 2 2
parallel to and at equal distance d from each other as µ0 i 8 a + b µ0 i 4 a 2 + b2
(A) (B)
shown in figure. The effective capacity of the system 4π ab 4π ab
between the terminals A and B is -
µ0 i 2 a 2 + b2 µ0 i a 2 + b2
(C) (D)
4π ab 4π ab
B
22. A short magnet produces a deflection of 30° when placed
at certain distance in tanA position of magnetometer. If
A another short magnet of double the length and thrice the
pole strength is placed at the same distance in tanB
3 ∈o A 5 ∈o A position of the magnetometer, the deflection produced
(A) (B) will be -
5 d 4 d
(A) 60° (B) 30°
5 ∈o A 4 ∈o A
(C) (D) (C) 45° (D) None of these
3 d 5 d
23. A solenoid has 2000 turns wound over a length of 0.30 m.
18. Read the following statements carefully : Its area of cross-section is 1.2 × 10–3 m2. Around its
Y : The resistivity of semiconductor decreases with central section a coil of 300 turns is wound. If an
increase of temperature. initial current of 2A in the solenoid is reversed in
Z : In a conducting solid, the rate of collision 0.25 sec, the emf induced in the coil is equal to -
between free electrons and ions increases with (A) 6 × 10–4 Volt (B) 4.8 × 10–2 Volt
increase of temperature. –2
(C) 6 × 10 Volt (D) 48 kV
(A) Both Y and Z are true and Z is correct
explaination of Y 24. A 100 volt AC source of frequency 500 Hz is
(B) Both Y and Z are true but Z is not correct connected to a L–C-R circuit with L = 8.1 mH,
explaination of Y C = 12.5 µF and R = 10 Ω, all connected in series.
(C) Y is true but Z is false The potential difference across the resistance is -
(D) Y is false but Z is true (A) 100 V (B) 200 V
(C) 300 V (D) 400 V
19. A 1m long metallic wire is broken into two unequal
parts A and B. The part A is uniformly extended into 25. Which one is correct ?
another wire C. The length of C is twice the length of (A) Resultant of two vectors of unequal magnitude
A and resistance of C is equal to that of B. The ratio can be zero
of resistances of parts A and C is - (B) Resultant of three non-coplanar vectors of equal
1 magnitude can be zero
(A) 4 (B) (C) Resultant of three coplanar vectors is always zero
4
1 (D) Minimum number of non-coplanar vectors whose
(C) 2 (D) resultant can be zero is four.
2
26. A stone thrown with the velocity V0 = 14 m/s at an
20. A 600 cm long potentiometer wire is connected to a
angle 45° to the horizontal, dropped to the ground at
circuit as shown in figure. The resistance of
a distance 'S' from the point where it was thrown.
potentiometer wire is 15r. The distance from point
From what height should the stone be thrown in
A at which the jockey should touch the wire to get
horizontal direction with the same initial velocity so
zero deflection in the galvanometer is –
that it fall at the same spot -
E r
(A) 14.2 m (B) 16.9 m
(C) 10.0 m (D) 9.6 m

J
27. A small body of mass 'm' is attached to one end of a
B
A E/2 light inelastic string of length l. The other end of the
r
G
string is fixed. The string is held initially taut and
horizontal and then body is released. The centripetal
(A) 320 cm (B) 230 cm acceleration of the body and the tension in the string
(C) 160 cm (D) 460 cm when the string reaches vertical position will be -
(A) g, mg (B) 2g, 3 mg
21. A rectangular loop of metallic wire is of length a and
(C) 3g, 2mg (D) 3g, 3 mg
breadth b and carries a current i. The magnetic field
at the centre of the loop is -

XtraEdge for IIT-JEE 71 APRIL 2010


28. Assertion : A rocket moves forward by pushing the 34. The substance not likely to contains CaCO3 is -
surrounding air backwards. (A) Dolomite (B) A marble statue
Reason : It derives the necessary thrust to move (C) Calcined Gypsum (D) Sea shells
forward, according to Newton's third law of motion. 35. On mixing 10ml of acetone with 50ml of CHCl3, the
(A) Both Assertion and Reason are true and Reason total volume of the solutions-
is a correct explanation of the Assertion (A) < 60ml (B) > 60ml
(B) Both Assertion and Reason are true but Reason is (C) = 60ml (D) unpredictable
not a correct explanation of the Assertion
(C) Both Assertion and Reason are false 36. On addition of He gas at constant volume to the
(D) Assertion is false but the Reason is true reaction N2 + 3H2 2NH3 at equilibrium-
(A) The reaction stops
(B) Forward reactions is favoured
29. While slipping on rough spherical surface of radius (C) Reaction remains unaffected
'R', block A of mass 'm' comes with velocity 1.4gR (D) Backward reactions is favoured
at bottom B. Work done in slipping the block from 37. The half life of a reaction is 24 hours . If we start
'B' to 'C' is – with 10gm of reactant, How many grams of it will
m
reaction after 96 hours, (I order reaction)
A C (A) 0.625gm (B) 6.25gm
(C) 1.25gm (D) 0.125gm
38. The current is passed in Ag2SO4 aqueous solution &
1.6 gm O2 is obtained. The amount of Ag deposited
B will be- [Ag = 108gm]
mgR (A) 107.8g (B) 1.6g (C) 0.8g (D) 21.6g
(A) (B) mgR
4 39. In decinormal solution CH3COOH is ionized to the
5 extent of 1.3% find the pH of solution.
(C) 1.3 mgR (D) mgR
4 (A) 3.89 (B) 2.89 (C) 4.89 (D) 5.89

30. A 2000 kg rocket in free space expels 0.5 kg of gas 40. A FCC element (atomic wt.= 60) has a cell edge of
per second at exhaust velocity 400 ms–1 for 5 400pm. Its density is-
seconds. What is the increase in speed of rocket in (A) 6.23 g/cm3 (B) 6.43 g/cm3
3
this time - (C) 6.53 g/cm (D) 6.63 g/cm3
(A) 2000 ms–1 (B) 200 ms–1
(C) 0.5 ms –1
(D) zero 41. Which set of quantum No. is not possible-
n l m s
(A) 2 0 0 +1/2
CHEMISTRY (B) 4 2 –3 –1/2
(C) 3 2 –2 +1/2
(D) 2 1 0 +1/2
31. Which of the following can act as a both Bronsted
acid & Bronsted base - 42. Give simplest formula of compound which
(A) Na2CO3 (B) OH– containing 6gm C, 3.01×1023 atom O and 2 mole H

(C) HCO3 (D) NH3 atoms-
(A) CH2O (B) CH4O (C) CHO (D) CH3O
32. In which compound the oxidation No. of Oxygen
1
is + -
2 43. The IUPAC Name of
(A) OF2 (B)O2F2
(C) O2[PtF6] (D) KO2 (A) 1,2-dimethyl Cyclohexene
(B) 2,3-dimethyl Cyclohexene
33. The favourable conditions for a spontaneous (C) 1,2-dimethyl Cyclohex-2-ene
reactions are - (D) 5,6-dimethyl Cyclohex-1-ene
(A) T ∆S > ∆H, ∆H = ⊕ , ∆S = ⊕
44. Which of the following reagent can make distinction
(B) T ∆S > ∆H, ∆H = ⊕ , ∆S = Θ
between Pri. and Sec. amines ?
(C) T ∆S = ∆H, ∆H = Θ , ∆S = Θ (A) NH3 (B) NaNO2/HCl
(D) T ∆S = ∆H, ∆H = ⊕ , ∆S = ⊕ (C) HCl (D) All

XtraEdge for IIT-JEE 72 APRIL 2010


45. Toluene reacts with Cl2 in the presence of light to 55. Select the true statement about benzene from
give - amongst the following-
(A) Benzyl chloride (B) Benzoyl chloride (A) Because unsaturation benzene easily undergoes
(C) p-chlorotoluene (D) o- chlorotoluene addition reaction
(B) There are two types of C-C bonds in benzene
46. Which compound is formed when excess of KCN is molecule
added to an aqueous solution of copper sulphate
(C) There is a cyclic delocalisation of π es– in
(A) Cu (CN)2 (B) K2 [Ca(CN)4]
(C) K [Cu(CN)2] (D) K3 [Cu(CN)4] benzene
(D) Monosubstitution of benzene group gives three
47. The Blue Print Process involves the use of- isomeric substances
(A) Indigo dyes (B) Iron compound
(C) Vat dyes (D) some other compounds OH H C − Cl
56. Zn
dust
→ B 3 
→ K
AlCl3
48. The ionic radii of N3–, O2–, F– and Na+ follow the alk.KMnO 4
order-   
→ D. Identity ‘D’ ?
(A) N3– > O2– > F– > Na+ (B) N3– > Na+ > O2– > F–
(C) Na+ > O2– > N3– > F– (D) O2– > F– > Na+ > N3– CH3
(A) (B)
49. Reaction
xy2 xy + y CHO COOH
(g) (g) (g) (C) (D)
Initial pressure of xy2 is 600 mm Hg & total pressure
at equilibrium is 800 mm Hg. Kp of reaction is - 57. In Lassaigne’s test, the organic compound is
(A) 50 (B) 100 (C) 166.6 (D) 400 first fused with sodium metal. The sodium
metal is used because
50. Cell: Zn|Zn+2|| Cu+2| Cu (A) The melting point of sodium metal is low
If the correct reactions of Zn+2 & Cu+2 ions are (B) Sodium metal reacts with elements present
doubled, the emf of the cells: in organic compounds to form inorganic
(A) doubled (B) halved
compounds
(C) same (D) zero
(C) All sodium salts are soluble in water
51. What is the pH of buffer solution containing 12g (D) All the above
CH3COOH & 16.4g CH3COONa in 500ml of
solution (Ka for CH3COOH = 1.8×10–5). 58. Concentrated hydrochloric acid when kept in open air
(A) 4.7447 (B) 4.4774 sometimes produces a cloud at white fumes the
(C) 4.4477 (D) None explanation for it is that-
(A) Oxygen in air reacts with the emitted HCl gas to
52. How many moles of ferrous oxalate are completely form a cloud of chlorine gas
oxidized by 1 mole KMnO4 in acidic medium- (B) Strong affinity of HCl gas for moisture in air
(A) 3/5 (B) 5/3
results in forming of droplets of liquid solution
(C) 1/5 (D) 5
which appears like a cloudy smoke
53. In an irreversible process taking place at constant-T (C) Due to strong affinity for water concentrated
& P and which only pressure-volume work is being hydrochloric acid pulls moisture of air towards
done, than (dG) and (dS), satisfy the criteria- itself. This moisture forms droplets of water and
(A) (dS)V, E > 0, (dG)T,P < 0 hence the cloud.
(B) (dS)V, E = 0, (dG)T,P = 0 (D) Concentrated hydrochloric acid emits strongly
(C) (dS)V, E = 0, (dG)T,P > 0 smelling HCl gas all the time
(D) (dS)V, E < 0, (dG)T,P < 0
59. Consider the following complex
54. 3,3-dimethylbutan-2-ol, on reaction with Conc.
[Co(NH3)5CO3]ClO4
H2SO4 at 443K will give…… as major product-
The coordination number, oxidation number, number
(A) 3,3-dimethyl but-1-ene
(B) 2,3-dimethyl but-2-ene of d-electrons and number of unpaired electrons on
(C) 2,2-dimethyl but-2-ene the metal respectively-
(D) 2,2-dimethyl-1- butene (A) 6, 3, 6, 0 (B) 7, 2, 7, 1
(C) 7, 1, 6, 4 (D) 6, 2, 7, 3

XtraEdge for IIT-JEE 73 APRIL 2010


60. Point out the incorrect statement about resonance a1 b1 c1
(A) Resonance structure should have equal energy
66. If ∆ = a 2 b2 c 2 , then
(B) In Resonance structure, the constituent atoms
should be in the same position a3 b3 c3
(C) In Resonance structure there should not be same a 1 + 2b1 − 3c1 b1 c1
number of electron pairs
(D) Resonance structure should differ only in the a 2 + 2b 2 − 3c 2 b2 c2 is equal to -
location of electrons around the constituent a 3 + 2b 3 − 3c 3 b3 c3
atoms (A) ∆ (B) 2∆
(C) – 3∆ (D) 0
MATHEMATICS 67. In a ∆ABC, angle A is greater than angle B. If the
measures of angles A and B satisfy the equation 3sin
61. If the angles of elevation of an aeroplane from two x – 4 sin3x – k = 0, 0 < k < 1, then the measure of
points 1 km apart be 60º and 30º, then the height of angle C is -
the aeroplane is - π π 2π 5π
(A) (B) (C) (D)
500 3 2 3 6
(A) 500m (B) m
3 68. If the p, q, r have truth values.
2000 F, F, T then the statement
(C) m (D) None (p ↔ q) ∨ ~ r → (p ∧ r) will be
3
(A) T (B) F (C) T, F (D) None
62. Suppose a population A has 100 observations 101, 69. The statement (p ∨ q) ↔ (q ∧ ~ p) is a
102, ..........200 and another population B has 100 (A) Tautology
observations 151, 152, ......... 250. If VA and VB (B) Contradiction
represents the variances of the two population (C) Neither tautology nor contradiction
V (D) None of these
respectively then A is -
VB
1 7 1 20 5
9 4 70. If nth term of sequence 2 ,1 ,1 , , .... is
(A) (B) 2 13 9 23 17
4 9
then value of n is -
2 (A) 20 (B) 10 (C) 5 (D) 13
(C) (D) 1
3
71. The ratio in which plane 2x – k = 0 divides the line
63. If vertices of a triangle are (1, 0), (2, b) & (c2, – 3) joining (–2, 4, 7) & (3, – 5, 8) is 9 : 1 then k equal to-
then centroid of triangle (A) 4 (B) 5 (C) 6 (D) 7
(A) can lie on y axis (B) always lie on x axis
(C) lie on x axis if a + b = 3 72. If |a| = 2, |b| = 5 and |a × b| = 8 then |a – b| is equal to:
(A) 12 (B) 15 (C) 17 (D) 5
(D) lie on y axis if c = 3 only
73. The extremities of a line segment of length 6 move in
64. The value of k in order that
two fixed perpendicular lines. If locus of a point P
f(x) = sin x – cos x – kx + b decreases for all real which divides this line segment in ratio 1 : 2 is an
values is given by : ellipse then eccentricity of this ellipse is -
(A) k < 1 (B) k ≥ 1 1 1 3 3
(A) (B) (C) (D)
(C) k > 2 (D) k < 2 2 2 2 4

2
lim e x – cos x 74. The value of p such that the vertex of parabola
65. x→0 is equal to - y = x2 + 2px + 13 is 4 units above x-axis & lies in
x2 first quadrant is :
(A) 3/2 (B) 1/2 (A) 3 (B) 4 (C) ± 3 (D) – 3
(C) 2/3 (D) None

XtraEdge for IIT-JEE 74 APRIL 2010


75. If the lines represented by x2 + 2λx + 2y2 = 0 & lines 84. In a class of 100 students there are 70 boys whose
represented by (1 + λ)x2 – 8xy + y2 = 0 are equally average marks in a subject are 75.If the average
inclined then λ equals : marks of the complete class is 72, then what is the
(A) – 2 (B) + 2 average of the girls.
(C) ± 2 (D) ± 4 (A) 73 (B) 65
(C) 68 (D) 74
76. locus of centre of a variable circle
tx2 + ty2 + 2(t2 + 1)x – 2(t2 – 1)y + t = 0 is a : 85. A letter is taken at random from the letters of word
'STATISTICS' and a another letter is taken at random
(A) Straight line (B) Parabola
from letters of word 'ASSISTANT'. The probability
(C) Ellipse (D) Hyperbola
that they are the same letter is -
(A) 1/45 (B) 13/90
4 4
77. If ∫−1
f ( x ) dx = 4 and ∫ (3 − f (x)) dx = 7 then the
2
(C) 19/0 (D) 5/18

4
86. If A = {x : x ∈ I ; – 2 ≤ x ≤ 2}
value of ∫ f (x) dx is -
2 Β={x:x∈I; 0 ≤ x ≤ 3}
(A) 2 (B) – 3 C = {x : x ∈ N ; 1 ≤ x ≤ 2} and
(C) – 5 (D) None D = {(x, y) ∈ N × N; x + y = 8} then -
(A) n(A ∪ (B ∪ C)) = 5 (B) n(D) = 6
78. Bisector of angle between lines 2x + y – 6 = 0 &
4x – 2y + 7 = 0 which contains origin is - (C) n(B ∪ C) = 5 (D) None of these
(A) acute angle bisector ; x = 5/8
(B) acute angle bisector ; y = 19/4 dy
87. Solution of sec2 y + 2x tan y = x3 is -
(C) obtuse angle bisector ; x = 5/8 dx
(D) obtuse angle bisector ; y = 19/4 2
(A) tan y = ce − x + (x2 – 1)
2 2

∫ [f (g(x))] (B) tan y = ce − x + (x2 – 1)


–1
79. The value of f 'g(x). g'(x) dx where
1 2
g(1) = g(2) is equal to - (C) tan y = ce − x – (x2 – 1)
(A) 1 (B) 2 (D) None of these
(C) 0 (D) None
88. The equation of common tangent to the curves
x 2 + sin 2 x y2 = 8x and xy = –1 is -
80. If f(x) = ∫ 1+ x2
sec2x dx and f(0) = 0 then (A) 3y = 9x + 2 (B) y = 2x + 1
f(1) = (C) 2y = x + 8 (D) y = x + 2
(A) 1 – π/4 (B) π/4 – 1
(C) tan1 – π/4 (D) None of these 89. Let f be twice differentiable function such that
f "(x) = – f(x) and f '(x) = g(x)
sec –1 x h(x) = (f(x))2 + (g(x))2 . If h(5) = 11
81. The domain of the function f(x) = is -
x − [x ] then h(10) is equal to
(A) R (B) R – {(– 1, 1)I} (A) 22 (B) 11
(C) R – I (D) R – [0, 1) (C) 0 (D) None

82. Let f(x) = sin–1x + sec–1x, then - 90. We are required to from different words with the help
(A) Df = [– 1, 1] (B) Rf = {–π/2, π/2} of letter of the word INTEGER. Let m1, be the
(C) Rf = {π/2} (D) None of these number of words in which I and N are never together
and m2 be the number of words which begin with
83. If z1, z2, z3 represents the vertices of an equilateral m
triangle such that I and end with R. Then 1 is given by -
|z1| = |z2| = |z3| then - m2
(A) z1 + z2 = z3 (B) z1 + z2 + z3 = 0 (A) 30 (B) 1/30
(C) z1z2 = 1/z3 (D) z1 – z2 = z3 – z2 (C) 6 (D) 42

XtraEdge for IIT-JEE 75 APRIL 2010


MOCK TEST – BIT-SAT
Time : 3 Hours Total Marks : 450

Instructions :
• This question paper contains 150 questions in Physics (40) Chemistry (40), Mathematics (45), Logical
Reasoning (10) & English (15). There is Negative Marking
• Each question has four option & out of them, ONLY ONE is the correct answer. There is – ve marking.
• +3 Marks for each correct & – 1 Mark for the incorrect answer.

PHYSICS
1. If the amplitude of a damped oscillator becomes half
in 2 minutes, the amplitude of oscillation w.r.t. initial
one after 6 minutes is V V
1 1 1 1 (A) 2 ms–1 (B) 2000 ms–1
(A) (B) (C) (D)
27 8 18 64 (C) 20 ms–1 (D) 200 ms–1

2. An infinite number of spring having force constants 7. A sound level I differ by 4 dB from another sound of
as k, 2k, 4k, 8k ..... ∞ and respectively are connected intensity 10 nW cm–2. The absolute value of intensity
in series; then equivalent spring constant is of sound level I in Wm–2 is
(A) k (B) 2k (C) k/2 (D) ∞ (A) 2.5 × 10–4 (B) 5.2 × 10–4
–2
(C) 2.5 × 10 (D) 5.2 × 10–2
3. A point particle of mass 0.1 kg is executing SHM of
amplitude 0.1 m when the particle passes through the 8. An ideal gas is taken through the cycle A → B → C
mean position. Its kinetic energy is 8 × 10–3 J. The → A as shown. If the net heat supplied to the gas in
equation of motion of this particle when the initial the cycle 5J, the work done by the gas in the process
phase of oscillation is 45º can be given by C→A
 π  π C
(A) 0.1 cos  4t +  (B) 0.1 sin  4t +  2 B
 4  4
3
V(m )
 π π 
(C) 0.4 sin  t +  (D) 0.2 sin  + 2t 
 4 2  1 A

4. A mass m is moving with constant velocity along a P(N/m2) 10


line parallel to x-axis away from the origin. It angular
momentum with respect to origin. (A) – 5 J (B) – 15 J (C) – 10 J (D) –20 J
(A) is zero (B) remains constant 9. There are n electrons of charge e on a drop of oil of
(C) goes on increasing (D) goes on decreasing density ρ. It is in equilibrium in an electric field E.
Then radius of drop is
5. A vessel containing oil (density = 0.8g/cm3) over 1/ 2 1/ 2
mercury (density = 13.6 g/cm3) has a homogeneous  2neE   neE 
(A)   (B)  
sphere floating with half of its volume immersed in  4πρg   ρg 
mercury and other half in oil. The density of material 1/ 3 1/ 3
of sphere in g/cm3 is  3neE   2neE 
(C)   (D)  
(A) 3.3 (B) 6.4 (C) 7.2 (D) 2.8  4πρg   πρg 

6. Two trains move towards each other with the same 10. Two identical cells of emf 1.5 V and internal
speed, speed of sound is 340 ms–1. If the pitch of the resistance 1 Ω are in series. A third cell of similar
tone of the whistle of one is heard on the other parameters is connected in parallel to the
changes by 9/8 times then the speed of each train is

XtraEdge for IIT-JEE 76 APRIL 2010


combination. The terminal voltage of the cells A, B, (A) double of that of α particle
C are (B) four times that of α particle
1.5 V 1.5 V (C) one half time that of α particle
1Ω (D) same as that of α particle
A 1Ω B
15. A coil of inductance 8.4 mH and resistance 6 Ω is
1.5 V 1Ω connected to a 12 V battery. The current in the coil is
1A at approximate time
C
(A) 500 s (B) 20 s (C) 35 ms (D) 1 ms
(A) 1, 1, 2 (B) 1.5, 1.5, 1.5
(C) 1.5, 0, 0 (D) 2, 1, 1
16. A fish rising vertically up towards the surface with
11. A wire has resistance of R ohm at T kelvin. At what speed 3ms–1 observe a bird diving vertically down
temperature the resistance of wire be 2R ohm when towards it with speed 9 m/s. The actual velocity of
temperature coefficient of resistance is α per degree bird is
centigrade.
(273 − T )α + 1 (273 − T )α − 1
(A) (B)
2α 2α y
(273 − T )α − 1 (273 − T )2α + 1
(C) (D)
α α

12. Two cells each of same emf but of internal resistance
r1 and r2 are joined to form a series circuit through an
external resistance R. Value of R in term of r1 and r2
for which cell 1 has zero p.d. across it is (A) 4.5 ms–1 (B) 5.4 ms–1
(C) 3.0 ms–1 (D) 3.4 ms–1
R 17. A concave lens of glass, refractive index 1.5 has both
surface of same radius of curvature R. on immersion in
E E
a medium of refractive index 1.75, it will behave as a
1 r (A) convergent lens of focal length 3.5 R
1 r2 2
(B) convergent lens of focal length 3R
(A) R = r1 – r2 (B) R = r1 + r2
(C) divergent lens of focal length 3.5 R
rr r +r (D) divergent lens of focal length 3 R
(C) 1 2 = R (D) 1 2 = R
r1 + r2 r1r2
18. I is the intensity due to source of light at any point P
13. A current i flows in the network shown. Resulting on the screen if light reaches the point P via two
mangnetic induction at point p is
different paths (a) direct (b) after reflection from a
A F a plane mirror then path difference between two paths
P
a is 3λ/2, the intensity at P is
a a (A) I (B) zero (C) 2I (D) 4I

2a E D 19. The surface of some material is radiated, in turn, by


a
waves of λ = 3.4 × 10–7 m and λ = 5.4 × 10–7 m
a respectively. The ratio of stopping potential in two
cases is 2 : 1, the work function is
B 2a C (A) 2.05 eV (B) 1.05 eV
(C) 3.05 eV (D) None
µ0i 2µ 0 i
(A) (B) –
4πa 8πa 20. A X-ray tube has a working voltage of 40 × 103 V.
8 µ 0i 2 µ0i The continuous spectrum limit of the emitted x-rays is
(C) – (D) (A) 0.17 Å (B) 0.13 Å (C) 0.13 Å (D) 0.31 Å
2 πa 8 πa

14. An alpha particle and a proton have same velocity 21. The number of alpha and beta deca 88Ra222
when they enter a uniform magnetic field. The period experiences before turning into stable Pb206 isotope is
of rotation of proton will be (A) 4, 2 (B) 2, 4 (C) 1, 3 (D) 6, 10

XtraEdge for IIT-JEE 77 APRIL 2010


22. The displacement of interfering light waves are 28. The potential energy of a particle varies with distance
y1 = 4 sin ωt and y2 = 3 sin (ωt + π/2). The amplitude A x
of resultant wave is x from a fixed origin as U = 2 , where A and B
x +B
(A) 5 (B) 7 (C) 1 (D) 0
are dimensional constant then dimensional formula
for AB is
23. A beam of light of wavelength 600 nm from a (A) M L7/2 T–2 (B) M L11/2 T–2
distance source falls on a single slit 1 mm wide and
(C) M2 L9/2 T–2 (D) M L13/2 T–3
resulting diffraction pattern is observed on a screen
2m away. Distance between first dark fringe on either
side of the central bright fringe. 29 A particle leaves the origin at t = 0 and moves in the
+ve x axis direction. Velocity of the particle at any
(A) 1.2 mm (B) 3.2 mm
(C) 2.4 mm (D) 4.2 mm  t
instant is given by v = u 1 −  . If u = 10 m/s and
 t´ 
24. The intensity of light from one source is double of t´ = 5 sec. Find the x coordinate of the particle at an
the other coherent source in a double slit experiment. instant of 10s.
The ratio of destructive to constructive interference in
the obtained pattern is u
(A) 34 (B) 1/34 (C) 17 (D) 1/17
v
25. Two radioactive material of half life T are produced
at different instants. Their activities area found to be
t´ = 5 sec
A1 and A2 respectively when A2 < A1. Their age
difference is t
A A (A) 0 (B) 10 m (C) 20 m (D) –10 m
(A) 0.44 T log 2 (B) 1.44 T log 1
A1 A2
30. An aero-plane drops a parachutist. After covering a
A A distance of 40 m, he opens the parachute and retards
(C) 4.44 T log 2 (D) 5.44 T log 2
A1 A1 at 2 ms–2. If he reaches the ground with a speed of
2ms–1, he remains in the air for about
26. Three concentric conducting spherical shell x, y and z
have radii a, b and c respectively such that c > b > a,
their surface charge density are σ, –σ and σ
respectively. Then potential Vx is given by
C
b

a
(A) 16 s (B) 3 s (C) 13 s (D) 10 s
x
y 31. A tank moves uniformly along x-axis. It fires a shot
z from origin at an angle of 30º with horizontal while
moving along positive x-axis & the second shot is
σ a2  σ also fired similarly that the tank moved along
(A)  − b + c (B) [a – b + c]
negative x-axis. If the respectively range of the shots
ε0  c  ε0
are 250 m and 200 m along x-axis, the velocity of the
σ σ tank.
(C) [a + b + c] (D) – [a + b – c]
ε0 ε0

27. A certain physical quantity is calculated from the 30º


π
formula (a2 – b2)h, where h, a and b are all lengths.
3
The quantity being calculated is
(A) velocity (B) length
(C) area (D) volume (A) 9.4 m/s (B) 4.9 m/s
(C) 3.9 m/s (D) 5.9 m/s

XtraEdge for IIT-JEE 78 APRIL 2010


32. A large number of particles are moving with same 50 N
magnitude of velocity v but having random
directions. The average relative velocity between any
two particles average over all the pairs is
π π 3 4
(A) v (B) v (C) v (D) v
4 2 π π

33. A body is moving with uniform speed v on an 2 kg 5 kg


horizontal circle from A as shown in the fig. Change
in the velocity in the first quarter revolution is (A) 0, 2.5 ms–2 (B) 0, 0
v1 (C) 2.5 m/s , 2.5 m/s (D) 1 m/s2, 2.5 m/s2
2 2

37. In the shown system m1 > m2. Thread QR is holding


A W E the system. If this thread is cut, then just after cutting
O

S
2
(A) v north (B) 2 v south west
(C) 2 v north-west (D) 2v west
34. A hollow vertical drum of radius r and height H has a m2
small particle in contact with smooth inner surface of Q
the upper rim at point P. The particle is given a m1
horizontal speed u tangential to the rim. It leaves the R
lower rim at Q vertically below P. Taking n as an (A) Acceleration of mass m1 is zero and that of m2 is
integer for number of revolution we get directed upward
P u (B) Acceleration of mass m2 is zero and that of m2 is
directed downward
(C) Acceleration of both the block will be same
H (D) Acceleration of system is given by
 m1 − m 2 
  kg, when k is the spring factor
Q
 m1 + m 2 
2π r 2π r 38. A car of mass M accelerates starting from rest.
(A) n = (B)
H 2H / g 1/ 2
 2pt 
Velocity of the car is given by v =   , where p
2π r u  M 
(C) n = (D) n = 2H / g
u 2H / g 2πr is the constant power supplied by the engine. The
position of car as a function of time is given as
35. A balloon is descending at a constant acceleration a. 1/ 2 1/ 2
 8p   9p 
The mass of the balloon is M. When a mass m is (A)   t3/2 (B)   t3/2
released from the balloon is starts rising with  9M   8M 
acceleration a. Assuming that volume does not  8p 
1/ 2
 9p  3
change when the mass is released, what is the value (C)   t2/3 (D)  t
of m.  9M   8M 
2a a+g 39. Six identical uniform rods PQ, QR, RS and ST, TV,
(A) M (B)  M
(a + g )  2a  UP each weighing w are freely joined at their ends to
form a hexagon. The rod PQ is fixed in a horizontal
2a Ma
(C) (D) position and middle points of PQ and ST are
(a + g ) M a+g connected by a vertical string. The tension in string is
P Q
36. Two blocks of masses 2 kg and 5 kg are at rest on
ground. The masses are connected by a string passing U R
over a frictionless pulley which in under the
influence of a constant upward force F = 50 N. The
T S
accelerations of 5 kg and 2 kg masses are
(A) W (B) 3W (C) 2W (D) 4W

XtraEdge for IIT-JEE 79 APRIL 2010


40. A body of mass 2 kg is being dragged with a uniform 9. The fraction of total volume occupied by the atoms in
velocity of 2 ms–1 on a horizontal plane. The a simple cube is -
coefficient of friction between the body and the π π
surface is 0.2. Work in 5 sec. is (A) (B) 2
4 8
(A) 39.2 J (B) 9.32 J (C) 23.9 J (D) 93.2 J
π π
(C) 2 (D)
6 6
CHEMISTRY 10. In the diagram given below the value X is -

1. 100 kg of iron ore (Fe2O3) containing 20% impurities Cu 2+ + → Cu + +
0.15 V 0.50 V
→ Cu
on reduction with CO give iron equal to -
E° = X Volt
(A) 112 kg (B) 80 kg
(A) 0.325 V (B) 0.65 V
(C) 100 kg (D) 56 kg
(C) – 0.35 V (D) – 0.65 V
2. Given : The mass of electron is 9.11 × 10–31 kg,
Planck’s constant is 6.626 × 10–34 Js, the uncertainty 11. For a first order reaction, A → B, t1/2 = 1 hr. What
involved in the measurement of velocity within a fraction of the initial conc. of A reacts in 4 hrs ?
distance of 0.1 Å is - 15 1 7 1
(A) 5.79 × 107 ms–1 (B) 5.79 × 108 ms–1 (A) (B) (C) (D)
16 16 8 8
(C) 5.79 × 105 ms–1 (D) 5.79 × 106 ms–1
12. An azeotropic solution of two liquids has boiling
3. The van der Waal equation for 0.5 mol of real gas is -
point lower than that of either of them if it -
 a  V−b (A) shows a –ve deviation from Raoult’s Law
(A)  P +    = RT
 4V 2   2  (B) shows a +ve deviation from Raoult’s Law
 a  (C) shows no deviation from Raoult’s Law
(B)  P +  (2V – b) = RT (D) is saturated
 4V 2 
 a  RT 13. In multi-molecular colloidal solution atoms or
(C)  P +  = molecules are held together by -
2 2 ( V – 2b)
 4V  (A) Hydrogen bonding
 a  2RT (B) Strong attraction forces
(D)  P + =
2 ( 2 V − b) (C) Van der Waal’s forces
 4V 
(D) Strong electrical forces
4. One mole of N2O4 is enclosed in a 5L container. At
equilibrium, the container has 0.5 mol of N2O4. The 14. Given : C + 2S → CS2 ; ∆H° = + 117 kJ
equilibrium constant for the decomposition of N2O4 C + O2 → CO2 ; ∆H° = –393 kJ
[N2O4 (g) 2NO2(g)] is- S + O2 → SO2 ; ∆H° = – 297 kJ
2 1 The value of ∆Hcombustion of CS2 in kJ mol–1 is
(A) 1 (B) (C) 3 (D)
5 5 (A) – 1104 (B) + 1104 (C) + 807 (D) – 807

5. Which one is the strongest Bronsted Lowry base out 15. Aspirin is chemically -
of the following - (A) Methyl salicylate
(B) Ethyl salicylate
(A) ClO– (B) ClO −2 (C) ClO 3− (D) ClO −4
(C) Acetyl salicylic acid
6. The pH of a solution obtained by mixing 50 mL of (D) o-hydroxy benzoic acid
0.4 M HCl with 50 ml of 0.2 N NaOH is -
16. Aniline when diazotized in cold and then treated with
(A) – log 2 (B) – log 2 × 10–1
dimethyl aniline gives a colored product. It’s
(C) 1.0 (D) 2.0
structure would be -
7. Oxidation number of sulphur in H2SO5 is- (A) (CH3)2N N=N
(A) +2 (B) + 4 (C) +8 (D) +6
(B) (CH3)2N NH
8. Equivalent mass of FeC2O4 in the reaction
FeC2O4 → Fe3+ + CO2 is - (C) CH3NH N=N NHCH3
(M.wt of FeC2O4 = M)
(A) M (B) M/2 (C) M/3 (D) 2M/3 (D) CH3 N=N NH2

XtraEdge for IIT-JEE 80 APRIL 2010


17. Ethyl ester  
3

CH MgBr
→ P. The product P will be - 23. Which one is electrophilic addition -
Excess (A) CH3 – CH3 + Cl2 → C2H5Cl + HCl
H3C CH3 H 3C C2H5 (B) CH3CH = O + HCN → (CH3)2C(OH)CN
(A) (B) (C) (CH3)2C = O + HCN → (CH3)2C(OH)CN
H3C OH H5C2 OH (D) CH3 = CH2 + Br2 → CH2BrCH2Br
C2H5 C2H5 H5C2 C2H5
(C) (D) 24. Which one of the following conformation of
C2H5 OH H7C3 OH
cyclohexane is chiral -
(A) Twist boat (B) Rigid
18. When m-chlorobenzaldehyde is treated with 50% (C) Chair (D) Boat
KOH solution, the product(s) obtained is -
OH OH 25. The dipole moment is the highest for -
CH – CH (A) Trans-2-butene (B) 1,3- dimethyl benzene
(A) (C) Acetophenone (D) Ethanol
OH OH
COO –
CH2OH 26. IUPAC name of the following compound -
+ O CH3
(B)
C N
OH OH CH3
COO– (A) N, N-dimethylcyclo propanecarboxamide
CH2OH (B) N-methylcyclopropanamide
(C) + (C) Cyclopropanamide
Cl (D) None of the above
Cl
OH OH
27. When a mixture of solid NaCl, solid K2Cr2O7 is
CH – CH
(D) heated with conc. H2SO4 orange red vapours are
obtained of the compound –
Cl Cl
(A) Chromous Chloride (B) Chromyl Chloride
(C) Chromic Chloride (D) Chromic sulphate
19. Phenol is less acidic than -
(A) Acetic acid (B) p-Nitrophenol 28. Which of the following will give a pair of
(C) Both (A) & (B) (D) None of these enantiomorphs -
(A) [Co(NH3)4Cl2] NO2 (B) [Cr(NH3)6] [Co(CN)6]
OH (C) [Co(en)2Cl2]Cl (D) [Pt(NH3)4] [Pt Cl6]

20. + C2H5I O


C2 H5
→ Product 29. In following reaction
Anhy.C2H5OH
x
In the above reaction product is - yMnO −4 + xH+ + C 2 O 24− → yMn++ + 2CO2 + H2O
(A) C6H5OC2H5 (B) C2H5OC2H5 2
(C) C6H5OC6H5 (D) C6H5I x and y are
(A) 2 and 16 (B) 16 and 2
21. When acetylene is passed through dilute H2SO4 (C) 8 and 16 (D) 5 and 2
containing Hg2+ ions, the product formed is -
(A) Acetone (B) Acetic acid 30. A reduction in atomic size with increase in atomic
(C) Acetaldehyde (D) Formaldehyde number is a characteristic of element of -
(A) High atomic mass
22. Among the following statements on the nitration of (B) d-block
aromatic compounds, the false one is - (C) f – block
(A) The rate of nitration of benzene is almost the same (D) Radioactive series
as that of hexadeuterobenzene
(B) The rate of nitration of toluene is greater than that 31. Which statement is not correct for nitrogen –
of benzene (A) It has a small size
(C) The rate of nitration of benzene is greater than that (B) It does not readily react with O2
of hexadeuterobenzene (C) It is a typical non-metal
(D) Nitration is an electrophilic substitution reaction (D) d-orbitals available for bonding

XtraEdge for IIT-JEE 81 APRIL 2010


32. Which one of the following is not an amphoteric
substance - MATHEMATICS
(A) HNO3 (B) HCO 3− (C) H2O (D) NH3
1. Consider the sequence (angles are measured in
33. Which reaction cannot be used for the production of radians) sin log102 , sin log103 , sin log104 ….then -
halogen acid – (A) all the terms of this sequence are positive
(A) 2KBr + H2SO4 → K2SO4 +2HBr (B) all the terms of this sequence are negative
(B) NaHSO4 + NaCl → Na2SO4 + HCl (C) 1001th term is negative
(D) 10001th term is negative
(C) NaCl + H2SO4 → NaHSO4 + HCl
(D) CaF2 + H2SO4 → CaSO4 + 2HF 2. The order relation between x, sin–1 x & tan–1x
x ∈(0 ,1) is -
34. B(OH)3 + NaOH NaBO2 + Na[B (OH)4] + H2O (A) tan–1x < x < sin–1x (B) sin–1x < tan–1x < x
How can this reaction is made to proceed in forward (C) x < sin–1x < tan–1x (D) None
direction -
(A) Addition of cis 1, 2-diol 3. The smallest positive valve of x satisfying the
equation log2 cos x + log2 (1 – tan x ) + log2(1 + tan x)
(B) Addition of borax
– log2sin x = 1 is -
(C) Addition of trans 1, 2-diol
(A) π/8 (B) π/6 (C) π/4 (D) π/6
(D) Addition of Na2HPO4
4. A pole stands at a point A on the boundary of a
35. Sodium thiosulphate is prepared by -
circular park of radius r and subtends an angle α at
(A) Reducing Na2SO4 solution with H2S
another point B on boundary. If arc AB subtends an
(B) Boiling Na2SO3 solution with S in alkaline
angle α at the centre of the path, the height of the
medium
pole is -
(C) Neutralising H2S2O3 solution with NaOH
(A) r sin α/2 tan α (B) 2r sin α/2 tan α
(D) Boiling Na2SO3 solution with S in acidic medium
(C) 2r sin α/2 cot α (D) None of these
36. The critical temperature of water is higher than that
of O2 because H2O molecule has - 5. The base of a triangle lies along the line x = a and is
(A) Fewer electrons than oxygen of length 2a. The area of the triangle is a2. If the third
(B) Two covalent bond vertex lies on the line -
(C) V-shape (A) x = 0 (B) x = – a
(D) Dipole moment (C) x = 2a, or x = 0 (D) x = 0 or x = – 2a

37. Zone refining is a technique used primarily for which 6. If y = mx bisects an angle between the lines
one of the following process - m2 –1
(A) Alloying (B) Tempring ax2 – 2hxy + by2 = 0 then =
m
(C) Sintering (D) Purification
b–a b–b a+b
(A) (B) (C) (D) None
38. Which one of the following elements has the highest h h h
ionization energy – 7. If the circle x2 + y2 + 2gx + 2fy + c = 0 passes
(A) [Ne] 3s2 3p1 (B) [Ne] 3s2 3p2 through all the four quadrant then -
2 3
(C) [Ne] 3s 3p (D) [Ar] 3d10 4s2 4p2 (A) g = – b (B) C > 0 (C) C < 0 (D) None

39. The correct order of dipole moment is - 8. The equation of the circle which has two normals
(A) CH4 < NF3 < NH3 < H2O (x–1) (y – 2) = 0 and a tangent 3x + 4y = 6 is
(B) NF3 < CH4 < NH3 < H2O (A) x2 + y2 – 2x – 4y + 4 = 0
(C) NH3 < NF3 < CH4 < H2O (B) x2 + y2 + 2x – 4y + 5 = 0
(D) H2O < NH3 < NF3 < CH4 (C) x2 + y2 = 5
(D) (x –3)2 + (y – 4)2 = 5
40. If Nx is the number of bonding orbitals of an atom 9. Circles drawn on the diameter as focal distance of
and Ny is the no. of the antibonding orbitals, then the any point lying on the parabola x2 – 4x + 6y + 10 = 0
molecule/atom will be stable if - will touch a fixed line whose equation is
(A) Nx > Ny (B) Nx = Ny (A) y = 2 (B) y = –1
(C) Nx < Ny (D) Nx ≤ Ny (C) x + y = 2 (D) x – y = 2

XtraEdge for IIT-JEE 82 APRIL 2010


10. The foci of a hyperbola coincide with the foci of the 19. Let f & g be differentiable function satisfying g´(a) = 2,
x2 y2 g(a) = b and fog = I (Identity function), then f´(b) is
ellipse + = 1. If eccentricity of the equal to
25 9
(A) 1/2 (B) 2 (C) 2/3 (D) None
hyperbola is 2, then its equation is
(A) x2 – 3y2 – 12 = 0 (B) 3x2 – y2 – 12 = 0 20. Tangents are drawn from origin to the curve y = sin x
(C) x2 – y2 – 4 = 0 (D) None of these points of contact lie on the curve
→ →
(A) x2 + y2 = x2y2 (B) x2 – y2 = xy
2 2 2 2
11. α and β are two mutually perpendicular unit vector (C) x – y = x y (D) None of these
→ → → → → → → → →
a α + a β + c( α × β ), α + ( α × β ) and c α + c β 21. Two positive numbers whose sum is 16 and sum of
→ → whose cubes is maximum are given by
+ b ( α × β ) are coplaner then c is (A) 8, 8 (B) no such number exist
(A) A.M. of a & b (B) G.M. of a & b (C) 0, 16 (D) None of these
(C) H.M. of a & b (D) None of these
1 1
22. Let f(x) = , g(x) = on [a, b], 0 < a < b. Let
12. The point of contact of the spheres x2 x
x2 + y2 + z2 + 2x – 4y – 4z – 7 = 0 f ( b ) − f (a ) f ´(c)
x2 + y2 + z2 + 2x – 4y – 16z + 65 = 0 = for same a < c < b then c is
g ( b ) − g (a ) g´(c)
(A) (1, 2, 6) (B) (1, 2, –6)
(A) A.M. of a & b (B) G. M. of a & b
(C) (1, –2, 6) (D) (–1, 2, 6)
(C) H.M. of a & b (D) None of these
13. If f(x) = 3 – 4{x2 – 4x + 8}–1 then range of f(x) is
(A) (–∞, 1) ∪ (3, ∞) (B) (2, 3)
(C) [2, 3] (D) None of these
23. ∫ 1 + cos 2 x . sin 2x cos 2x dx =

2
(A) (1 + cos2x)3/2(3 – 2cos2x)2 + c
14. If x > 0 and g is a bounded function then 5
f ( x )e nx + g( x ) 2
lim is (B) (1 + cos2x)3/2(3 – 2 cos2x) + c
5
n →∞ e nx + 1
(A) 0 (B) f(x) (C) g(x) (D) None 2
(C) (1 + cos2x)3/2(3 + 2 cos2x) + c
5
15. If a1 = 1 and an = n(1 + an–1) ∀ n ≥ 2 than (D) None of these
 1  1   1 
lim 1 + 1 + ...1 +  =  x2 x4 
n →∞
 a 1  a 2   an 
(A) 1 (B) e (C) 1/e (D) None
24. ∫ 1 −


2
+ .... dx
4 

(A) sin x (B) – sin x (C) cos x (D) None
16. Let f(x) = |2 sgn 2x| + 2 then f(x) has
(A) removable discontinuity x
(B) infinte discontinuity
25. lim
∫ π/2
(2 − cos t − 1)dt
=
(C) No discontinuity
π x2

(D) essential discontinuity x→
2 ( t − π / 2)dt
π2 / 4

π  log e 2 ln 2 2ln 2
17. If f(x) = cos  [ x ] − x 3  , 1 < x < 2 and [.] = G.I.F. (A) (B) (C) (D) None
 2  π 2π π
 π
then f´  3  is 2/ n2 4 / n2 6/ n2
 2  1   22   32 
  26. lim 1 + 2  1 +  1 + 
n →∞  n   n2   n2 
(A) 0 (B) 3(π/2)2/3    
(C) –3(π/2)2/3 (D) None of these 2n / n 2
 n2 
.... 1 + 2  =
18. If yex = cos x then, y4/ y =  n 
 
(A) –1 (B) 2 (C) –4 (D) None (A) 4/e (B) 3/e (C) 2/e (D) None

XtraEdge for IIT-JEE 83 APRIL 2010


|x| ex
27. The area bounded by the curves y = 2x2 & y = 37. If = B0 + B1x + B2x2 + .... then Bn – Bn–1 = ?
x 1− x
and x = 0 is equal to 1 1 1
(A) (B) (C) (D) None
2 2 2 2 n n n −1
(A) (B) (C) (D) None
3 6 6
38. The number of point (x, y, z) in space whose each
28. Order and degree of the differential equation coordinate is a negative integer such that x + y + z +
y" = (y´+ 3)1/3 are respectively 12 = 0 is
(A) 2, 2 (B) 2, 3 (C) 3, 2 (D) None (A) 55 (B) 110 (C) 75 (D) None
18 21 28
29. If x = y = z then 3, 3 logyx, 3 logz y, 7 logxz are 39. Six boys and six girls sit along a line alternatively
in with probability P1 & along a circle (again
(A) A.P. (B) G.P. (C) H.P. (D) None alternatively) with probability P2 then P1/P2 is equal
to
30. If log2x + log2y ≥ 6 then least possible value of x + y (A) 1 (B) 1/5 (C) 6 (D) None
is
(A) 32 (B) 16 (C) 8 (D) None
40. If f(x) is a polynomial satisfying
31. No. of real roots of the equation 1
x3 + x2 + 10x + sin x = 0 is f (x) f   − f (x )
1 x
(A) 1 (B) 2 (C) 3 (D) ∞ f(x) = and f(2) = 17
2 1
1 f 
32. The roots of the equation ax2 + bx + c = 0, a ∈R+ are x
two consecutive odd positive integers then then the value of f(5) is
(A) |b| ≤ 4a (B) |b| ≥ 4a (A) 624 (B) –124 (C) 626 (D) 126
(C) |b| ≥ 2a (D) None of these
1 x 
33. The sum of the terms of an infinitely decreasing G.P. 41. If A =   is idempotent then x =
is equal to the greatest value of the function 0 2 
f(x) = x3 + 3x – 9 on the interval [–2, 3] and the (A) 0 (B) 2
difference between the first two terms is f´(0) then (C) no such x exist (D) None of these
sum of first terms is
(A) 19 or – 37 (B) 19 42. Let R be a relation on the set of integers given by
(C) –37 (D) None of these a R b if a = 2k b for some integer k then R is
(A) an equivalence relation
34. If the complex number z1 = a + i, z2 = 1 + ib, z3 = 0 (B) reflexive and symmetric but not transitive
form an equilateral triangle (a, b are real number (C) reflexive and transitive but not symmetric
between 0 & 1) then : (D) symmetric and transitive but not reflxive
(A) a = 3 – 1, b = 3/2
(B) a = 2 – 3 , b = 2− 3 b+c c+a a+b
43. Minimum value of + + , (for real
a b c
1 3
(C) a = ,b= +ve numbers a, b, c) is
2 4
(A) 1 (B) 2 (C) 4 (D) 6
(D) None of these
44. From mean value theorem f(b) – f(a) = (b – a) f´(x1);
n
 1 3r 7r 
35. ∑
r =0
(−1) Cr  r + 2 r + 3r + .....∞  is equal to
2

r n
2 2 

a < x1 < b if f(x) =
1
x
then x1 =

1 3 2 a+b 2ab b−a


(A) (B) (C) (D) None (A) ab (B) (C) (D)
2 −1n n
2 −1 n
2 −1 2 a+b a+b

36. The coefficient of x3y4z in the expansion of 1 3 π π


(1 + x + y – z)9 is ∫
45. If f(x) = cot 4 x dx +
3
cot x – cot x and f   =
2 2
(A) 2 . 9C7 . 7C4 (B) – 2 . 9C2 . 7C3 then f(x) is
9 7
(C) C7 . C4 (D) None of these (A) π – x (B) x – π (C) x (D) None

XtraEdge for IIT-JEE 84 APRIL 2010


LOGICAL REASONING
(C) (D)
1. Fill in the blank spaces.
11, 12, 17, 18, 23, 24, (?)
(A) 12 (B) 29 (C) 30 (D) 35
7. In the following question, find out which of the
2. Choose the best alternative. answer figures (A), (B), (C) and (D) completes the
Dum-Dum : Calcutta : : Palam : ? figure-matrix ?
(A) Kerala (B) Delhi (C) Madras (D) Bombay
3. Pick the odd one out –
(A) Wheat (B) Paddy (C) Towar (D) Mustard
4. Which of the following figures (A), (B), (C) and (D)
when folded along the lines, will produce the given ?
figure (X) ?

(A) (B) (C) (D)

(X) 8. The question that follow contain a set of three figures


X, Y and Z showing a sequence of a piece of paper.
Fig. (Z) shows the manner in which the folded paper
(A) (B)
has been cut. These three figures are followed by four
answer figures from which you have to choose a
figure which would most closely resemble the
(C) (D) unfolded form of fig. (Z).

5. In each of the following questions, choose the set of


figures which follows the given rule.
Rule : The series becomes complex as it proceeds :
X Y Z
(A)

(B) (A) (B)

(C)

(D) (C) (D)

6. In following question below, you are given a figure


9. In the following question, complete the missing
(X) followed by four figures (A), (B), (C) and (D)
portion of the given pattern by selecting from the
such that (X) is embedded in one of them. Trace out
given alternatives (A), (B), (C) and (D).
the correct alternative.
?

(X)
(X)

(A) (B) (C) (D)


(A) (B)
10. In the following question, find out which of the
answer figures (A), (B), (C) and (D) complete the
figure - matrix ?

XtraEdge for IIT-JEE 85 APRIL 2010


(A) took to (B) took for
(C) took in (D) took up
9. Select the one which best expresses the same
sentence in Passive or Active Voice.
Get the box broken.
? (A) Get someone to break the box.
(B) They have broken the box
(C) Have the broken box
(A) (B) (C) (D) (D) Break the box
10. Choose the one which best expresses the correct
answer in the speech :
ENGLISH He said, "How shabby you are looking!"
(A) He asked how shabby I was looking
(B) He exclaimed with disgust that I was looking
1. Choose the one which best expresses the meaningful
very shabby
concept :
(C) He exclaimed with sorrow that they were
The state's duty is to . . . . the safety of its Citizens.
looking much shabby
(A) assure (B) ensure (C) insure (D) accept
(D) He told that I was looking much shabby
2. Choose the one which best expresses the meaningful 11. Pick out the mis-spelt word –
concept : (A) Neigh (B) Rein (C) Neice (D) Neither
The company went . . . . in the 1990's recession.
(A) burst (B) bust (C) bursted (D) busted 12. Find out which part of the sentence has an error :
I wonder / what he has done with the book /
(a ) (b)
3. Choose the one which best expresses the meaningful
concept : I lend him / No Error
What can we . . . . from this evidence, Watson ? ( c) (d)
(A) deduce (B) deduct
(C) reduce (D) conduce (A) Wonder
(B) What he has done with the book
4. Choose the one which best expresses the meaningful (C) I lend him
concept in opposite meaning : (D) No Error
Zenith : 13. Pick out the most appropriate pair to fill in the
(A) Sky (B) Firmament blanks in the same order, to make the sentence
(C) nadir (D) naive meaningfully complete :
She was . . . . . because all her plans had gone . . . . .
5. Pick up the correct Synonym for the following word (A) distraught, awry (B) Frustrated, Magnificently
Voracious : (C) Elated, wild (D) Dejected, splendidly
(A) Hungry (B) Wild
(C) Quick (D) Angry 14. Pick out the most effective word from the given
words to fill in the blanks to make the sentence
6. One who travels from place to place : meaningfully complete :
(A) Journey man (B) Tramp Most of the issues discussed in the meeting were
(C) Itinerant (D) Mendicant trivial and only a few were :
(A) Interesting (B) Practical
7. Choose the one which best expresses the meaning of (C) Complex (D) significant
the given idiom/proverb :
To fly off the candle : 15. Pick out the most appropriate pair to fill in the
(A) To dislocate (B) To lose one's temper blanks in the same order, to make the sentence
(C) To take off (D) To be indifferent meaningfully complete :
The . . . . . of glory lead but to the . . . . .
8. Fill in the blanks with one of the options given (A) Paths, grave (B) Ways, happiness
below: (C) Acts, Prosperity (D) Achievements, Suffering
Gandhi Ji . . . . . . smoking in his youth.

XtraEdge for IIT-JEE 86 APRIL 2010


SOLUTION FOR MOCK TEST
IIT-JEE (PAPER - I)
I is more electron deficient and facilitates a faster
CHEMISTRY attack.
5.[A] As water introduces, water dissolves HCl(g) and a
O press drop is produced Liquid level in the
O ||
|| C capillary rises.
1.[A] C6H5 – C CH3 Orientation for
C –C6H5 aldol condensation
6.[D]
H3C
C ||
|| O 7.[B] The energy of AOs depends on the
O
(n + l) values n + l value of (n – 1) d = n + 1 ;
Ozonolysis NH OH, H SO
2.[B] A   → B  
2
 2 
4
→ oxime n + l value of ns = n

(n + l) value of (n + 1)d = n + 3 ; n + l value of nf
formation followed by Beckmann rearrangement. = (n + 3)
C –NH–CH3 But due to lower value of principle quantum no.
|| energy of nf < (n + 1) d
O ∴ energy of (n + 2) s < nf.
N-methyl benzamide
8.[A] A, B and C are magnesium ,aluminium and
So, B is and
C=O silicon. Magnesium form ionic oxide, MgO ;
| Aluminium forms amphoteric oxide ,Al2O3 and
CH 3 silicon forms a giant molecule SiO2.
A is C = C
| | 9.[ B,C,D] Due to resonance cyclohexatriene cation is
CH 3 CH 3 aromatic which causes it's stability.
K Cr O , H +
3.[D] (CH3)3CCH2CH2OH →
2 2 7
10.[ A,B,D] Catalyst lower the activation energy of
H 2O, Heat
forward reaction & backward direction keeping
(CH3)3 CCH 2 COOH the same enthalpy of reaction.
(A)

 
2SOCl
→ (CH3)3 CCH2COCl 11.[ A,B,C] Greater the value of (IE – ∆eg H) greater is the
(B) electronegativity E.N. of
~
P = (1680 + 340)/(4.18) (125) – 4
(CH3)2NH
12.[A,B,C] H = E + PV
O  dH   dE   dV 
||   =  +P   +V
(CH3)3 CCH 2 C N(CH3)2  dP  T  dP  T  dP  T
(C)  dH   dE   dV 
For liquid,   =     +
 dP  T  dV  T  dP  T
LiAlH , ether  dV 
(CH3)3 CC H2CH2N(CH3)2 ←H4O 
 P  +V
( D) 2  dP  T
 dV  ~
4.[C] Attack of nucleophile is a rate determining step For incompressible liquid,   – 0.
O O  dP  T
 dH  ~
F3C O CH3 ∴  – V
 dP  T
I II

XtraEdge for IIT-JEE 87 APRIL 2010


 dH   O 
For ideal gas,   =0  || 
 dP  T  CH 3 − CH − C H − C − OAg Br
*
 CH 3 − CH − CH − Br 
→
2
 | | | | 
 dE   CH 3 CH 3 CH 3 CH 3 
For the real gas, if   = 0,  
 dV T  
 ( ± J ) 
 dH   dV  O
then   =P   +V≠0
 dP  T  dP  T ||
F = CH3– CH − CH − C − O − CH − CH − CH 3
| | | |
13.[A] F(Monoester) Molecular weight = 186 CH 3 CH 3 CH 3 CH 3
No Br2 reaction ⇒ Saturated
Two oxygen ⇒ 2 × 16 = 32
14.[C] I exists as diastereomers and H is optically active.
No. of CH2 = (186 – 32)/14 = 11
So, H is HO– CH − CH2CH2–CH3
Hence, molecular formula of saturated monoester |
F, is C11H22O2 CH 3
Hydrolysis
H2SO4

G H
H H
Optically active, Optically active,
soluble in (Alcohols are not C=C , and
NaOH ⇒ Acid soluble in NaOH) H3C CH2CH3
We have, +ve iodoform test,
Br it suggests O
Ag+ salt →2
±J
Hunsdiecker reaction; HO– CH – R On warning ||
| with H2SO4 G is CH3CH2CH2– CHC − OH
radical intermediate, CH 3 |
so racemic mixture (dehydration)
CH 3
(J contains one I (no diastereomers means
carbon less than G) no geometrical isomers) 15.[D] Since H is optically active and gives negative
It suggests same alkyl group iodoform test, so H is
on one of the doubly bonded HO–CH2– CH – CH2–CH2 and
|
carbon atoms CH 3
I. TsCl O
H optically active ⇒ J
II. NaBr ||
It suggests G contains one more G is CH3CH2 CHCH 2 C –OH
|
carbon atom than H. CH 3
Hence, molecular formula of ester F, is 16.[A]
O
+
|| 17.[D] Final pH = 1.7 or – log [H ] = 1.7
C5H11– C –O–C5H11 + +
or log [H ] = – 2 + 0.3 or [H ] = 2 × 10
–2
O Let v ml of 0.1 (M) HA solution is mixed with
|| –2
C5H11– C –OH(G) C5H11OH(H) 100 ml of 10 (M) HCl. In the mixed solution,
CH 3 v × 10 −4 0 .1 v
| H CH3 [HA] = × 103 = (M) and
* H 2SO 4 , ∆ C=C (I) (100 + v) (100 + v)
⇒ HO – CH – CH – CH3  →
H3C CH3 10 −3 ×103 1
CH3 [HCl]= =
No cis-trans isomers (100 + v) (100 + v)
TsCl * + –
H → CH3– CH – C H – Br HA H + A
NaBr | | 0 .1 v 0.1 v α 0.1 v α
CH 3 CH 3 (1 – α)
(J) (100 + v) (100 + v) (100 + v)
O 1
|| +
(100 + v)
So, G is CH3– CH − CH − C − OH
| | | + (0.1 v α + 1) –2
∴ [H ] = = 2 × 10
CH 3 CH 3 CH 3 (100 + v)

XtraEdge for IIT-JEE 88 APRIL 2010


–2
or 0.1 v α + 1 = 2 + 2 × 10 × v
(0.1vα + 1) 0.1vα
MATHEMATICS
×
(100 + v) (1.00 + v) α –2
Ka = = × 2 × 10 1.[A] Let us first count the number of elements in F.
0.1v(1 − α) (1 − α)
Total number of functions from A to B is 34 = 81.
(100 + v)
The number of functions which do not contain
α – α 1 x(y) [z] in its range is 24.
∴ × 2 × 10 2 = 10–2 or =
(1 − α) 1− α 2 ∴ the number of functions which contain exactly
two elements in the range is 3 . 24 = 48.
or 2α = 1 – α or α = 1/3 The number of functions which contain exactly
1 –2 one element in its range is 3.
∴ 0.1 v × + 1 = 2 + 2 × 10 × v
3 Thus, the number of onto functions from A to B is
81 – 48 + 3 = 36
v v 20 v
or – =1 or =1 [using principle of inclusion exclusion]
30 50 30 × 50 n (F) = 36.
150 Let f ∈ F. We now count the number of ways in
or v = = 75 ml which f –1(x) consists of single element.
2
We can choose preimage of x in 4 ways. The
18.[D] For isohydric solution, Ka1C1 = Ka2C2 remaining 3 elements can be mapped onto {y, z}
is 23 – 2 = 6 ways.
Column Matching ∴ f –1 (x) will consists of exactly one element in
19. [A] → r,s,t; [B] → p,r,s; [C] → s; [D] → q,t 4 × 6 = 24 ways.
NaNO / HCl Thus, the probability of the required event is
For CH3CH2CH2NH2   2 
→ 24/36 = 2/3
+
CH 3CH 2 CH 2 N 2 2.[A] Let E1 denote the event that the letter came from
Diazonium ion is TATANAGAR and E2 the event that the letter
simply int ermediate; came from CALCUTTA. Let A denote the event
not the product that the two consecutive alphabets visible on the
CH3–CH2–CH2–OH envelope are TA. We have P(E1) = 1/2, P(E2) =
CH3 1/2, P(A / E1) = 2/8, P (A / E2) = 1/7. Therefore,
+ OH by Bayes' theorem we have
CH3 N – CH3 on heating doesn't give alkene.
P(E 2 ) P( A / E 2 )
CH3 P(E2 / A) =
P ( E1 ) P ( A / E1 ) + P ( E 2 ) P ( A / E 2 )
20. [A] → q,r; [B] → p,r; [C] → r,s; [D] → p,t 4
OH − / Br2 =
C6H5CH2–CHO  
→ C6H5– CH –CH = O 11
|
Br 3.[D] Required probability = 1 – P (all the letters are
(±) put in correct envelops)
OH The number of the ways of putting the letters in
| the envelops = 4P4 = 4!
CH3CHO  OH −
→ CH3– CH –CH2–CH = O The number of ways of putting letters in correct
0 − 25º C (±) envelops = 1
OH − 1 23
CH3–CH2CH = O→ CH3CH2–CH= C − CHO
25º C
∴ Required probability = 1 – =
| 24 24
CH 3
Conc. OH −
4.[D] We have
CH3– CH − CH = O  → 5x 0 0  1 0 0 
| 
CH 3 AB =  0 1 0  = 0 1 0
O  0 10x − 2 5x  0 0 1
||
CH3– CH − C –O– + CH3– CH − CH2OH ⇒ x = 1/5
| | 5.[B] Greatest term in the expansion of (x + y)n is
CH 3 CH 3
 (n + 1) y 
kth term where k =  
 x+y 

XtraEdge for IIT-JEE 89 APRIL 2010


In the present case
(cos 2A − sin 2A) 2 + 1
 (50 + 1)(2 x )   (51)(2 / 5)  102  10.[A,B,C,D] y =
k=  =  =   =6 (cos 2A − sin 2A) 2 − 1
 3 + 2x   3 + 2 / 5   17 
Thus, 6th term is the largest term. ± (cos 2A − sin 2A) + 1
⇒ y=
± (cos 2A − sin 2A) − 1
4 4 4 4 which gives us four values of y, say y1, y2, y3 and
6.[D] We have | z | = z − + ≤ z− +
z z z |z| y4. We have
4 cos 2A − sin 2A + 1 (1 + cos 2A) − sin 2A
=2+ y1 = =
|z| cos 2A − sin 2A − 1 (cos 2A − 1) + sin 2A
⇒ | z |2 ≤ 2 | z | + 4 ⇒ (| z | – 1)2 ≤ 5 2 cos 2 A − 2 sin A cos A
=
⇒ |z|–1≤ 5 ⇒|z|≤ 5 +1 − 2 sin 2 A + 2 sin A cos A
Also, for z = 5 +1 cos A(cos A − sin A)
= = cot A
4 sin A(cos A − sin A)
z− =2 −(cos 2A − sin 2A) + 1 (1 − cos 2A) + sin 2A
z y2 = =
− (cos 2A + sin 2A) − 1 − (1 + cos 2A) − sin 2A
Therefore, the greatest value of | z | is 5 + 1.
2 sin 2 A + 2 sin A cos A
7.[D] Integrating by parts, the given integral is equal to = = – tan A
16
− 2 cos 2 A − 2 sin A cos A
16
x 1 (cos 2A − sin 2A ) + 1 (1 + cos 2A) − sin 2A
x tan–1 x −1
1
− ∫
1
x 4 x x −1
dx y3 = =
− (cos 2A + sin 2A) − 1 − (1 + cos 2A) − sin 2A

16 1
16
dx 2 cos 2 A − 2 sin A cos A cos A − sin A
=
3
π –
4 ∫
1 x −1
= 2
− 2 cos A − 2 sin A cos A
=–
cos A + sin A
1 − tan A π  π 
3 =– = – tan  − A  = – cot  + A 
16 1 4t (1 + t 2 ) 1 + tan A  4   4 
=
3
π –
4 ∫
0
t
dt ( x = 1 + t2)
−(cos 2A − sin 2A) + 1 (1 − cos 2A) + sin 2A
y4 = =
=
16
3
π – ( 3 + 3 ) = 163 π – 2 3
(cos 2A + sin 2A) − 1 − (1 − cos 2A) + sin 2A
2 sin 2 A + 2 sin A cos A cos A + sin A
= =
8.[C] The intersection of y – x + 1 = 0 and y + x + 5 = 0
2
− 2 sin A + 2 sin A cos A cos A − sin A
is (– 2, –3). Put x = X – 2, y = Y – 3. The given 1 + tan A π 
= = tan  + A  .
dY Y−X 1 − tan A 4 
equation reduces to = . This is a
dX Y+X 11.[B, C, D]
homogeneous equation, so putting Y = υX, we get Equations of the given circles can be written as
dυ υ2 + 1 (x – 3)2 + y2 = 32 (1)
X = − and (x + 1)2 + y2 = 12 (2)
dX υ +1
Equation of any tangent to circle (2) is
 υ 1  dX (x + 1) cos θ + y sin θ = 1 (3)
⇒ − 2 − 2  dυ =
 υ +1 υ +1 X This will be a tangent to circle (1) also if
1 (3 + 1) cos θ − 1
⇒ – log (υ2 + 1) – tan–1 υ = log | X | + C = ± 3 ⇒ 4 cos θ – 1 = ± 3
2 cos 2 θ + sin 2 θ
Y 1
⇒ log (Y2 + X2) + 2 tan–1 =C That is, cos θ = 1 or cos θ = – . When cos θ = 1,
X 2
y+3 we have sin θ = 0, and the equation of the
⇒ log ((y + 3)2 + (x + 2)2) + 2 tan–1 =C
x+2 common tangent (3) becomes
x + 1 = 1 or x = 0 (4)
9.[A] We have
(1 + x)n – nx – 1 When cos θ = –1/2, we have sin θ = ± 3 / 2 , and
= C0 + C1 x + C2 x2 + … + Cn xn – nx – 1 the equations of the common tangents are
= x2 [C2 + C3x + … + Cn xn–2] 1 3
– (x + 1) ± y = 1 ⇒ x – 3 y + 3 = 0 (5)
[Q C1 = n, C0 = 1] 2 2
Thus, (1 + x)n – nx – 1 is divisible by x2.
and x + 3y + 3 = 0 (6)

XtraEdge for IIT-JEE 90 APRIL 2010


12.[A, B, C, D] 18.[A]
2
x2 t − 5t + 4
Let F(x) = ∫ 0 2 + et
dt
C1
x 4 − 5x 2 + 4
⇒ F′(x) = 2
.2x
2 + ex
So from F′(x) = 0, we get x = 0 or 2
60º
5 ± 25 − 16 5 ± 3 • 1
x2 = = = 4, 1 C2
2 2
Hence x = 0, ±2, ±1.

13.[B]
(h + 1)2 + k2 = (1 + 2)2 (circle)
D C Column Matching
y = +4 19. [A] → r; [B] → p,r; [C] → s; [D] → r
10
(A) ∑
r =0
20
C r = 20C0 + 20C1 +……+ 20C10

But, 20C0 + 20C1 +……+ 20C20 = 220


–1 1
x=4 Also, 20C20 = 1 = 20C0, 20C19 = 20C1, 20
C18 = 20C2
x = –4 etc.
∴ given sum = (20C0 + 20C1 +……+ 20
C20)
A y = –4 B – (20C11 +…..+ 20C20)
220 + 20C10– (20C10 + 20C9 + ……+ 20C0)
Shaded region is S0. Area of S0 ∴ 2 (20C0 + 20C1 +…..+ 20C10)
1 = 220 + 20C10
=4×2– π (1)2 = 8 – π/2
2 100
(B) ∑ 100
C r (x –3)100–r 2r
14.[D] y ∈ [0, 4], x ∈ [–1, 1] r =0

m (t) = cost = ((x–3) +2)100 = (x –1)100 = (1 –x)100


100 100
lines y = 2x + 0.4 lies inside the region so
∑ 100
C r (− x ) r = ∑ (−1) (−1)
r r 100
Cr x r
⇒ t ∈ [0, 1] r =0 r =0
2 2
t + (2t + 0.4) – 1 ≥ 0 ⇒ t ∈ [0.28, 1] ∴ Coeff. of x = (–1)53 100C53 = – 100C53
53

15.[B] (Slope)max. = (cos t)max = cos (0.28) and point is (C) We have
(π, 1) (1+ x)10 = 10C0 + 10C1 x +10C2x2 +……+ 10C10 x10
....(1)
y −1 10 10 10 10 2
= cos (0.28) Also (1–x) = C0 – C1x + C2x +…….
x−π
…..+ 10C10x10 ....(2)
Multiplying, we get
C1S r 3
16.[C] = 1 = (1 –x2)10 = (10C0 + 10C1 x + 10C2x2 +……
C 2S r2 1
….+ 10C10x10) × (10C0 –10C1x + 10C2x2+…
3(−1) − 1(3) −6 …...+ 10C10 x10)
x= = =–3
3 −1 2 Equating the coefficients of x10, we get
10
x C5 (–1)5 =10C010C10 –10C1 10C9 + 10C2 10C8 +…
17.[D] tangents = y = ± + RT ….+ 10C10 10C0
3
⇒ – 10C5 = (10C0)2 – (10C1)2 + (10C2)2 +……
RT
tan30º = ⇒ RT = 3 …+ (10C10)2
3

XtraEdge for IIT-JEE 91 APRIL 2010


5 5.[D] Net force acting on container due to liquid
(D) 95
C4 + ∑
j= 0
100 − j
C3 coming out from the holes is given by
 3H H
F = ρA 2g × − 2g ×  = ρgAH towads left
= C4 + 99C3 + 98C3 + 97C3 + 96C3 +95C3
95
 4 4
= (95C4 + 95C3) +96C3 + 97C3 + 98C3+ 99C3 ∴ F = f = ρgAH towards right.
= (96C4 + 96C3) + 97C3 +98C3 +99C3 H
Now, τF = ρgAH × into the plane of paper.
= (97C4 +97C3) +98C3 + 99C3 2
= (98C4 + 98C3) + 99C3 H
τf = ρgAH × out of plane of papers
= 99C4 + 99C3 2
∴ τF = τf hence τN = 0
= 100C4
20. [A] → p,q; [B] → q,t; [C] → q; [D] → s PA VA P V
(A) Given lines intersect if 6.[C] TA = and TB = B B
nAR n BR
2 −1 3 − 4 4 − 5
Given, PA = PB , VA = VB and nA = 2nB
1 1 λ =0 T
∴ TA = B
λ 2 1 2
⇒ λ = 0, – 1 V TA M B
Now, A = × =2
 x +1  VB TB M A
 1− 
(B) lim 4x  tan −1 x + 2  =2 = y2 + 4y + 5
x →∞  x +1  7.[A] Equivalent circuit
 1+ 
 x+2  C1V0

⇒ y = –1, – 3
(C) y2 – ax (– x – y) = 0 ε1
11' 33' 2, 2'
⇒ for perpendicular lines a + b = 0 1 1' 2' ε2 C2V0
⇒ 1+a=0 ⇒ a=–1 2 3 3'

(D) ( a × b ) × a = ( ĵ – k̂ ) × a
Middle plate V0
⇒ (a .a )b – (a .b ) a = ( ĵ – k̂ ) ×a
on solving, ε ε A ε ε A 
we get b = î Total charge on 2 & 2' plate =  1 0 + 2 0  V
 d1 d2 
ε ε 
PHYSICS σ = ε0V  1 + 2 
d
 1 d 2
8.[A]
2
1.[B] 2
T = m1r1ω1 also T = m2 g + ω22 r22 A
C
∴ m1r1ω12 = m2 g 2 + (ω22 r2 ) 2 30º 15º
AI
1
or, 0.1 × ω12 = (10) 2 + (10) 2 15
2 B
2
O 30
0.1 ω1 = 1
ω1 = 10 rad/s v1 = r1ω1 = 10 m/s
2.[A] As f = µN = mg
g C
or, µmlω2 = mg ⇒ ω= 45º
µl

3.[C] At terminal velocity net force is zero.


4 4 l
6πη(r1 + r2) VT + π (r13 + r23) ρg = π (r13 + r23) σg 120º
3 3 30º
B
4.[C] As supporting plane is lowered slowly O R
∴ N = mg – kx Sine rule

XtraEdge for IIT-JEE 92 APRIL 2010


R l as d increase
=
sin 45º sin 30º C decrease ∴ U decrease
R sin 45º option (C) is correct.
= = 2
l sin 30º
13.[C] As springs are in parallel
9.[A,B] As aT = aN F (k + k 2 ) x
∴ a = net = 1
vdv − v2 mass (m1 + m 2 )
∴ = which can also
ds R
14.[A] Frictional force on m2 will act in direction of
dv v2
be written as =– displacement if
dt R k2x > m2 a
Integrating the above equations answer is
obtained. 15.[A] as k2A – µm2g = m2amax
L L  k + k2 
10.[A,C] (A) ρBLAg = ρ × Ag + 2ρ × Ag ∴ k2A – µm2g = m2  1 A
4 4  m1 + m 2 
L 3L Solve to get answer. ]
(B) FB = ρ × Ag + 2ρ × Ag
4 4 2
7 F − mg 1 l Bωl 2
FB = ρLAg; a = B 16.[C] Induced emf across OP = Bω  =
2 2 8
4 m
11.[B,D] Bωl 2
(i) current = … (i)
a 8R
l/2
← eq. ckt. Bil 2
8V 10
Torque on the rod = 2 ∫
0
Bi x dx =
4
…(ii)

2
6V Ml dω B ωl 4 2
∴ × =− [substituting τ = I α]
• c 12 dt 32R
3Ω b ω t
dω 3B 2l 2
13V – ∫
ω0
ω
=
8RM ∫ dt
0
11
Using Kirchoff's Law Solve the circuit. Solving this eq. & eq. (i)
12.[A,B,C] Bω0l 2 –αt
i= e
CV –CV 8R

17.[B] θ= ∫ i dt
0

1 2
V 18.[A] Heat generated = Iω0
2

If battery is disconnected and plate are pulled Column Matching


apart, then charge will remain constant 19. [A] → p; [B] → p; [C] → r; [D] → q
Q Q As cube is floating ρsALg = ρLAxg
E= ×2=
2A ∈0 Α ∈0 ρ 
∴ x =  s  L
∴ E remain same (A) is correct work is done  ρL 
against attractive force
+ −
Felc.
←
Fext
→ by Fext. (B) is correct. 20. [A] → r; [B] → p; [C] → s; [D] → q
1
1 S= × 2 × 16 = 16 m
U= CV2 2
2 | Wg | = mg S =
V = constant [as battery is connected] WN = m(g + a) cos2θ. S
∈ A Wf = m(g + a) sin2θ . S
C= 0
d

XtraEdge for IIT-JEE 93 APRIL 2010


XtraEdge for IIT-JEE 94 APRIL 2010
SOLUTION FOR MOCK TEST
PAPER
IIT-JEE - II - II)
(PAPER
Grignard reagent reacting with acyl halide usually
CHEMISTRY gives 3º alcohol.
7.[B,C,D] The order of H-bond energies
1.[C] …….. – …….. ……..
F–H F >F–H O>F–H F>
NOBr …….. …….. ……..
O–H O>O–H F>N–H N
+ 8.[C,D]
NH2 NH2
Br 9.[C,D]
Br – RHE reaction : Hg2Cl2(s) + 2e → 2Hg(l) + 2Cl–
+ Enantiomer
LHE reaction : 2Ag(s) + 2Cl– → 2AgCl(s) + 2e
Net reaction : Hg2Cl2 (s) + 2Ag(s)
COOH COOH
CH2=C H2/Pd CH3–CH (resolvable) → 2Hg (l) + 2AgCl(s)
2.[B] Ph Ph In case of same concentration of Cl– ions in the
'A' two half cells, Ecell is independent on the
CH=CH–COOH H2/Pd concentration of Cl–. Other substances are either
pure solids or liquids, which have unit activities
'B' irrespective of their amounts.
CH2–CH2–COOH
Column Matching :
Optically inacitve
10. [A] → p, r, s ; [B] → q;
3.[B] Distribution of electrons in the MO's in He2 is [C] → p, q,,s ; [D] → q
σ1s2 σ∗1s2 . He2 is unstable
Distribution of electrons in the MOS in H2 is σ1s2 11. [A] → p, q,t ; [B] → p, r, s;
. H2 is stable. [C] → p, q, r; [D] → p, q, r

– Numerical Response type questions :


4.[B] After mixing total moles of A
–3 –3 12. [4]
= 100 × 0.2 × 10 + 100 × 0.3 × 10
–3 W Q 2.977 3 × 1× 60 × 60
= 100 × 10 × 0.5 moles = or =
After mixing total moles of HA E 96500 106.4 / n 96500
–3
= 100 × 0.1 × 10 + 100 × 0.2 × 10
–3 ∴ n=4
–3
= 100 × 0.3 × 10 moles 13. [8]
5
After mixing resulting pH = 5 + log
3 NH 2 NH 2
O KOH
 →
/ glycol
NH2 ( Wolff − Kishner
HO CH2–CH* reduction )
5.[A,B,C] COOH (W)
HO
O
It contains 2 phenolic hydrogens and a carboxylic HO
acidic hydrogens O /H O
+ 
3

2

NH2 ( Ozonolysis)
HO CH2–HC
COO– HO
HO Zwitter ion O
(X)
6.[A,B,C]

XtraEdge for IIT-JEE 95 APRIL 2010


Ca ( OH )
 2 → O
∆ MATHEMATICS
(Y) 1. [A] We have
O n 2n n
C 6 H 5COOOH
   →
( Baeyer − Villiger
∑r =1
(2r − 1) 4 = ∑r =1
r4 − ∑ ( 2r )
r =1
4
= f(2n) – 16f(n)
oxidation ) O
(Z) 2. [D] Using De Moivre's theorem
2 2
14. [1] fr (α) = e iα / r e 2iα / r …. e iα / r
2
Molecular weight of starch = 162n = e ( iα / r )(1+ 2 +......+ r )

∴ moles of C6H12O6 to be produced = 1 mol 2


= e (iα / r )[ r ( r +1) / 2] = e (iα / 2)(1+1/ r )
3000
∴ moles of ATP required = = 100 mol ∴ lim f n (π) = lim e iπ / 2(1+1/ n )
30 n →∞ n →∞

15. [7] π π


= eiπ/2 = cos   + i sin   = i
0
P −P 2
  2
= x1 = mole fraction of solute
P0
x1 = 0.0125 α + βt
3. [D] z= ⇒ ( γ + δt) z = α + βt
 1  1000 γ + δt
 −1 = ⇒ (δz – β)t = α – γz
 x1  m × 18
α − γz
∴ m = 0.70 ⇒ t= [Q αδ – βγ ≠ 0 ]
δz − β
16. [5] α − γz α − γz
Eq. of metal = Eq. of hydrogen As t is real, =
δz − β δ z − β
0.1 43.9
= ⇒ X=2 ⇒ (α – γz)( δ z – β ) = ( α – γ z )(δz – β)
51 / X 11200
Now eq. of metal = eq. of KMnO4 ⇒ ( γ δ – γ δ )z z +(γ β – α δ)z + (α δ – β γ ) z
0.1 0.1× 58.8 = (α β – α β) ...(1)
=
51 / Y − 2 1000 γ γ γ
∴ Y=5 Since is real, = or γ δ – δ γ = 0
δ δ δ
17. [7] Therefore (1) can be written as a z + a z = c ...(2)
For the first bulb p1v1 = n1RT where a = i(α δ – β γ ) and c = i( α β – α β )
& second bulb p2v2 = n2RT
Note that a ≠ 0 for if a = 0 then
or p1v1 + p2v2 = (n1 + n2) RT …(1)
Suppose equilibrium pressure at each bulb = p atm α γ γ γ
aδ –βγ =0⇒ = = [Q is real]
Then p(v1 + v2) = (n1 + n2)RT …(2) β δ δ δ
From eq. (1) & (2) ⇒ αδ – βγ = 0,
p1v1 + p2v2 = p(v1 + v2) which is against hypothesis.
9 × 5 + 6 × 10 = p × 15 Also, note that c = i( α β – α β ) is a purely real
p = 105/15 = 7 number.
18. [2] α + βt
Thus, z = represents a straight line.
•• O γ + δt
F F
Xe dy d2y
Xe 4. [C] = (x – 1)(x – 2)2 so = (x – 2) × (3x – 4).
O O dx dx 2
F •• F
O d2y
The points of inflection are given by =0
XeO3 XeOF4 dx 2
19. [6] So x = 2, x = 4/3 are points of inflection.
Each has two geometrical isomers and two optical
isomers (shown by cis-isomer).

XtraEdge for IIT-JEE 96 APRIL 2010


5.[A,B,D] We have adj A = |A| A–1 Column Matching :
adj (AB) = |AB| (AB)–1 = |A| |B| (AB)–1
= |A| |B| (B–1 A–1) 10. [A] → p, r; [B] → p,q, r; [C] → t ; [D] → p, q, r
= ( |B| B–1) (|A| A–1) 3 π
= (adj B) (adj A) sin θ = = sin
2 3
6.[A,B,C] We have π π
⇒ θ = nπ + (–1)n = 2nπ +
cos α − sin α 0  3 3
A(α, β)′ =  sin α cos α 0  4 sin θ cos θ – 2 sinθ – 2 3 cos θ + 3 = 0
 0 0 e β  ⇒ (2 sin θ – 3 ) (2 cos θ – 1) = 0
 cos(−α) sin( −α ) 0  3
⇒ sin θ =
= − sin( −α) cos(−α) 0  2
 0 0 e β  π 1
⇒ θ = nπ + (–1)n , cosθ =
= A (–α, β) 3 2
Also, A (α, β) A( – α, –β) π
⇒ θ = 2nπ ±
 cos α sin α 0  cos α − sin α 0  3
= − sin α cos α 0   sin α cos α 0  = I (C) sin 2θ + cos 2θ + 4 sin θ = 1 + 4 cos θ
⇒ 2 sin θ cos θ + 1 – 2 sin2θ + 4 sin θ = 1+ 4cos θ
 0 0 e β   0 0 e −β 
⇒ 2 sin θ (cos θ– sin θ) – 4(cosθ– sinθ) = 0
⇒ A(α, β)–1 = A(–α, –β) ⇒ (2 sin θ –4) (cos θ – sin θ) = 0
Next, adj A(α, β) = |A (α, β)| A (α, β)–1 ⇒ sin θ = 2 or sin θ = cos θ
= eβ A(– α, – β). ⇒ tan θ = 1 ⇒ θ = nπ + π/4, n ∈ I
7.[A,B,C,D] π π
n
(D) cos2 θ = 1/4= cos2 ⇒ θ = 2nπ ±
 5  3 3
Let (r + 1) th term of  2 + x 4  be
x  11. [A] → r ; [B] → r ; [C] → p ; [D] → q
independent of x. We have (A)We have b – a = c – b and (c – b)2 = a(b – a)
n −r ⇒ (b – a)2 = a(b – a) ⇒ b = 2a and so c = 3a.
 5  Thus a : b : c = 1 : 2 : 3
Tr + 1 = nCr  2  (x4)r
x  a+b
n n – r 6r – 2n (B) If the numbers are a and b, then x = and
= Cr 5 x 2
For this term to be independent of x, 6r – 2n = 0 1
or n= 3r. As each of 18, 21, 27 and 99 is divisible 3  b 3
b = ar ⇒ r =  
by 3, each of this can be a possible value of n. a
8.[A,B] Now,
Let k = 2n + 1, then 2n + 1Cr is maximum when y3 + z3 a 3r 3 + a 3r 6 a (1 + r 3 ) a+b
r = n. Also 2n+1Cn = 2n + 1Cn+1 = = = =2
xyz 2
x (ar ) (ar ) x a+b
Thus, kCr is maximum when
2
1 1
r = (k – 1) or r = (k + 1). (C) c > 4b –3a ⇒ ar2 + > 4ar –3a
2 2 ⇒ r2 – 4r + 3 > 0 ⇒ r < 1 or r > 3, But the terms
9.[A,B,C,D] are positive so r ∈ (0, 1) ∪ (3, ∞)
As a912, a951 and a480 are divisible by 3, none of  1   2 
them is prime. For a91, we have (D) tan–1  2  = tan–1  2 
 2r   4r 
1 1
a91 = (99....9) = (1091 – 1) (2r + 1) − (2r − 1)
9 14 2 43 9 = tan–1
91 times 1 + (2r + 1) (2r − 1)
1 = tan–1 (2r +1) –tan–1(2r–1)
= [(107)13 – 1] n
9 −1  1 
 (10 7 )13 − 1  10 7 − 1 
∴ ∑ tan
r =1
 2
 2r
–1
 (2n+1) – tan (1)

=    
 10 − 1   10 − 1 
7
π
= tan–1 (2n + 1) –
= [(107)12 + (107)11 + ….+ 107 + 1] 4
× [106 + 105 + ….. + 10 + 1]
= a91 is not prime.

XtraEdge for IIT-JEE 97 APRIL 2010


 n Hence, required number is 2.
 1  (2n + 1) − 1 n

∴ tan  tan −1  2
 r =1  2r
 = =
 1 + (2n + 1).1 n + 1 15. [1] We have
E = 2n + 1C1 + 2n + 1C2 + … + 2n + 1Cn – 2n + 1C0
 n  1  n
n →∞ ∑
∴ lim tan  tan −1  2
 r =1  2r
 = lim
 n → ∞ n +1
=1 – 2n + 1C1 – … – 2n + 1Cn
[using nCr = nCn – r]
= – 2n + 1C0 = – 1.
Numerical Response type questions :
12. [3] ∴ |E| = 1
We have 16. [3] The given system of equations will have a non-
9 trivial solution if
= (2 + cos θ)2 + sin2 θ = 5 + 4 cos θ (1)
| z |2 a + 2t b c
and ∆= b c + 2t a =0
3 3 c a b + 2t
+ = 4 + 2 cos θ (2)
z z
Clearly ∆ is a cubic polynomial in t and has 3 roots.
Eliminating θ from (1) and (2), we get
9 1 1 17. [3] None that it is not given that f is a differentiable
– 6 +  = – 3
|z| 2
z z function We have
⇒ 3 = 2( z + z) – |z|2 f (4 + h ) − f (4)
f ′(4) = lim
h →0 h
13. [6]
Clearly x > 0 and x ≠ 1/5 = lim
( )
f ( 4 + h ) 2 − f (2 2 )
h →0 h
log 5 5 − log 5 x
log5x (5/x) = (4 + h ) 3 / 2 − 8 8[(1 + h / 4) 3 / 2 − 1]
log 5 5 + log 5 x = lim = lim
h →0 h h →0 h
Putting log5x = t, then equation becomes
1− t  3h  3 
t2 + =1 81 + + ...... − 1 8 h + .....
24  = lim  8
1+ t = lim  
⇔ t3 + t2 – 2t = 0 h →0 h h →0 h
⇔ t(t – 1) (t + 2) = 0
18. [2]
⇔ t = 0, 1, – 2
dy dy x
So integral roots are 1 and 5. 2x + 2y =0 ⇒ =–
dx dx y
14. [2]
Let the A.P. be a – d, a, a + d, a + 2d. Note that a dy 1
⇒ =–
and d must be integers. Also as this is an increasing dx (1, 3 ) 3
a + 2d is the largest. We have
a + 2d = (a – d)2 + a2 + (a + d)2 Therefore, the equation of the tangent at (1, 3 ) is
= 3a2 + 2d2
y – 3 = (–1/ 3 ) (x – 1)
⇒ 3a2 – a + 2d2 – 2d = 0
As a is real, and the point of intersection of this tangent with the
1 – 8(d2 – d) ≥ 0 x-axis is (4, 0). The equation of the normal at
1 (1, 3 ) is y – 3 = 3 (x – 1), and the point of
⇒ d2 – d – ≤ 0
8 intersection of this normal with the x-axis is (0, 0).
2
 1 3 Hence the required area is
⇒ d −  ≤
 2  8 1
.4 3=2 3
1 3 1 3 2
⇒ – ≤ d≤ +
2 2 2 2 2 2 19. [4] The two curves represent parabolas with
As d is an integer, d = 0, 1 vertices at (0, 0) and (3, 0). They intersect at (1, 1)
But d ≠ 0, therefore, d = 1. and (1, –1), so the required area is
Thus 3a2 – a = 0 ⇒ a = 0 or a = 1/3. area of OPMQO = 2 (area of OPMO).
As a is an integer, a = 0.

XtraEdge for IIT-JEE 98 APRIL 2010


(1, 1) λ 5.5 × 10 −7
tmin = = = 99.6 nm
P 4n 4 × 1.38
dq q
3.[D] I= ; q = it + a; V=
dt C
it + a
O M(3, 0) V=
C
∴ V is proportional to time
4.[A] Phase difference corresponding to y1 = –π/2 and
Q
that for y2 = + π/2
∴ Average intensity between y1 and y2
Fig. π/2
1 φ (π + 2)
 1 3 3 − x  = ∫ I max cos 2   dφ = Imax
= 2


∫ 0
x dx + ∫ 1 2
dx


π
−π / 2
2 2π

1  2
2 1 3 Hence required ratio = 1 + 
 3/ 2 1 2 3/ 2  2  π
=2 x − . (3 − x )
3 2 3  M.C.Q. Type questions :
 0 1
5.[A,C]
2  1 2 3 / 2  2 4 a
= 2  −  0 − . 2  = 2 +  = 4. dr
 3  2 3  3 3
r

PHYSICS l
Consider a cylindrical element of radius r,
1.[D] thickness dr. If dR is the resistance of this
1 element then
1′ ρ( r ) l
d/2 dR =
2 2πr dr
2′
3 Total resistance of the cylinder is given by
a
3′ 1 1 2π
∫ ∫ r dr
3
Cl = =
R total dR αl
0

1 1 2π a 4 
⇒ =  
1′ R total αl  4 
2  
3 Cl 2′ 1 2αl
3′ ⇒ =
(∈ A) × 2 ∈ A R total πa 4
=2× 0 =4. 0 = 4C
d d R total 2πα
⇒ =
l (πa 2 ) 2
 2m + 1  λ
2.[A] 2t =   2πα
 2 n ⇒ R=
A2
V
Since E = (in magnitude)
l
R 
⇒ E = I  total  (By Ohm's Law)
 l 
t n mgF2
2παI
⇒ E=
A2

6.[A,D] Consider only one plate as shown in figure.

XtraEdge for IIT-JEE 99 APRIL 2010


The field on both sides of plate is shown in figure. R = 120 × 7.5 = 900 MeV
Applying Ampere's Circuital Law to the contour P = 2 × 60 × 8.5 = 1020 MeV
C, we get ∆E = 120 MeV
L EXOTHERMIC
If it was D
B
X→Y+Z
R = 90 × 8.0 = 720 MeV
P = 60 × 8.5 + 30 × 5.0 = 660 MeV
B ∆E = – 60 MeV
C
ENDOTHERMIC
2BL = µ0 (jL)
µ j Column Matching :
B= 0
2
Superimposing, the field due to two plates we get 10. [A] → p ; [B] → q, r, s; [C] → p,s ; [D] → q,r
at P both fields cancel each other and at Q, they When cohesive forces are greater then adhesive
added to give B0 = µ0j forces shape of meniscus is concave from liquid
side and pressure is greater in concave side due to
7.[B,D] v < g(t) surface tension.
⇒ v < 2g {due to Lenz's Law} 11. [A] → r ; [B] → p, s; [C] → q ; [D] → s
1  dV  dT 1  dV  1
Also, s < gt2  = ⇒
2 (A) 
V T dT V =T
 P  P
⇒ s < 2g {due to Lenz's Law}
kT
(B) λ =
8.[B,C] 2 πd 2 ρ
R3 (C) Ideal gas law are valid at all temperatures.
(D) conceptual
R1 R0 R1
R2 Numerical Response type questions :
S C C
V0
12. [1]
V M
v12 α v'12
R 2R 3 VR 2 v1 α
⇒ R0 = and V0 = 30º
R2 + R3 R2 + R3 N
O
–v2
 30º
R 2R 3 
⇒ τ = C(R1 + R0) = C  R 1 +  →
 R 2 + R 3  In the figure v12 = velocity of ball w.r.t. wedge
⇒ q = q0 (1 – e–t/τ) →
before coolision, and v12
' = velocity of ball w.r.t.
⇒ q = CV0 (1 – e–t/τ)
wedge after collision, which must be in vergically
CVR 2
⇒ q= (1 – e–t/τ) upward direction as shown.
R2 + R3 → →
In elastic collision v12 and v12' will make equal
9.[C] If it was A angle (say α) with the normal to the plane.
Y → 2Z We can show that α = 30º
Reactant : R = 60 × 8.5 = 510 MeV ∴ ∠MON = 30º
Product : P = 2 × 30 × 5 = 300 MeV
v1 1
∆E = – 210 MeV Now = tan 30º = ≈1
ENDOTHERMIC v2 3
If it was B
W→X+Z 13. [5]
R = 120 × 7.5 = 900 MeV A
P = 90 × 8 + 30 × 5 = 870 MeV
∆E = – 30 MeV h B
ENDOTHERMIC
h
If it was C
C
W → 2Y

XtraEdge for IIT-JEE 100 APRIL 2010


KT 16. [1]
For a cylinder = 2 in case of
KR π Pr 4 π Pr 4 π Pr 4
V1 = , V2 = , V3 =
pure rolling upto B : 8ηl1 8ηl 2 8ηl 3
2 π Pr 4
KT = mgh and V=
3 8ηl
1 Now V = V1 + V2 + V3
and KR = mgh
3 Substituting the values, we get
After B : rotational kinetic energy is constant l1l 2 l 3
however translational kinetic energy increases. l=
l1l 2 + l1l 3 + l 2 l 3
2 5
At C : KT = mgh + mgh = mgh 1× 2 × 3 6
3 3 = = m
1× 2 + 1× 3 + 2 × 3 11
1
while KR = mgh 17. [1]
3
π 3π
KT At x1 = and x2 =
∴ =5 3k 2k
KR Sin k x1 or sin k x2 is not zero.
14. [2] Therefore, neither of x1 or x2 is a node
According to the formula, the position of the  3 1  π 7π
∆x = x2 – x1 =  −  =
particle is at the centre of the path. It goes by 7 units  2 3  k 6k
to the right and then by another 7 units back to the
initial position and then it goes to the negative side 2π π
Since > ∆x >
by 7/2 units for the first time. k k
T λ  2π 
So the minimum time is 2 × plus the additional λ > ∆x > k = 
4 2  k 
time (t) to cover 7/2 units on the negative side. Therefore, φ1 = π
π 2π π 7π
Here ω = or = or T = 4s and φ2 = k.∆x =
2 T 2 6
4 φ1 6
∴ tmin = 2 × + t = 2 + t ∴ = ≈1
4 φ2 7
7 π π π 1
Now = 7sin t ⇒ t = sin 30º =sin ⇒t = s 18. [1]
2 2 2 6 3
With an AC source, current in the circuit is
1 maximum when
∴ tmin = 2 + = 2.33s ≈ 2
3 Z = Zmin = R (the resistance of coil)
24
∴ R= = 4Ω
15. [6] Let ω be the actual angular velocity of the 6
satellite from east to west and ωe be the angular When connected with an dc source of emf 12V.
speed of the earth (west to east). 12
Then ωrelative = ω – (–we) = ω + ωe ⇒ ω = ωrel – we i= (r internal resistance of source)
r+R
By the dynamics of circular motion
12
GMm 2 2 gR e2 = = 1.5 A
2
= mω R or ω = 3
(Q GM =gRe2) 4+4
R R
19. [1] For incident electron
gR e2 gR e2 h h h
⇒ ω= ∴ ωrel = + ωe λ1 = = =
R3 R3 p 2Km 2mVe
10 × 6.4 2 × 1012 hc
⇒ ωrel = + 7.27 × 10–5 and shortest wavelength of X-rays is λ2 =
2 3 × 10 21 Ve
2π λ1 1  V  e 
= 7.27 × 10–5)
(Q ωe = ∴ =   
86400 λ2 c  2  m 
⇒ ωrel =22.6×10–5 +7.27×10–5 = 30×10–5 rad s–1
λ1
2π 2π Substituting the values, we get =1
∴ τ= = = 2.09×104 s = 5hr 48 min. λ2
w rel 30 × 10 −5

XtraEdge for IIT-JEE 101 APRIL 2010


SOLUTION FOR MOCK TEST
PAPER
AIEEE- II
12.[B]
PHYSICS 13.[B]

1.[C]

2.[B] v ∝ n ∝ T because λ = constant


F
N+4 324 18 f f
= =
N +1 289 17
d
17N + 68 = 18N + 18
It is only possible if object and image coincide.
50 = N
φ
3.[B] 14.[D] I = I 0 cos2
2
4.[A] I0 φ
= I 0 cos2
5.[B] u=0 2 2
a = constant φ 1
cos =
v0 = u + at = at 2 2
 v + v0  ant 2π
n' =  n=n+
 φ = 90° = ∆x
 v  v λ
π 2π
6.[C] LC = 1 MSD – 1VSD = ∆x
2 λ
 29  1
LC = 1 −  MSD = MSD λ yd λ
 30  30 ∆x = ⇒ =
4 D 4
720 MSD = 360°
360° λ D 5 ×10 −7 ×1 5
1 MSD = = 1/2° y= = = × 10–3
4d − 4 4
720 4 ×1×10
1 1° 1° 60' y = 1.25 mm
So LC = × = = = 1' → → →
30 2 60 60 → →
15.[C] τ = p × E = q (2 a ) × E

7.[C] Here 2 a = (2 – 1) î + (– 1–0) ĵ + (5 – 4) k̂

8.[D] Ticker timer is a better device than a stop watch. = î – ĵ + k̂



M1 + M 2 10.30 + 12.62 E = 0.20 î V/cm = 20 î V/m
9.[C] M= = = 11.46 g
10.[B]
2 2


τ = (4 × 10–6) [(î – ĵ + k̂ )× 20 î ] = 8 × 10 –5

11.[B]
∆Y
=
2∆ D
+
∆l ( )
k̂ + ˆj
Y D l
Magnitude of torque τ = 8 2 × 10–5 N-m
∆Y  0.01   0.05  16.[D]
=2   +  
Y  0.4   0.8  C +σ

= 2× 0.025 + 0.0625 –σ
B
∆Y + σ
= 0.05 + 0.0625 = 0.1125 A a
Y
∆Y = 2× 1011 × 0. 1125 = 0.225 × 1011 c b
So (2 ± 0.2) × 1011 N/m2

XtraEdge for IIT-JEE 102 APRIL 2010


Potential of shell A is, ρ LA / AA
RA L A
∴ = = A × C
1  4 π a 2 σ − 4π b 2 σ + 4π c 2 σ  RC ρ L c /A c LC AA
VA =
4π∈o  a b c 

× A/ =
LA A 2 1
σ =
= (a – b + c) 2L A AA 4
∈0
20.[A] Current flowing through potential wire is –
Potential of shell C is,
E E
1  4π a 2 σ − 4 π b 2 σ + 4 π c 2 σ  I= =
VC =   15 r + r 16 r
4 π∈0  c c c 
 Potential drop across potential wire is,
σ  a 2 b 2  15 E
= − + c V = I × 15 r =
∈0  c c 

16
15
As VA = VC E
Potential gradient, K = 16
σ σ  a 2 − b 2 
∴ (a – b + c) = + c 600
∈0 ∈0  c c 
 E E 15E
∴ = Kl or = ×l
or a – b =
(a − b ) (a + b )
or a + b = c
2 2 16 × 600
c 16 × 600
17.[C] or l= = 320 cm.
5
15 × 2

4 21.[A]
B D i C
3
2 A
O φ2
1 Q b
φ2
1 2 φ1 φ1

A
P B
3 5 4 a
2
≡ A B
µ0 i
BAB = BCD = (sin φ1 + sin φ1)
4π (b / 2)
µ0 4 i a
3 4 = .
4π b a + b2
2
∈0 A
The capacity of each capacitor is, C0 = µ0 4 i b
d BBC = BDA = . .
4π a a 2 + b2
5 5 ∈o A
From fig. it is clear that Ceq = C0 =
3 3 d ∴ B = BAB + BBC + BCD + BDA
µ 4i a b a b
18.[B] Resistivity of conductors increases with = 0 .  + + + 
increase in temperature because rate of 4π a 2 + b2  b a b a 
collisions between free electrones and ions
2 2
increases with increase of temperature. µ0 8i a + b
However, the resistivity of semiconductors = .
4π ab
decreases with increase in temperature because
more and more covalent bonds are broken at 22.[A] In tan A position,
higher temperatures. µ0 2 M B
= BH tan 30° = H …….(1)
19.[B]let LA and LB be length of parts A and B 4π d 3
3
R L Magnetic moment of second magnet,
Then A = A [as cross-section is same]
RB LB M' = (3m)(2 × 2l) = 6M
Now Lc = 2 LA and (volume)c = (volume)P In tan B position,
i.e.Lc × Ac = 2 LA × Ac = LA × AA µ0 6 M
= BH tanθ …… (2)
where Ac = AA are cross-sectional area of part C 4π d 3
and A. dividing eq. (2) by (1) we get
∴ Ac = AA/2

XtraEdge for IIT-JEE 103 APRIL 2010


6 tan θ Now,
= or tan θ = 3 or θ = 60° W B → C = mg R + 0.3 mgR
2 1/ 3
= 1.3 mgR
µ 0 N 1 i1
23.[B] Magnetic field of solenoid, B1 = dm dv
l 30.[C] ×u=M
dt dt
Magnet flux of coil, φ2 = N2 B1 A2 = N2
∆v
 µ 0 N 1 i1  0.5 × 400 = 2000 × ⇒ ∆v = 0.5 ms–1
  A2 5
 l 
φ µ N N A
As φ2 = M i1, so M = 2 = 0 1 2 2
i1 l CHEMISTRY
di1
∴ induced emf, |e| = M 31.[C] HCO3– can donate a proton to CO32– and it can
dt
accept a proton to form H2CO3.
µ 0 N1 N 2 A 2 di
or |e| = × 1
l dt 32.[C] O F2 ⇒ O = +2
4π ×10 –7 × 2000 × 300 × 1.2 ×10 −3 4 O2 F2 ⇒ O = +1
= ×
0.30 0.25 O2 [PtF6] ⇒ O2+ + [PtF6]–1
= 4.8 × 10–2 Volt O2+ ⇒ 2x = +1
x = +1/2
24.[A] Z= R 2 + (X L − X C )2
33.[A] ∆G = ∆H –T ∆S
Here XL = 2πfL = 2 × 3.14 × 500 × (8.1 × 10–3)
if ∆H = ⊕ & ∆S = ⊕
= 25.4 Ω
& T ∆S > ∆H than
1 1
and XC = = ∆G = Θ & process is spontaneous.
2πf C 2 × 3.14 × 500 ×12.5 ×10 −6
= 25.4 Ω 34.[C] Calcined Gypsum is calcium sulphate
∴ Z= (10)2 + (25.4 − 25.4)2 = 10 Ω 35.[A] Inter particles forces between CH3COCH3 &
E 100 CHCl3 are strong H-bonding. Thus solution
Now irms = rms = = 10 A shows negative deviation. ∆Vmixing = Negative.
Z 10
∴ VR = irms × R = 10 × 10 36.[C] Addition of inert gas at constant volume does
= 100 V not cause any effect on the equilibrium.
25.[D]
37.[A] For I order Reaction t1/2 is constant
u 2 sin 2θ 2h
26.[C] = u 10gm 
g g  t1 / 2 = 24
↓ 
(14) 2 sin 2 (45) 2 (h )
= 14 ⇒ h = 10 m 5g 
9.8 9.8  t1 / 2 = 24
↓  = 96 hours.
27.[B] Loss in P.E. = gain in K.E.
2.5g 
1 
mg r = mv2 ⇒ v2 = 2g r ↓
2  t1 / 2 = 24
1.25g 
v2
ac = = 2g
r ↓ 
 t1 / 2 = 24
m v2 0.625g 
T – mg cos θ = ⇒ T = 3 mg
r
28.[D] Eq. wt. of Ag wt. of Ag
38.[D] =
Eq. wt. of O 2 wt. of O 2
29.[C] Loss in P.E. =
gain in K.E. + work done against friction 108 W
=
1 8 1.6
mg R = m (1.4 gR) + Wf WAg = 21.6 gm,
2
Wf = 0.3 mgR

XtraEdge for IIT-JEE 104 APRIL 2010


39.[B] Solution is decinormal, that is N/10 & x factor 0.059  Zn +2 
is 1, so conc. = 0.1M Ecell = E°cell – log  + 2 
n  Cu 
[H+] = c.α  
= 0.1 ×1.3/100 = 13×10–4 So doubling the conc. of ions.
pH = –log [13×10–4] = 2.89 Ecell remains same.

40.[A] The no. of atoms in fcc lattice = 4 51.[A] pKa = –log Ka = –log (1.8×10–5) = 4.7447
a = 400 pm = 4×10–10 m 12 mol
[CH3COOH] = = 0.4
= 4×10–8cm 60 × 0.5 L
n×M 4 × 60 16.4 mol
d= 3
= [CH3COONa] = = 0.4
No × a 6 × 10 × (4 × 10 –8 ) 3
23
82 × 0.5 L
d = 6.23 g/cm3.  salt 
Now, pH = pKa + log  acid 
41.[B] For any Value of l possible values of m are  
m = –l to + l  0.4 
l = 2, m = –2, –1, 0 +1, +2 = 4.7447 + log  0.4  = 4.7447
 
So option is (B)
Increase = 1
42.[B] C : O : H
6gm : 3.01 × 1023 atoms : 2 mole 52.[B] FeC2O4 → Fe+3 + CO2
Ratio ½ : ½ : 2 +2 +3 +3 +4
of mole
1 : 1 : 4 Increase = 1
COH4 or CH4O Total Increase in O.N. = 3
So valence factor of FeC2O4 = 3
3 KMnO4 →Mn+2
2 +7 +2 v.f. (KMnO4) = 5
43.[B] gm E KMnO4 = gm E of FeC2O4
Mole × v.f. = Mole × v.f.
1 1× 5 = x × 3 ⇒ x = 5/3
44.[B] NaNO2/HCl gives HNO2 which gives different
53.[A] ∆G = ∆H – T ∆S
products with Pri. and Sec. amines.
∆G = Θ , ∆G < 0 (Spontaneous process)
CH3 ∆G = ∆H – T ∆S
hν CH2Cl +HCl
45. [A] + Cl2 
→ = ∆E + P ∆V – T ∆S
(∆G)E,V = 0 + 0 – T ∆S
(∆S)E,V = ⊕ ⇒ ∆G = Θ (Spontaneous process)
46.[D] 2 CuSO4 + 2 KCN → Cu(CN)2 + K2SO4 54.[B]
2Cu(CN)2 → 2CuCN ↓ + (CN)2 ↑ CH3 CH3
3 KCN + CuCN → K3[Cu(CN)4] |
H + | +
H3C – C – CH – CH3  → CH3 – C – C H – CH3
47. [B] Blue print process occurs with the help of Iron | | – H 2O |
Compound. CH3 OH CH3
2° Carbocation
48.[A] Effective nuclear charge increases therefore
ionic radius follow the order.
1, 2-Methyl shift
49.[B] xy2 xy + y
t = 0 : 600 0 0
+
teqm : 600-P P P H 3C – C = C – CH 3 ←– H + H C – C – CH – CH
 3 3
Now : 600 – P + P + P = 800 | | | |
P = 200 mm Hg CH CH CH3 CH3
3 3
(Pxy)(Py) 200 × 200 3° Carbocation
Kp = = = 100 mm Hg
(Pxy 2 ) 400 (More stable)
+2 +2
50.[C] Cell reaction : Zn + Cu → Zn + Cu
Cell emf: 55.[C] Factual Q.

XtraEdge for IIT-JEE 105 APRIL 2010


56.[D] 64.[C] f(x) = sin x – cos x – kx + b
f '(x) = cos x + sin x – k
H C – Cl
OH Zndust
→ 
3
→ CH3
AlCl3
f '(x) = 2 sin(π/4 + x) – k
if f(x) is decreases for all x ∴ f '(x) is –ve
alk. KMnO 4
COOH i. e. k > max. of 2 sin(π/4 + x)

→
i. e. k > 2

57.[D]
lim  e x − cos x 
2

65.[A] x→0  
2
58.[A] 4 HCl + O2 → 2 H2O + 2 Cl2  x 
 
Chlorine is in the form of cloud.
lim  e − 1 1 − cos x 
x2
59.[A] Coordination no. = 6 = x→0  +
 x2 x 2 
Oxidation no. = 3 
no. of d electron = 6
lim 2 sin 2 x / 2 1 3
no. of Unpaired 1+ x→0 =1+ =
2
d electron = 0 x 2 2
60.[C] Resonance structure should have same number 66.[A] Second determinant has been obtained
of electron pairs. from the first by the operation
C1 → C1 + 2C2 – 3C3. so its value remains
MATHEMATICS unchanged
67.[C] given 3sin x – 4sin3x – k = 0
61.[A] ⇒ 3sin x – 4sin3x = k
P ⇒ sin3x = k
.........(i)
angle A and B satisfy the equation (i)
h
∴ sin 3A = sin3B = k ⇒ sin3A = sin3B
But A > B ⇒ A B
30º 60º Now, sin3A = sin(π – 3B)
A O 3A = π – 3B
Let the height of the tower is h π 2π
1000 = h cot 30º – h cot 60º A+B= ⇒C=
3 3
1000
⇒h=
cot 30º − cot 60º 68.[B] given statement
h = 500 3 m (p q) ∨ ~ r → (p ∧ r)
(F ↔ F) ∨ F → (F ∧ T)
62.[D] since variance is independent of change in T∨F→F
origin. Hence variance of observations 101, T→F=F
102, ...... 200 is same as variance of 151, 152,
.....250. 69.[C]
∴ VA = VB
p q p ∨ q ~ p q ∧ ~ p (p ∨ q ) ↔ q ∧ ~ p
V
⇒ A =1 T T T F F F
VB
T F T F F F
63.[C] centroid of triangle is given by F T T T T T
 c +3 a + b−3
2 F F F T F T
 , 
 3 3  Hence neither tautology nor contradiction
 
if centroid lie on y axis ⇒ abscissae = 0
c2 + 3 = 0 ⇒ no real c exist 70.[D] Given sequence can be written as
if centroid lie on x axis ⇒ ordinate = 0 5 20 10 20
, , , , ..............
⇒a+b–3=0 2 13 9 23
⇒a+b=3 20 20 20 20
or , , , ..........
8 13 18 23

XtraEdge for IIT-JEE 106 APRIL 2010


which is a H.P. AP 1
nth term of corresponding A.P. if =
PB 2
8 5 5n + 3  a 2b 
an = + (n – 1) = co-ordinates of point P given by  , 
20 20 20
3 3 
1 20
nth term of H.P. = =  3y 
2
an 5n + 3 locus of point P, a2 + b2 = (3x)2 +   = 36
 2 
20 5
also = x 2 y2
5n + 3 17 or + =1
17 4 16
5n + 3 = 20 × = 68
5 a2 3
b > a then eccentricity e = 1 − 2
=
65 b 2
n= = 13
5
74.[D] parabola y = x2 + 2px + p2 + 13 – p2
(y – (13 – p2)) = (x + p)2
71.[B] Let any point P divides line joining
vertex is given by (– p, 13 – p2)
A(–2, 4, 7) & (3, –5, 8) in ratio λ : 1
is 4 units above x-axis
 − 2 + 3λ 4 − 5λ 8λ + 7  ⇒ 13 – p2 = 4 ⇒ p = ± 3
then P  , , 
 λ +1 λ +1 λ −1  also lies in Ist quadrant ⇒ p < 0
if P lies on plane 2x – k = 0 ⇒p=–3
k
⇒x=
2 75.[C] If lines x2 + 2λx + 2y2 = 0
k −2 + 3λ & (1 + λ) x2 – 8xy + y2 = 0
= given λ = 9
2 λ +1 are equally inclined
⇒ Their bisectors eqn must be same
 25 
k=   ×2⇒k=5 x 2 − y 2 xy
 10  ⇒ eqn =
1− 2 λ
72.[C] |a × b| = |a| |b| sin θ x 2 − y2 xy
& = are same
8 4 (1 + λ ) − 1 − 4
sin θ = =
5.2 5 xy xy
⇒ x2 – y2 = & x2 – y2 = are same
3 −λ −4/λ
⇒ cos θ =
5 −4
⇒–λ=
|a – b|2 = |a|2 + |b|2 – 2a.b λ
= 4 + 25 – 2|a| |b| cos θ ⇒ λ2 = 4
3 ⇒λ=±2
= 29 – 2.2.5 = 29 – 12
5
|a – b| = 17 76.[D] given circle in standard form is
 1  1
x2 + y2 + 2  t +  x – 2  t −  y + 1 = 0
73.[C]  t  t
(0, b)
A  1 1
centre is given by  − t − , t − 
1  t t
P  1 1
or h = –  t +  & k = t –
2  t  t
2 2
 1  1
h2 – k2 =  t +  –  t −  = 4
B(a, 0)  t  t
2 2
given length AB = 6 locus x – y = 4 which is a hyperbola
or a2 + b2 = 36

XtraEdge for IIT-JEE 107 APRIL 2010


4
 cos 2 x  2
77.[C] ∫ f ( x ) dx = 4 = ∫ 1 −
 1+ x2 

 sec x dx

−1
2 4  1 
∫  sec
2
= x−  dx
⇒ ∫
−1
f ( x ) dx + ∫ f ( x ) dx = 4 1+ x2 
2
= tan x – tan–1x + c
2 4
∴ f(0) = 0 ⇒ tan0 –tan–10 + c = 0
⇒ ∫ f (x) dx = 4 – ∫ f (x) dx ...........(i)
−1
⇒c=0
2 ∴ f(x) = tan x – tan–1x
4
∴ f(1) = tan1 – tan–11
Q ∫ (3 – f(x)) dx = 7
2
(given) = tan1 – π/4
4 4 81.[B] we know x –[x] = {x}
⇒ ∫
2
3dx – ∫
2
f ( x ) dx = 7 ∴ Domain of {x} is R & {x} ∈ [0, 1)
but in f(x), {x} is in denominator & it should
4 not be equal to zero
∴ {x} 0 ⇒ x I
⇒ 3[x]42 = 7 + ∫ f (x) dx
2
and domain of sec–1x is R –(–1, 1)
4 4
∴ domain of f(x) is
R – (–1, 1) – I
⇒3 × 2 = 7 + ∫ f (x) dx ⇒ ∫ f (x) dx = – 1
2 2 82.[C] Q sin–1x is defined for |x| ≤ 1 and
.....(ii) sec–1x is defined for |x| ≥ 1 therefore
put this value from (ii) in (i), we get both defined for |x| = 1 ⇒ x = {1,–1}
2 ∴ Df = {–1, 1}
∫ f (x) dx = 4 – (–1) = 5 further f(–1) = sin–1 (–1) + sec–1 (–1)
−1 = – π/2 + π = π/2
−1 and f(1) = sin–1(1) + sec–1(1) = π/2 + 0 = π/2
⇒ ∫ f (x) dx = – 5
2
Hence Rf = {π/2}
83.[B] Let A(z1), B(z2) and C(z3) be the vertices of the
triangle then
78.[A] lines – 2x – y + 6 = 0 & 4x – 2y + 7 = 0
|z1| = |z2| = |z3|
(make c1 & c2 +ve)
now a1a2 + b1b2 = – 8 + 2 = – 6 < 0 ⇒ |OA| = |OB| = |OC|
bisector by +ve is acute and contains origin eqn O being the origin
⇒ O is circumcentre of the triangle, Also, the
−2 x − y + 6 ( 4 x − 2 y + 7)
given by =+ triangle is equilateral, therefore circumcentre
5 2 5 coincide with the centroid
– 4x – 2y + 12 = 4x – 2y + 7 ⇒ origin is centroid
8x = 5 z + z + z3
⇒ 1 2 =0
2
3
⇒ z1 + z2 + z3 = 0
∫ [f g(x)]
−1
79.[C] f 'g(x). g'(x) dx
1
84.[B] Total students n = 100
put f g(x) = t ⇒ f 'g(x) g'(x) dx = dt Average marks x = 72
∴ required integral Total boys n1 = 70
[log f g( x )]12 average marks of boys x 1 = 75
log (f g(2)) – log (f g(1)) = 0 Total girls n2 = 30
{Q g(1) = g(2)} n x +n x
now x = 1 1 1 2
n
x 2 + sin 2 x nx − n 1 x 1
80.[C] f(x) = ∫ 1+ x2
sec2x dx ∴ x2 =
n2
 x 2 + 1 − cos 2 x  100 × 72 − 70 × 75
= ∫ 

 1+ x 2
 sec2x dx


x2 =
30
= 65

XtraEdge for IIT-JEE 108 APRIL 2010


85.[C] Letters of word 'STATISTICS' are 88.[D] The given curves are y2 = 8x ...(1)
1A, 2I, 1C, 3S, 3T total = 10 and xy = – 1 ...(2)
Letters of word 'ASSISTANT' are any tangent to (1) is
2A, 1I, 1N, 3S, 2T total =9 2
y = mx +
common letters are A, I, S & T m
1 2 2 ...(3)
probability of choosing A is = × =
10 9 90 Q tan gent to y 2 = 4ax
 a
2 1 2
probability of choosing I = × = is y = mx +
10 9 90 ∴ 4a = 8 ⇒ m a=2

3 3 9
probability of choosing S = × = we shall find m so that it touches (2)
10 9 90
3 2 6 ∴ from (2) & (3)
probability of choosing T = × =
10 9 90  2
x  mx +  = −1 ⇒ m2x2 + 2x + m = 0 ...(4)
2 2 9 6 19  m 
total = + + + =
90 90 90 90 90 (3) touches (2) if quadratic (4) has equal roots
∴D=0
86.[D] A = { – 2, –1, 0, 1, 2} ⇒ n(A) = 5 ⇒ 4 – 4m3 = 0 ⇒ m3 = 1 ⇒ m = 1
B = {0, 1, 2, 3} ⇒ n(B) = 4 ∴ required common tangent is y = x + 2
C = {1, 2} ⇒ n(C) = 2
89.[B] h(x) = (f(x))2 + (g(x))2
D = {(1, 7),(2, 6),(3, 5),(4, 4),(5, 3)(6, 2),(7, 1),}
∴ h'(x) = 2f(x)f '(x) + 2g(x) g'(x)
⇒ n(D) = 7
= 2f(x)g(x) + 2g(x)g'(x) {Q f '(x) = g(x)}
(A ∪ B ∪ C) = {– 2, – 1, 0, 1, 2, 3}
Also f '(x) = g(x)
⇒ n(A ∪ B ∪ C ) = 6
∴ f"(x) = g'(x) ⇒ – f(x) = g'(x)
n(D) = 7
∴ h'(x) = 2f(x) g(x) – 2g(x)f(x)
n(B ∪ C) = 4
h'(x) = 0
87.[B] this is of the form ∴ h(x) = constant for all x
dy but h(5) = 11 ∴ h(x) = 1 for all x
f '(y) + f(y). f(x) = Q(x)
dx ∴ h(10) = 11
put tan y = z
90.[A] for m1 : Let I & N are assumed single letter in
dy dz IN order Now total no. of letter are IN T E G E
∴ sec2y =
dx dx R = 6 letter
dz these 6 letter can be arranged in row
∴ + 2xz = x3
dx 6
= × 2 = 720 ways
2
∴ e∫ = e∫
pdx 2 xdx 2
= ex
∴ Letter of words INTEGER can be arranged in

2 2
∴ soln is z e x = e x . x 3 dx 7
a row = = 2520 ways
2
1
∫e
x2
= . x 2 2x dx Now no. of ways in which IN are not together is
2
m1 = 2520 – 720 = 1800
1 1
∫ e .tdt = 2 e (t – 1) + c
t t
= Now for m2 : I- - - - - R → rest five can arrange
2 5
2 1 x2 2 = = 60 ways
∴ tan y e x = e (x – 1) + c 2
2
m1 1800
−x2 1 2 ∴ = = 30
⇒ tan y = c e + (x – 1) m2 60
2

XtraEdge for IIT-JEE 109 APRIL 2010


SOLUTION FOR MOCK TEST
PAPER - II
BIT-SAT
v + v0 ν´ v − v0
PHYSICS 6. [C] v´ =
v − vs
v We get
ν
=
v − vs
9 340 + v´
1.[B] In case of damped vibrations, amplitude i.e. = ∴ (v0 = vs = v´)
8 340 − v´
decreases exponentially with time
i.e. v´ = 20 ms–1
A
∴ A = A0e–bt or = e–bt
A0 I2 I2
7. [A] Ilog = dB; i.e. 4 = 10 log
1 A´ I1 (10 × 10 −9 × 10 4 )
or = eb×2 and = e–b×b
2 A0 Then I2 = 2.5 × 10–4 Wm–2
3
A´ 1 1 8. [A] Here work done = pdv and area under the curve
or = (e–2b)3 =   = given work done
A0 2 8
∴ 10 + WCA = 5 or WCA = –5 J
1 1 1 1
2.[C] Here = + + + ....... ∞ 9. [C] For equilibrium
ke k 2k 4k F = qE = mg
1 1 1 1  4 3  3neE 
1/ 3
=  + + + ......∞  or neE = πr ρg or r =  
k  1 2 4  3  4πρg 
1 1  2 k
=   = i.e. ke = 1.5 + 1.5 − 1.5
k  1−1/ 2  k 2 10.. [A] i = = 0.5 A
1+1+1
1 1 As the current has to from A to C to B,
3.[ B] KEmax = kA2 = Mω2A2 for kirchhoff's law,
2 2
VA = 0.5 × 1 + 1.5 = 1V (Q v = E – ir)
1
8 × 10–3 = × 1 × ω2 × (0.1)2 VB = 0.5 × 1 + 1.5 = 1V
2
VC = 0.5 × 1 – (–1.5) = 2V
ω = 4 rad/sec
y = A sin(ωt + φ) or y = 0.1 sin (4t + π/4) 11. [D] R = R1 + (273 – T) α ...(i)
or 2R = R0[1 + T´α] ...(ii)
→ → → →
4. [B] Here L = m ( r × v ) = m v y(– k ) 1 1(273 - T)α
Deciding (i) by (ii) =
(Where y is the vertical distance of particle from 2 1 + T´α
x axis) or 1 + T´α = 2 + (273 – T)2α

(273 - T)2α + 1
Here m,v and y all are fixed so L . remains or T´ =
constant. α
2E
5. [B] Using, weight of floating body = weight of 12. [A] Current, I =
R + r1 + r2
liquid displaced.
V V 2Er1
we get V ρ g =   (13.6) g – (0.8g) P.O. across cell 1 = Ir1 =
R + r1 + r2
2 2
(buoyant forces of mercury and oil act in For zero p.d. the fall of potential should be
opposite direction) equal to in emf.
13.6 − 0.8 2Er1
Then, ρ = = 6.4 E= i.e. R = r1 – r2
2 R + r1 + r2

XtraEdge for IIT-JEE 110 APRIL 2010


13. [B] Point P lies on the arms CD and AF so inclusion 18. [D] Reflection of light from plane mirror gives
at P due to them is zero.
additional path difference of λ/2 between two
Magnetic induction at p due to currents in AB waves.
and BC is given by
3λ π
µ 0i ∴ Total path difference = + = 2λ
B1 = B2 = sin 45º 2 2
4π(2a )
which satisfy the condition of maxima.
µ 0i Resultant intensity ∝ (A2 + A2) [Q I ∝ Α2]
= (Q distance of p from AB or BC is
8 2 πa 4Α2 = 4Ι
2a)
similarly due to DE and EF λ 2 ( λ 0 − λ1 ) 2
19. [B] Here =
µ i µ0i λ1 ( λ 0 − λ 2 ) 1
B3 = B4 = 0 sin 45º = ⊗
4πa 4 2 πa 5.4 (λ 0 − 3.5 × 10 −7 ) 2
or =
Net induction = 2(B1 – B3) 3.7 (λ 0 − 5.4 × 10 − 7 ) 1
µ 0i µ 0i 2µ 0 i or λ0 = 11.8 × 10–7 m
=2 – =–
8 2 πa 4 2 πa 8πa hc (6.6 × 10 −34 )(3 × 108 )
But ω = = = 1.05
2π λ 0 (11.8 × 10 − 7 )(1.6 × 10 −19 )
14. [C] v = rω = r × eV
T
2πr 2πm km 20. [D] Let whole the energy of electrons be converted
or T = = =
v qB q in x-rays. eV = hv
Now mα = 4mp and qα = 2qv hc
or eV =
mp m λ
Tp = k and Tα = k α
qα qα hc (6.6 × 10 −34 )(3 × 108 )
or λ = =
4m P m eV (1.0 × 10 −19 )(40 × 10 3 )
=k = 2k P
2q P qα i.e. λ = 3.1 × 10–11 m or λ = 0.31 Å
1 −
⇒ Tα = 2Tp or Tp = Tα α
21. [A] Here 88Ra222 → 218 β
→ 218
2 86R 87Fr

15. [D] i = i0(1 – e–t/τ) α
→ 85Al
214 β
→ 80Rn
214 α
→ 84PO
210

v  −t /
L  12  −t
8.4×10 −3 
 →α
82Pb
206

i= 1− e R  = 1− e 6
R  6  =1 4α deceys and 2β decays.
  
  π
22. [A] A1 = 4, A2 = 3 and θ = = 90º
(Q i = 1A given) 2
⇒ t = 0.97 × 10–3 s , i.e. t ≈ 1ms ∴ Resultant amplitude,
16. [A] Optical distance between fish and the bird is Α= A12 + A 22 + 2A1A 2 cos 90º = A12 + A 22
Differentiating w.r.t.
ds dy´ µdy = 4 2 + 32 = 25 = 5 unit
= +
dt dt dt 23. [C] Using d sin θ = nλ
4 dy λ
i.e. 9 = 3 + sin θ = θ =
3 dt D
dx 3 dy nD
or =6× = 4.5 ms–1 ∴ = nλ or y = λ
dt 4 D d
1× 2 × (6 × 10 −7 )
1  µg  1 1  i.e. = 1.2 × 10–3 = 1.2 mm
17. [A] =  −1  −  1× 10 −3
f  µm   R1 R 2  Distance between first minima on either side of
1  1.5   1 1 1 centrar maxima ∆y = 2y = 2.4 mm
= −1 ×  −  =
f  1.75   − R R  3.5R
24. [B] For constructive inteference
( ) =( )
i.e. f = 3.5 R. 2 2
In the medium it behaves as a convergent lens. Imax = I1 + I 2 2I + I

XtraEdge for IIT-JEE 111 APRIL 2010


For destuctive interference 1 2 2h
Imin = ( I1 − I 2 ) =(
2
2I − I )
2 30. [A] using h =
2
gt , we get t1 =
g
2 let t1 be the time taken from instants of jumping
I min ( 2I − I ) 2  2 − 1  1 to the opening of parachute, then
Then = = =
I max ( 2I + I ) 2   34
1+ 2  2× 40
t1 = = 2.86 sec
25. [B] Using decay equation 9.8
A2 = A1e–λt His velocity at this point is given by
A A v12 = 2gh1 = 2 × 9.8 × 40
or e–λt = 2 or λt = log 1
A1 A2 = 784 or v1 = 28 ms–1
for the remaining journey,
log A1 / A 2 A 
time t = = 1.44T loge  1  v = v1 + at2
λ  A2  v−u 2 − 28
or t2 = = = 13sec
(4πa 2 )σ k (4πb 2 )σ k (4πc 2 )σ a −2
26. [B] Vx = k – +
a b b ∴ total time = t1 + t2 = 2.86 + 13
1 = 15.86 ≅ 16 s
= 4π σ(a – b + c)
4πε 0 31. [B] Let u be the velocity of projectile w.r.t. tanks
 1  velocity v then
Q k =  ux = u cos 30 + v; uy = u sin 30º
 4πε 0 

2u sin 30º
σ and T =
= (a – b + c) g
ε0 2u sin 30º
27. [D] Let the given quantity be x1 then, Range, R1 = uxT = (u cos 30º + v)
g
π π for y axis
x = (a2 – b2)h = (a2h – b2h)
3 3 ux´ = u cos 30º – v and uy´ = u sin 30º
π 2 π 2 2u sin 30º
= a h– b h T=
3 3 g
Each term has the dimension of x1 then Range e, R2 = Tu´x
[x] = [a2h] = [L2L] = [L3] and also 2u sin 30º
[x] = [b2h] = [L2L] = [L3] quantity is volume. = (u cos 30º – v)
g
A x U( x 2 + B)
28. [B] U = or A = 4u 2
x2 + B x Then R1 + R2 = (sin 30º cos 30º)
g
2
Here dimensions of x and B should be same.
4u
i.e. [B] = [x2] = [L2] R1 – R2 = v sin 30º
g
 ML2 T −2 
Also [A] =  1/ 2  [L2] = [ML7/2T–2] Eliminating u we get
 L 
g (R 1 − R 2 ) 2
Then [AB] = [ML T ] [L2] = [ML11/2T–2
7/2 –2 v2 =
4 tan 30 (R 1 + R 2 )
 t dx  t
(250 − 200) 2
29. [A] v = u 1 −  or = u 1 −  10
 t´  dt  t´ = = 24 m2s–2
4 tan 30 (250 + 200)
 t 2  ⇒ 4.9 ms–1
integrating, x = u  t − +C
 2t´ 
  32. [D] Let α be the angle between velocities of pair of
at t = 0, n = 0 and c = 0 particles then relative velocity is given by
 t 2   t´  α
∴ x = u  t − = 10t  t −  vr = v 2 + v 2 − 2 v × v × cos α = 2 v sin
  10  2
 2t´ 

Putting t = 10 2v(sin α / 2) 4
 10 
average vr = ∫ 2π
dα =
π
v
x = 10 × 10 1 −  = 0
 10 
0
∫ dα
0

XtraEdge for IIT-JEE 112 APRIL 2010


33. [B] For quarter revolution 5
→ → → 25 – 20 = 2a or a = = 2.5 ms–2
∆ V = V2 – V1 2
→ → 37. [A] On cutting of string QR, the resultant force on
angle between V1 and V2 is 90º m1 remains zero because its weight mg is
N balanced by the tension is the spring but on
v1 block m2 a resultant upward Force (m1 – m2)g is
den sped. The block m1 will have no resultant
v2 acceleration where as m2 does have an upward
W a E (m1 − m 2 )g
acceleration given by
m2

–v1
1/ 2
S  2pt 
38. [A] Here v =  
∴ ∆v = v 22 + v12 = v2 + v2 = 2V  M 
1/ 2
v ds  2pt 
Also tan–1 = 45º or = 
v dt  M 
→ 1/ 2
∴ ∆v = 2 v south west  2pt 
or ds =   dt
 M 
34. [D] For vertical motion
1/ 2
1  2p  2 3/2
H = gt2 or t = 2H / g integrating s =   t +C
2 M 3
For horizontal motion, distance covered is given at t = 0, S = 0, so c = 0
by 1/ 2
 8p 
2πrn = ut S=   t3/2
 9M 
or 2πrn = u 2H / g
4 39. [B] Let a small displacement be given to the system
or u = 2H / g
2πr in vertical plane of frame such that ST remains
horizontal then let vertical displacement of
35. [A] On descending centres of rods up and QR be y then vertical
(Mg – f) – Ma = 0 displacment of centres of VT and RS will be 3y
(where f is the upthrust due to buoyancy) and that of TS will be 4y. Equating total vertical
On ascending, work to zero we get
f – (M – m)g – (M – m)a = 0
P Q
 2a  y
m =   M W
a+g y
W W W R y
36. [A] The masses will be lifted if the tension of the
string is more than the gravitational pull of y
W
mass. W
T S
50 N
W
(w + w)δy + (w + w)3 δy + w(4δy) – T(4δy) = 0
or 2w + 6w + 4w = 4T or T = 3w

40.[A] Normal reaction R = mg = 2 × 9.8 N


Frictional force,
F = µR = 0.2 × 2 × 9.8 = 3.92 N
Distance traveled
20 N 50 N
2 × 5 = 10 m
50 – 2T = 0 or T = 25N ∴ Work done = f × s = 3.92 × 10
So, 5 kg weight cannot be lifted but 2 kg weight = 39.2 J
will be lifted

XtraEdge for IIT-JEE 113 APRIL 2010


8.[C]
CHEMISTRY (+2)(+3) 3+ (+4)
FeC2O4 → Fe3+ + CO2
–1e
1.[D] Fe2O3 + 3CO → 2Fe + 3CO2 –2e
160 g 112 g ∴ Valence factor (FeC2O4) = 1 + 2 = 3
100 × 80 M
Pure Fe2O3 in ore = = 80 kg ∴ E( FeC2O4) =
10 3
Iron produced from 80 kg of Fe2O3
112 9.[D] The fractional of total volume occupied in simple
= × 80 = 56 kg 3
160 4 a
π 
h volume of particles 3 2 π
2.[D] ∆v = cube = = =
4πm(∆x ) volume of cube a3 6
6.626 × 10 –34
=
4(3.14)(9.11× 10 – 31 )(10 −10 × 0.1) 10.[A] Cu2+ + e– → Cu+ ; E10 = 0.15 V
= 5.79 × 106 ms–1 ∴ ∆G10 = –1 × 0.15 × F = – 0.15 F
3.[B] Given n = 0.5 Cu+ + e– → Cu ; E02 = 0.5 V
 n 2a  ∴ ∆G 02 = – 1 × 0.5 × F = – 0.5 F
Then  P + 2  (V – nb) = nRT
 V  Cu2+ + 2e– → Cu ; E 30 = ?
(0.5) 2 a ∴ ∆G 30 = – 2 × E 30 × F = – 2F E 30
⇒ [P + ] [V – 0.5b] = 0.5 RT
V2 Also, ∆G 30 = ∆G10 + ∆G 02
 a  = – 0.15 F + (– 0.5 F)
⇒ P +  (2V – b) = RT
 4V 2  = – 0.65 F
4.[B] N2O4(g) 2NO2(g) Now – 2 E 30 F = – 0.65 F
n0 : 1 0 0.65
ne : 1 – 0.5 2 × 0.5 or E 30 = = 0.325 F
2
= 0.5 1
0. 5 1 11.[A] 4hrs = 4 half lives
Ce : M
5 5 1
= 0.1 M = 0.2 M 1 1 1 1
→
t1/ 2
 →
t1/ 2
 t→
1/ 2 t1/ 2
→

[ NO2 ]2 (0.2) 2 0.04 2 4 8 16
then KC = = = = 0.4 4
[ N 2O 4 ] (0.1) 0 .1 1 1
fraction left after 4 half lives = or  
16 2
HClO < HClO 2 < HClO 3 < HClO 4 1 15
5.[A] → fraction reacted in 4 half lives = 1 – =
Acidic strength 16 16
Weakest acid has strongest conjugate base.
12.[B] A solution showing +ve deviation has higher
6.[C] HCl : NA × VA = (0.4 × 1) × 50 = 20 meq. vapour pressure and lower boiling point.
NaOH : NB × VB = (0.2) × 50 = 10 meq.
Q NAVA > NBVB 13.[C] In multi molecular solutions the different layers
N V − N B VB 20 − 10 hold each other through van der Waal's forces.
∴ [H+] = A A =
VA + VB 100 14.[A] CS2 + 3O2 → CO2 + 2SO2 ; ∆rH = ?
–1
= 0.1 M = 10 M, ∴ pH = 1 ∆rH° = Σ∆ f H 0( P ) – Σ∆f H (0R )
7.[D]
(–2) = [2(– 297) + (– 393)] – (117)
O = – 1104 kJ mol–1
(+1) (–1) (+1)
H – O – O – S(x) – O – H 15.[C] Aspirin chemically acetyl salicylic acid
(–1) (–2)
O OCOCH3
(–2)
COOH
2(+1) + 2 (– 1) + x + 3(–2) = 0
⇒x=+6

XtraEdge for IIT-JEE 114 APRIL 2010


NH2 O CH3
0 − 5° C 26.[A] C N
16.[A] + NaNO2 + 2HCl  
→ CH3
N,N-dimethyl cyclopropane carboxamide.
N2Cl

+ 2H2O + NaCl 27.[B] NaCl + H2SO4 → NaHSO4 + HCl


K2Cr2O7 + 2H2SO4 → 2KHSO4 + 2CrO3 + H2O
(CH3)2N H + Cl – N = N −HCl
→ CrO3 + 2HCl → CrO2Cl2
orange red vapour

(CH3)2N N=N 28.[C]


en + en +
17.[A] Cl Cl
O
Co Co
R – C – O – C2H5 + CH3MgBr →
:O– OC2H5 O Cl
en Cl en
  − → R – C
− C 2 H 5O CH3 Cis-d-isomer Cis-l-isomer
R CH3
CH MgX
29.[B] 2MnO −4 + 16H+ + C 2 O −4 →2Mn2+ +2CO2 +
3→ 8H2O
XMgO CH3 HO CH3 30.[C] Lanthanoid contraction takes place.
HOH / H +
  
→
R CH3 R CH3 31.[D] In nitrogen d orbital is absent.

32.[A] HNO3 is acidic in nature.


18.[C] It is cannizzaro reaction
CHO COO– CH2OH 33.[A] 2KBr + H2SO4 → K2SO4 + 2HBr
KOH
→ +
∆ 34.[A] Due to formation of chelate compound it act as
Cl Cl Cl strong acid and proceed in forward direction.
CH – O O – CH
B +H+
19.[C] Phenol is less acidic than acetic acid and CH – O O – CH
p-nitrophenol.
NaOH Na2S2O3
35.[B] Na2SO3 + S  
→
OH O – C2H5 Sod. thiosulphate
Anhy.
20.[A] + C2H5I → 36.[D] Critical temperature of water is more than O2
C 2 H 5OH
due to its dipole moment (Dipole moment of
40% H 2SO 4 water
21.[C] CH ≡ CH CH2 = CH – OH
1% HgSO 4 = 1.84 D, Dipole moment of O2 = 0D.)
Keto − enol tautomerism
       → CH3 – C – H 37.[D] By the process of zone refining, semiconductors
O like Si, Ge and Ga are purified.
Acetaldehyde

22.[C] The rate of nitration is greater in 38.[C] Half filled orbitals are more stable in
hexadeuterobenzene comparison of partial filled.

23.[D] Halogenation on alkene occurs by electrophilic 39.[A] The dipole moment of


addition. CH4 = 0
NF3 = 0.2 D
24.[A] Twisted boat is chiral as it does not have plane NH3 = 1.47 D
of symmetry. H2O = 1.85 D

25.[C] C – CH3 40.[A] Molecule existence is possible in such case


O when no. of bonding electron is greater than
Acetophenone has highest dipole moment. antibonding.

XtraEdge for IIT-JEE 115 APRIL 2010


∴ h = 0 or h = 2a
MATHEMATICS ∴ locus of P is x = 0 or x = 2a
6. [B] Equation of angle bisector of the pair of straight
1.[D] Clearly all term can Neither be positive nor
negative x 2 − y2 xy
lines is =
T1001 = sin log101000 = sin 3 > 0 a−b −h
T10001 = sin log1010000 = sin 4 > 0 ⇒ hx2 + (a – b)xy – hy2 = 0
∴ (0 < 2 < π & π < 4 < 2π) y y
2
⇒ h + (a – b) – h  = 0
2. [A] sin x < x < tan x in (0, 1) ⇒ tan–1x < x < sin–1 x x x
Altiter : Now, y = mx is one of the bisector
f(x) = sin–1x – x ∴ hm2 – (a – b)m – h = 0
1 h(m2 – 1) = (a – b)m
f '(x) = –1>0∀ x ∈ (0, 1)
1– x2 m2 −1 a−b
⇒ =
∴ f(x) is increasing function m h
∴ x > 0 ⇒ f(x) > f(0) = 0
7.[C] x2 + y2 + 2gx + 2fy + c = 0 passes through all
⇒ sin–1 x > x
the four quadrants
Similarly g(x) = x – tan–1 x is increasing fun
⇒ origin in an interior point
and x > 0 ⇒ x – tan–1x > 0
⇒c<0
3.[A] log2 cos x + log2(1 – tan x) + log2(1 + tan x)
– log2 sinx = 1 8. [A] two normal are x – 1 = 0 and y – 2 = 0, their
⇒ log2(1 – tan2x) – log2 tan x = 1 point of intersection (1, 2) is the centre & radius
(cos x > 0, sin x > 0, – 1 < tan x < 1) of circle perpendicular distance from centre
1 (1, 2) on tangent 3x + 4y = 6
⇒ (1 – tan2x) × =2 3 + 4. 2 − 6
tan x = =1
tan2x + 2 tan x – 1 = 0 9 + 16
tan x = –1 ± 2 ∴ equation of circle is
(x – 1)2 + (y – 2)2 = 12
tan x = 2 – 1 (Q 0 < tan x < 1) ⇒ x = π/8 ⇒ x2 + y2 – 2x – 4y + 4 = 0

4. [B] 9.[B] x2 – 4x + 6y + 10 = 0
P ⇒ (x – 2)2 = –6(y + 1)
tangent to the vertex is y + 1 = 0
circle drawn on focal distance as diameter
always touch the tangent at the vertex i.e. the line
y + 1 = 0.
α x2 y2
B α A 10. [B] Given ellipse is + =1
25 9
r r
a 2 − b2 4
a2 = 25, b2 = 9, e = =
a 5
AB = 2r sin α/2 ⇒ ae = 4
α ∴ Foci of ellipse are (± ae, 0) = (± 4, 0)
h = AB tan α = 2r sin tan α For hyperbola e = 2
2
⇒ 2a = 4 ⇒ a = 2
5.[C] Let vertex P be (h, k), then perpendicular Also b2 = a2(e2 – 1)
distance of P from the base x = a is |h – a| ⇒ b2 = 4 × 3 = 12
∴ Since length of the base is 2a, we have ∴ equation of hyperbola
1
× 2a|h – a| = a2 x2 y2
2 – =1
4 12
⇒ |h – a| = a (a ≠ 0)
⇒ 3x2 – y2 – 12 = 0
So h – a = – a or h – a = a

XtraEdge for IIT-JEE 116 APRIL 2010


→ →
 1 a   1 1 a 
11.[B] ∴ α & β are two mutually perpendicular = lim  + n  = lim  + + n −1 
unit vector. n →∞ n n  n →∞ n n −1 n −1 
   
→ →
∴ α × β is a unit vector perpendicular to  1 1 1 1 a 
= lim  + + .... + + 1 
→ → → → → n →∞  n n −1 2 1 1
both α & β . So we can consider α , β , α ×  
→  1 1 1 
β as î , ĵ & k̂ . = lim 1 + + + ....  {Q a1 = 1}
n →∞  1 2 n
Given vector are coplanar so  
a a c =e
1 0 1 =0
c c b − 2 ; x < 0

16.[A] 2 sgn 2x =  0 ; x = 0
⇒ a(– c) + a(c – b) + c2 = 0 ⇒ c2 = ab  2 ; x>0

12. [D] S1 = x2 + y2 + z2 + 2x – 4y – 4z – 7 = 0
centre C1 = (–1, 2, 2) and radius r1 = 4 4 x ≠ 0
f(x) = |2 sgn 2x| + 2 = 
S2 = x2 + y2 + z2 + 2x – 4y – 16z + 65 = 0 2 x = 0
centre C2 = (–1, 2, 8) radius r2 = 2 ∴ By defining f(0) = 4, f(x) will become
C1C2 = 6; r1 + r2 = 6 continuous function at x = 0 as then
∴ sphere S1 & S2 touches externaly f(0 – 0) = f(0 + 0) = 4
∴ point of contact divides C1C2 in the ratio 4 : 2 Hence at x = 0 f(x) has removable discontinuity
∴ point of contact = (–1, 2, 6)
17.[A] Q 1 < x < 2 ⇒ [x] = 1
4
13.[D] y = f(x) = 3 – π 
x 2 − 4x + 8 ∴ f(x) = cos  − x 3  = sin x3
2  2 
⇒ (3 – y)x – 4(3 – y)x + 20 – 8y = 0
f ´(x) = cos x3 . 3x2
x∈R ∴D≥0
2/3
⇒ 16(3 – y)2 – 4(3 – y) (20 – 8y) ≥ 0, y ≠ 3  π π π
⇒ f ´  3  = 3 cos .   = 0
⇒ –y2 + 5y – 6 ≥ 0; y ≠ 3  2 2 2
 
⇒ (y – 2) (y – 3) ≤ 0 ⇒ 2≤y<3
18.[C] yex = cos x ...(1)
14. [B] x > 0, g(x) is bounded yex + y1ex = – sin x ...(2)
f ( x )e nx + g( x ) again differentiating
∴ lim
n →∞ 1 + e nx yex + y1ex + y1ex + y2ex = – cos x
yex + 2y1ex + y2ex = – yex (from (1))
f ( x ) + g ( x ) / e nx
lim = f(x) ⇒ 2y + 2y1 + y2 = 0 ...(3)
n →∞ 1 + 1 e nx again differentiating
g( x ) finite 2y1 + 2y2 + y3 = 0
[g(x) is bounded ⇒ ⇒ = 0]
e nx
infinte again differentiating
2y2 + 2y3 + y4 = 0 ...(4)
15.[B] a1 = 1, an = n(1 + an–1) from (1) (2) & (3)
a 4y + y4 = 0
⇒ 1 + an–1 = n
n y4
∴ =–4
 1 1   1  y
∴ lim 1 +  1 +  .... 1 + 
n →∞
 a 1  a 2   a n 
19.[A] fog = I ⇒ fog(x) = I(x) = x
 1 + a1   1 + a 2  1+ an  ∴ fg(x) = x
lim    ....  
n →∞  a 1   a 2 ⇒ f´g(x) × g´(x) = 1
    an 
a a a n +1 1 ⇒ f´g(a) × g´(a) = 1
= lim 2 . 3 ........ . ⇒ f´(b) × 2 = 1
n →∞ 2 3 n + 1 a 1a 2 ...a n
1
a n +1 (1 + a n )(1 + n ) ⇒ f´(b) =
= lim = lim 2
n →∞ n + 1 n →∞ n +1

XtraEdge for IIT-JEE 117 APRIL 2010


20.[C] Let P(h, k) be one of point of contact then  4 
k = sin h ...(1) = (1 + cos2x)3/2 − (1 + cos 2 x ) + 2 + c
 5 
equation of tangent is y – k = cos h (x – h)
which parries through origin 2
= (1 + cos2x)3/2 (3 – 2cos2x) + c
∴ k = h cos h ...(2) 5
from (1) & (2)
k2 x2 x4
2
+ k = sin h + cos h = 1 2 2 24.[A] 1 – + – .......... = cos x
h2 3 4
⇒ k2 + k2h2 = h2 ⇒ x2 – y2 = x2y2
∴ locus of (h, k) is x2 – y2 = x2y2
∴ I= ∫ cos x dx = sin x
x
21.[B] x + y = 16, x, y > 0
s = x3 + y3 = x3 + (16 – x)3 25.[A] lim
∫ π/2
(2 −cos t − 1) dt
0
 
π x2  π 0
ds

x→
= 3x2 – 3(16 – x)2 2  t −  dt
dx π2 / 4  2
ds 2 − cos x − 1 2 − cos x ln 2. sin x ln 2
=0 lim = lim =
dx x→ 
π π x→
π 4x − π π
2  x −  .2 x 2
⇒ x2 = 256 – 32x + x2 ⇒ x = 8  2
d 2s
= 6x – 6(16 – x) = 6[2x – 16] 2 4 6
dx 2  1  n2  22  n2  32  n2
at x = 8 26.[A] y = lim 1 + 2  1 + 2  1 + 2 
n →∞  n   n   n 
d 2s d 3s
= 0 and = 12 ≠ 0 2n
dx 2 dx 3  n2  n2
.... 1 + 2 
Hence there in no minimum exist.  n 
 
1 1 2 n
 r 2  2r
− 2 − 3
22.[C] b
2
a = c = 2 log y = lim
n →∞
∑r =1
log1 + 2  . 2
 n  n
1 1 1 c
− − 2
b a c n
 r 2  2r 1
1 1 2
= lim
n →∞
∑ log1 + 2  . .
 n  n n
⇒ + = r =1
b a c 1

∫ 2x log(1 + x
2
2ab = ) dx
⇒c=
a+c 0

∴ c is H.M. of a & b 2

∫ log t dt = (t log t − t )
2
= 1


1
23.[B] I= 1 + cos 2 x sin 2x cos 2x dx
4
= 2 log 2 – 1 = log
put 1 + cos2x = t2 e
⇒ –2sin x cos x dx = 2tdt ⇒ y = 4/e
⇒ – sin 2x dx = 2tdt
27.[B] required area
∴ I=– ∫ t 2 .(2t dt ).(2 cos 2 x − 1)
1
y = 2x2
∫ t .2 t . ( 2 t
2
=– − 3) dt

 2 t 5 3t 3 
= –2  −  +c
 5 3  0 1/√2
4
=– (1 + cos2x)5/2 + 2(1 + cos2x)3/2 + c –1
5

XtraEdge for IIT-JEE 118 APRIL 2010


1/ 2 We have – b = 4ak ⇒ – b ≥ 4a
1
∫ 2x dx
2
– ⇒ |b| ≥ 4a {b < 0 ∴ |b| = –b}
2 0

1 2 3 1/ 2
33.[B] Let G.P. be a + ar + ar2 ......
⇒ – [ x ]0 G.P is infinite so – 1 < r < 1
2 3
G.P. is decreasing
1 2 1 2 2 2 ⇒ r > 0 so 0 < r < 1 and therefore a > 0
⇒ – . = =
2 3 2 2 3 2 6 f´(x) = 3x2 + 3 > 0
⇒ f(x) is strictly increasing function
28.[B] y´´ = (y´ + 3)1/3 ⇒ (y´´)3 = (y´ + 3)2 ∴ f(x)max on [–2, 3] is f(3) = 27 & f´(0) = 3
a
 d2y 
3 2 ∴ = 27 & a – ar = 3
   dy  dy 1− r
⇒ –   –6 –9=0
 dx 2  dx
  dx 2 4
  ⇒ r= or Qr<1
∴ order is 2 & degree = 3 3 3
2 2
∴r= & if r = ; a = 9
29.[A] x18 = y21 = z28 3 3
⇒ 18 log x = 21 log y = 28 log z ∴ Sum of first three terms = 9 + 6 +4 = 19
⇒ logy x = 7/6, logz y = 4/3, logxz = 9/14
Now, 3, 3 logyx, 3 logz7, 7 logx2 34.[B] If z1, z2 & z3 vertex of equilateral triangle then
7 4 9 z12 + z 22 + z 32 = z1z2 + z2z3 + z3z1
= 3, , 3 × , 7×
2 3 14 ∴ (a + i)2 + (1 + ib)2 + 0 = (a + i) (1 + ib) + 0 +
7 9 0
= 3, , 4, ⇒ a2 – 1 + 2ia + 1 – b2 + 2ib = a + iba + i – b
2 2
which are in A.P. ⇒ a2 – b2 + i(2a + 2b) = a – b + i(ab + 1)
on comparing
a2 – b2 = a – b and 2a + 2b = ab + 1
30.[B] log2x + log2y ≥ 6
Here x > 0, y > 0 ⇒ (a – b) (a + b – 1) = 0 & 2a + 2b = ab + 1
∴ log2xy ≥ 6 ⇒ xy ≥ 26 ⇒ a = b or a + b = 1 ....(1)
⇒ xy ≥ 64 ⇒ 2a + 2b = ab + 1 .... (2)
Now from (1) take a = b put in (2)
Now, A.M ≥ G.M.
2a + 2a = a2 + 1
x+y
∴ ≥ (xy)1/2 ⇒ a2 – 4a + 1 = 0 ⇒ a = 2 ± 3
2
⇒ x + y ≥ 2(64)1/2 Qa<1⇒a=2– 3
x+y≥2×8 Q a=2– 3 &a=b=2– 3
⇒ x + y ≥ 16
It we take a + b = 1 & put in (2) then it becomes
∴ (x + y)min = 16 ab = 0 which not possible because a & b lies
between 0 and 1
31.[A] f(x) = x3 + x2 + 10x + sinx
f´(x) = 3x2 + 2x + 10 + cos x
n  1  3  r  7  r 

 1
2
29
35.[A] S = ∑ (−1) r  r +   +   + .... nCr
 2  4   8  
= 3 x +  + + cos x > 0 ∀ x r =0 
 3  3 n n r
1 3
⇒ f(x) is strictly increasing = ∑ (−1)
r =0
r
2r
.n C r + ∑ (−1) . 4  . C
r =0
r n
r
Also x → ∞ ⇒ f(x) → ∞,
n r
x → – ∞ ⇒ f(x) → – ∞ 7
∴ f(x) has only one real root.
+ ∑ (−1) . 8  . C
r =0
r n
r + ....

n n n
32.[B] Let roots be (2k – 1) & (2k + 1) k ∈ N  1  3  7
= 1 −  + 1 −  + 1 −  + .....
b  2  4  8
the Sum of roots : 4k = –
a 1 1 1 1
= + + + ..... =
Q a ∈ R+, b < 0 as k ≥ 1 2 n
2 2n
2 3n
2 −1 n

XtraEdge for IIT-JEE 119 APRIL 2010


36.[B] The general term in the expansion of (x1 + x2 + 41.[C] A is idenpotent ⇒ A2 = A
n 1 x  1 x  1 3x 
...xn)n given ..... . x1p1 x p2 2 .... x pmm , A2 =     =   ≠A
p1 p 2 ... p n 0 2  0 2   0 4 
p1 + p2 + p3 .... + pm = n ∴ not possible for any x
Now in (1 + x + y – z)9, coefficient of x3y4z
= coeft of u0x3.y4z1 in (u + x + y – z)9 42.[A] for any a ∈z ⇒ a = 20a
9 ⇒aRa∀a∈z
= × (–1)1 = –2 . 9C2 . 7C3 ∴R is reflexive
0 3 4 1
a R b ⇒ a = 2kb, k ∈ z ⇒ b = a.2–k, – k ∈ z
⇒bRa
ex
37.[B] = B0 + B1x + B2x2 + ..... ∴ R is symmetric
1− x
⇒ ex = B0 – B0x + B1x – B1x2 + B2x2 – B2x2 + Let a R b, b R c ⇒ a = 2 k1 b , b = 2 k 2 c
.... a = 2 k1 2 k 2 c = 2 k1 + k 2 c , k1 + k2 ∈ z
⇒ ex = B0 + (B1 – B0)x + (B2 – B1)x2 + ..... ⇒ aRc
x2 x3 x4 ∴ R is transitive
⇒1+x+ + + + .... Hence R is an equivalence Relation.
2 3 4
= B0 + (B1 – B0)x + (B2 – B1)x2 + ...
43.[D] Q A.m ≥ G.m
∴ Bn – Bn –1 is coeff. of xn
1a b b a a b
1 ∴  +  ≥ . =1⇒ + ≥2
On comparing coeff. of xn = 2b a a b b a
n
b c a c
Similarly + ≥2& + ≥2
38.[A] x + y + z + 12 = 0 c b c a
x, y, z are negative integers Adding we get
Let x = – a, y = –b, z = – c, a b b c a c
a, b, c are +ve integer then required number of + + + + + ≥6
b a c b c a
points (x, y, z)
= Number of positive integral solution of b+c c+a a+b
⇒ + + ≥6
a + b + c = 12 a b c
= 12–1C3–1 = 11C2 = 55 ∴ minimum value is 6.

2 6 6 5 6 1 1
39.[A] p1 = p2 = 44.[A] f(x) = ⇒ f ´(x) = – 2
12 11 x x
f ( b ) − f (a )
p1 2 6 6 ∴ = f ´(x1)
= × 11 = 1 b−a
p2 12 5 6
1 1  1 
⇒ – = (b – a) − ;
f ( x ) f (1 / x ) − f ( x ) b a  x2 
 1
40.[C] 2f(x) =
1 f (1 / x ) a < x1 < b
1 1 ⇒ x12 = ab ⇒ x1 = ab
= f(x) .   – f   + f(x)
x
  x
∫ cot x dx = ∫ cot x (cosec x –1)dx
4 2 2
1 1 45.[C]
⇒ f(x) + f   = f(x) . f  
x
  x
n
= ∫ cot x cosec x dx – ∫ (cos ec x − 1) dx
2 2 2
⇒ f(x) = 1 ± x
f(2) = 17 1 3
=– cot x + cotx + x + c
⇒ 1 ± 2n = 17 ⇒ ± 24 = 16 3
∴ +ve sign will be take 1 1
⇒ 2n = 16 ⇒ n = 4 ∴f(x) = – cot3x + cotx + x + c + cot3x – cotx
3 3
Now, ∴ f(x) = 1 + x4 =x+c
⇒ f(5) = 54 + 1 = 626

XtraEdge for IIT-JEE 120 APRIL 2010


x π π π Accept :
∴ f  = + c ⇒ = +c Irrelevant meaning in respect of the sentence.
2
  2 2 2
⇒c=0 2.[B] burst : (punctured)
∴ f(x) = x Irrelevant meaning.
bust : (collapsed)
Quite relevant meaning. Hence, correct option.
LOGICAL REASONING bursted : irrelevant because this is an improper
form of the verb 'burst'.
busted :
1.[B] The given sequence is a combination of two Irrelevant because this is an incorrect form of
series: the verb 'bust'
I series : 11, 17, 23, (?) 3.[A] deduce : (to conclude) (to infer)
II series : 12, 18, 24 This word really suits to the given sentence
Pattern in both is + 6 making it a meaningful one.
So, missing number = 23 + 6 = 29 deduct : (to take away)
Irrelevant, because it means something else.
2.[B] Dum Dum is an airport in calcutta and Palam is e.g. Tax is 'deducted' from his salary every year.
an airport in Delhi. reduce : (to decrease)
Different meaning makes the sentence
3. [D] All except mustard are food grains, while meaningless. Hence, this is an incorrect option.
mustarel is an oilseed. Conduce : (to suit)
Irrelevant word.
4.[C]
4.[C] Sky (Firmament)
Irrelevant 'opposite'
5. [B] Firmament :
irrelevant 'opposite'.
nadir : (lowest point)
'Zenith' means highest point'.
6.[D] Hence this is the most suitable word in opposite,
naive (Simple)
Irrelevant word that doesn't serve any purpose.

7.[C] The 3rd figure in each row comprises of parts 5.[A] Hungry : (a voracious eater) (a voracious
which are not common to the first two figures. reader)
Wild : (Irrelevant)
8. [C] It's Synonym is 'Savage'.
Quick : (Soon) therefore, irrelevant.
Angry : (furious) Hence, irrelevant.
6.[C] Journey man : (Irrelevant)
9.[D] Because it is a person who journeys regularly on
a particular route.
Tramp : (Irrelevant)
because it means 'a vagabond' who gets about
10.[B] The number of each type of figures decrease by purposelessly.
1 at each step from left to eight in each row. Itinerant : (Relevant)
It's a person who moves from one place to
another during his travel.
ENGLISH Mendicant : (Irrelevant)
It's a religious preacher who goes from place to
place in the form of a beggar.
1.[B] Assure : (persuade that all is well)
Hence, irrelevant meaning. 7.[B] To dislocate :
Ensure : (guarantee) irrelevant meaning.
This is a relevant option as it properly suits to To lose one's temper :
the meaningful expression. Quite Relevant
Insure : (to cover against any loss) It's often used when a person is about to get
Hence, irrelevant meaning. angry.

XtraEdge for IIT-JEE 121 APRIL 2010


To take off : 11.[C] Neigh : Correct spelling as it means 'the cry of
irrelevant horse'.
To be indifferent : Reign : Correct spelling as it means 'the
Irrelevant controlling chord of an animal.'
Niece : Incorrect spelling as the correct one is
8.[A] took to : (to be accustomed to/to be addicted to) 'niece'. (Opposite of 'Nephew').
Correct because it suits to the sentence when Neither : Correct spelling. It is a conjunction to
Gandhi Ji was addicted to smoking. be used with 'nor' for one of two options.
took for : (to be mistaken while recognising)
irrelevant 12.[C] I wonder :
took in : (to deceive someone) No error in it.
irrelevant What he has done with the book.
took up : (to adopt) No error
irrelevant I lend him
(Erroneous) because there is an error of
9.[A] Get someone to break the box. 'Tenses'. The word 'lend' is to be 'lent'.
Correct answer because : No error :
• The given sentence is in Passive Voice which There is an error.
requires its answer in Active Voice.
• The given verb is Causatives in Imperative. 13.[A] Distraught, awry :
• This option is Causative Active Voice in Correct answer : 'Distraught' means to 'get
imperative form. upset' and 'awry' means in 'disorder'.
They have broken the box. Frustrated, Magnificently :
Incorrect Answer : Because Both are opposite. One is positive and the other
• The given verb is not causative. one is negative. Therefore, no meaningful
• The given sentence is not imperative. sentence.
Have the broken box. Elated, Wild :
Incorrect : Because No co-ordination, therefore incorrect answer.
• The verb is not causatives. Dejected, splendidly :
• 'Broken' has been used as an adjective in this No co-ordination, therefore incorrect answer.
option.
Break the box 14.[D] Interesting :
Incorrect because : " . . . and only a few were . . . . " phrase shows
• The subject 'you' (implied) wan't break it but that something opposite is required here. The
will get someone to break it. given option is not opposite to 'trivial'. Hence,
Irrelevant option.
10.[B] He asked how shabby I was looking. Practical :
(Incorrect option) because : Like aforesaid logic, this option also is
The required answer (type of sentence) is irrelevant.
wrong. Complex :
He exclaimed with disgust that I was looking 'Complex' can't be opposite to 'trivial'.
very shabby. Therefore, can't be relevant.
(Correct answer because) Significant :
• This option is exclamatory. This is the relevant option making the sentence
• Past Indefinite Tense has been used. quite meaningful with two contradictory words,
• The mood of the speaker is correct. i.e. trivial and 'Significant connected with the
He exclaimed with sorrow that they were phrase ". . . . . and only a few were . . . . . "
looking much shabby.
(Incorrect option because) : 15.[A] Paths, grave :
• Mood of exclamatory sentence is wrong. It is a meaningful pair of words to make the
sentence idiomatically correct.
• 'much' will be replaced with 'very'
Ways, happiness :
• 'They' won't be used as a singular subject is
Not a meaningful pair. Hence, irrelevant option.
required.
Acts, prosperity :
He told that I was looking much shabby.
Not a meaningful pair. Hence, irrelevant option.
Incorrect answer because –
Achievements, Suffering :
• Type of sentence is assertive whereas the Not a meaningful pair. Hence, irrelevant option.
required type is 'exclamatory'.
• 'much' is to be replaced with 'very'. Hence, inappropriate

XtraEdge for IIT-JEE 122 APRIL 2010


XtraEdge for IIT-JEE 123 APRIL 2010
CAREER POINT
In association
with

Launches Online Test Series for

BIT-SAT
[ For Admission to BITS Pilani, Hyderabad, Goa ]

Please fill the following form to enroll

Application Form

Student Name : __________________________________________________


Paste your
Recent
Father’s Name : __________________________________________________ Passport size
Photograph
Current Institute/Batch : __________________________________________________

Current Class : __________________________________________________

Date of Birth : __________________________________________________

Address (Local) : _________________________________________________________________________

_________________________________________________________________________

City : ___________________________________________ Pin : _________________________________________

Email : ________________________________________________________________________________________

Are you already a user of a2zExam.com? Yes/No (Mention User Name, if known_________________________________

Send the above filled form to


BIT SAT
C/o Career Point, CP Tower, Road No.1, IPIA, Kota Rajasthan – 324005
Phone : 0744-3040000, Fax : 0744 -3040050

XtraEdge for IIT-JEE 124 APRIL 2010

You might also like